Sie sind auf Seite 1von 34

Title : G.R. No.

L-2068 October 20, 1948 On April 23, 1952, before the trial of said criminal cases, petitioner filed a motion praying that assessors be appointed to assist the court in considering the questions of fact involved in said
cases as authorized by section 49 of Republic Act No. 409, otherwise known as Revised Charter of the City of Manila, which provides that "the aid of assessors in the trial of any civil or
criminal action in the Municipal Court, or the Court of First Instance, within the City, may be invoked in the manner provided in the Code of Civil Procedure." This motion was opposed by the
DOMINADOR B. BUSTOS, petitioner, vs. ANTONIO G. LUCERO, Judge of First Instance of Pampanga, respondent. City Fiscal who appeared for the People of the Philippines.

Topic: Remedial Law RTC denied Motion :

The petitioner herein, an accused in a criminal case, filed a motion with the Court of First Instance of Pampanga after he had been bound over to that court for trial, praying that the record of On April 28, 1952, the court issued an order denying the motion holding in effect that with the promulgation of the Rules of Court by the Supreme Court, which became effective on July 1,
the case be remanded to the justice of the peace court of Masantol, the court of origin, in order that the petitioner might cross-examine the complainant and her witnesses in connection with 1940, all rules concerning pleading, practice and procedure in all courts of the Philippines previously existing were not only superseded but expressly repealed, that the Supreme Court,
their testimony, on the strength of which warrant was issued for the arrest of the accused. having been vested with the rule-making power, expressly omitted the portions of the Code of Civil Procedure regarding assessors in said Rules of Court, and that the reference to said
statute by section 49 of Republic Act No. 409 on the provisions regarding assessors should be deemed as a mere surplusage. Believing that this order is erroneous, petitioner now comes to
RTC: this court imputing abuse of discretion to the respondent Judge.
The motion was denied

SC
Denial is the subject matter of this proceeding. Issues raised by the petioner:
I. The right of the petitioner to a trial with the aid of assessors is an absolute substantive right, and the duty of the court to provide assessors is mandatory.
Petioner’s Position: II. The right to trial with the aid of assessors, being a substantive right, cannot be impaired by this court in the exercise of its rule-making power.
III. Section 154 of the Code of Civil Procedure and Section 2477 of the Old Charter of Manila, creating the right to trial with the aid of assessors, are substantive law and were not repealed
According to the memorandum submitted by the petitioner's attorney to the Court of First Instance in support of his motion, the accused, assisted by counsel, appeared at the preliminary by Rules of Court.
investigation. In that investigation, the justice of the peace informed him of the charges and asked him if he pleaded guilty or not guilty, upon which he entered the plea of not guilty. "Then his IV. Granting without admitting that the provisions on assessors of the Code of Civil Procedure and the old Charter of Manila were impliedly repealed, nevertheless, the same provisions were
counsel moved that the complainant present her evidence so that she and her witnesses could be examined and cross-examined in the manner and form provided by law." The fiscal and later reenacted by reference in section 49 of the Revised Charter of Manila, which is now the source of the right to trial with the aid of assessors and which refers to the Code of Civil
the private prosecutor objected, invoking section 11 of rule 108, and the objection was sustained. "In view thereof, the accused's counsel announced his intention to renounce his right to Procedure merely to indicate the procedure for appointing assessors.
present evidence," and the justice of the peace forwarded the case to the court of first instance. V. Section 49 of the Revised Charter of Manila is not invalid class legislation and does not violate the constitutional provision that the rules of pleading, practice and procedure 'shall be
uniform for all the courts of the same grade.
Issue: Issue raised by respondent:
Respondents now contend that the right to trial with the aid of assessors, with all its concomitant provisions, cannot now be invoked because, being procedural in nature, the same must be
Whether the accused, after renouncing his right to present evidence, and by reason of that waiver he was committed to the corresponding court for trial, is estopped. deemed to have been impliedly eliminated.

Ruling: Ruling: Petition Granted

No. This respondent’s claim would be correct if we were to hold that the right to trial with the aid of assessors is not substantive but procedural or adjective in nature. If it were merely procedural,
not having been incorporated in the Rules of Court, the logical conclusion is that the rule- making power has deemed wise to eliminate it.
In Dequito and Saling Buhay vs. Arellano, G.R. No. L- 1336, r, in which case the respondent justice of the peace had allowed the accused, over the complaint's objection, to recall the
complainant and her witnesses at the preliminary investigation so that they might be cross-examined, SC sustained the justice of the peace's order. SC said that section 11 of Rule 108 does
not curtail the sound discretion of the justice of the peace on the matter.SC said that "while section 11 of Rule 108 defines the bounds of the defendant's right in the preliminary investigation, The trial with the aid of assessors as granted by section 154 of the Code of Civil Procedure and section 2477 of the old Charter of Manila are parts of substantive law and as such are not
there is nothing in it or any other law restricting the authority, inherent in a court of justice, to pursue a course of action reasonably calculated to bring out the truth." embraced by the rule-making power of the Supreme Court. This is so because in said section 154 this matter is referred to as a right given by law to a party litigant. Section 2477 of the
The "defendant can not, as a matter of right, compel the complaint and his witnesses to repeat in his presence what they had said at the preliminary examination before the issuance of the Administrative Code of 1917 is couched is such a manner that a similar right is implied when invoked by a party litigant. It says that the aid may be invoked in the manner provided in the
order of arrest.The constitutional right of an accused to be confronted by the witnesses against him does not apply to preliminary hearings' nor will the absence of a preliminary examination Code of Civil Procedure. And this right has been declared absolute and substantial by this Court in several cases where the aid of assessors had been invoked (Berbari vs. Concepcion, et
be an infringement of his right to confront witnesses." As a matter of fact, preliminary investigation may be done away with entirely without infringing theconstitutional right of an accused al., 40 Phil., 320; Colegio de San Jose vs. Sison, 54 Phil., 344.) Thus, it was there said that these provisions "necessarily lead to the conclusion that the intervention of the assessors is not
under the due process clause to a fair trial. an empty formality which may be disregarded without violating either the letter or the spirit of the law. It is another security given by the law to the litigants, and as such, it is a substantial
right of which they cannot be deprived without vitiating all the proceedings. Were we to agree that for one reason or another the trial by assessors may be done away with, the same line of
reasoning would force us to admit that the parties litigant may be deprived of their right to be represented by counsel, to appear and be present at the hearings, and so on, to the extent of
In another resolution: GR No. L-2068, Mar 08, 1949 DOMINADOR B. BUSTOS v. ANTONIO G. LUCERO omitting the trial in a civil case, and thus set at naught the essential rights granted by the law to the parties, with consequent nullity of the proceedings." (Colegio de San Jose vs. Sison, 54
Phil., 344, 349.)
Issue:
It is contended that Section 11 of Rule 108 of the Rules of Court infringes Section 13, Article VIII, of the Constitution. It is said that the rule in question deals with substantive matters and
impairs substantive rights.

Ruling:
Title: G.R. No. 141524 September 14, 2005
SC disagrees . The court is of the opinion that Section 11 of Rule 108, like its predecessors, is an adjective law and not a substantive law or substantive right. Substantive law creates
substantive rights and the two terms in this respect may be said to be synonymous. Substantive rights is a term which includes those rights which one enjoys under the legal system prior to
the disturbance of normal relations. (60 C. J. 980.) Substantive law is that part of the law which creates, defines and regulates rights, or which regulates the rights and duties which give rise
to a cause of action; that part of the law which courts are established to administer; as opposed to adjective or remedial law, which prescribes the method of enforcing rights or obtains DOMINGO NEYPES, LUZ FAUSTINO, ROGELIO FAUSTINO, LOLITO VICTORIANO, JACOB OBANIA AND DOMINGO CABACUNGAN, Petitioners,
redress for their invasion. (36 C. J. 27; 52 C. J. S. 1026.)

vs.
HON. COURT OF APPEALS, HEIRS OF BERNARDO DEL MUNDO, namely: FE, CORAZON, JOSEFA, SALVADOR and CARMEN, all surnamed DEL MUNDO, LAND BANK OF
Title : G.R. No. L-6120 THE PHILIPPINES AND HON. ANTONIO N. ROSALES, Presiding Judge, Branch 43, Regional Trial Court, Roxas, Oriental Mindoro, Respondent.

June 30, 1953 Topic: Remedial Law

Petitioners Domingo Neypes, Luz Faustino, Rogelio Faustino, Lolito Victoriano, Jacob Obania and Domingo Cabacungan filed an action for annulment of judgment and titles of land and/or
CIPRIANO P. PRIMICIAS, petitioner, reconveyance and/or reversion with preliminary injunction before the Regional Trial Court, Branch 43, of Roxas, Oriental Mindoro, against the Bureau of Forest Development, Bureau of
vs. Lands, Land Bank of the Philippines and the heirs of Bernardo del Mundo, namely, Fe, Corazon, Josefa, Salvador and Carmen.
FELICISIMO OCAMPO, as Judge-at-large presiding over Branch C of the Court of First Instance of Manila and EUGENIO ANGELES, as City Fiscal of Manila, representing the
PEOPLE OF THE PHILIPPINES, respondents.
RTC:
In the course of the proceedings, the parties (both petitioners and respondents) filed various motions with the trial court. Among these were: (1) the motion filed by petitioners to declare the
respondent heirs, the Bureau of Lands and the Bureau of Forest Development in default and (2) the motions to dismiss filed by the respondent heirs and the Land Bank of the Philippines,
Topic: Remedial Law respectively.

This is a petition which seeks to prohibit respondent Judge from proceeding with the trial of two criminal cases which were then pending against petitioner without the assistance of In an order dated May 16, 1997, the trial court, presided by public respondent Judge Antonio N. Rosales, resolved the foregoing motions as follows: (1) the petitioners ’ motion to declare
assessors in accordance with the provisions of section 49 of Republic Act No. 409 in relation to section 154 of Act No. 190, and as an auxiliary remedy, to have a writ of preliminary injunction respondents Bureau of Lands and Bureau of Forest Development in default was granted for their failure to file an answer, but denied as against the respondent heirs of del Mundo because
issued so that the trial may be held pending until further orders of this court. the substituted service of summons on them was improper; (2) the Land Bank ’s motion to dismiss for lack of cause of action was denied because there were hypothetical admissions and
matters that could be determined only after trial, and (3) the motion to dismiss filed by respondent heirs of del Mundo, based on
Petition was originally filed with CA prescription, was also denied because there were factual matters that could be determined only after trial.
This petition was originally filed with the Court of Appeals, but was later certified to this court on the ground that the main basis of the petition is section 49 of Republic Act No. 409, otherwise
known as Revised Charter of the City of Manila, approved on June 18, 1949, and respondents assail the constitutionality of said section in that it contravenes the constitutional provision that MR filed:
the rules of court "shall be uniform for all courts of the same grade . . . . (Section 13, Article VIII of the Constitution.).
The respondent heirs filed a motion for reconsideration of the order denying their motion to dismiss on the ground that the trial court could very well resolve the issue of prescription from the
RTC: bare allegations of the complaint itself without waiting for the trial proper.

Petitioner was charged before the Court of First Instance of Manila with two statutory offenses, namely, (1) with a violation of Commonwealth Act No. 606, which was docketed as criminal RTC decision:
case No. 18374, in that he knowingly chartered a vessel of Philippine registry to an alien without the approval of the President of the Philippines and (2) with a violation of section 129 in In an order dated February 12, 1998, the trial court dismissed petitioners’ complaint on the ground that the action had already prescribed. Petitioners allegedly received a copy of the order of
relation to section 2713 of the Revised Administrative Code, which was docketed as Criminal Case No. 18375, in that he failed to submit to the Collector of Customs the manifests and dismissal on March 3, 1998 and, on the 15th day thereafter or on March 18, 1998, filed a motion for reconsideration. On July 1, 1998, the trial court issued another order dismissing the
certain authenticated documents for the vessel "Antarctic" and failed to obtain the necessary clearance from the Bureau of Customs prior to the departure of said vessel for a foreign port. motion for reconsideration which petitioners received on July 22, 1998. Five days later, on July 27, 1998, petitioners filed a notice of appeal and paid the appeal fees on August 3, 1998.

Before Trial of RTC petitioner filed motion: Notice of appeal denied :


On August 4, 1998, the court a quo denied the notice of appeal, holding that it was filed eight days late. This was received by petitioners on July 31, 1998. Petitioners filed a motion for
reconsideration but this too was denied in an order dated September 3, 1998. Title :G.R. No. L-286 March 29, 1946

CA :

Petitioners position: FREDESVINDO S. ALVERO, petitioner,


In the appellate court, petitioners claimed that they had seasonably filed their notice of appeal. They argued that the 15-day reglementary period to appeal started to run only on July 22, vs.
1998 since this was the day they received the final order of the trial court denying their motion for reconsideration. When they filed their notice of appeal on July 27, 1998, only five days had M.L. DE LA ROSA, Judge of First Instance of Manila, JOSE R. VICTORIANO, and MARGARITA VILLARICA, respondents.
elapsed and they were well within the reglementary period for appeal.

Ca Decision: Topic: Remedial Law

On September 16, 1999, the Court of Appeals (CA) dismissed the petition. It ruled that the 15-day period to appeal should have been reckoned from March 3, 1998 or the day they received Facts:
the February 12, 1998 order dismissing their complaint. According to the appellate court, the order was the "final order" appealable under the Rules.
This is an original petition for certiorari filed in this court.
MR filed.
Petitioners filed a motion for reconsideration of the aforementioned decision. This was denied by the Court of Appeals on January 6, 2000. RTC:

Respondent:
Issue: The record shows that, on June 25, 1945, respondent Jose R. Victoriano had filed a complaint, in the Court of First Instance of the City of Manila, against petitioner Fredesvindo S. Alvero
and one Margarita Villarica, alleging two causes of action, to wit, (1) to declare in force the contract of sale, made on October 1, 1940, between said Jose R. Victoriano and Margarita
In this present petition for review under Rule 45 of the Rules, petitioners ascribe the following errors allegedly committed by the appellate court: Villarica, of two (2) parcels of land in the Manotoc subdivision, Balintawak, in the barrio of Calaanan, municipality of Caloocan, Province of Rizal, with a combined area of 480 square meters,
I which land was subsequently sold by said Villarica, in favor of petitioner Fredesvindo S. Alvero, on December 31, 1944, for the sum of P100,000 in Japanese military notes; and (2) to
THE HONORABLE COURT OF APPEALS ERRED IN DISMISSING THE PETITIONERS’ PETITION FOR declare said subsequent sale null and void.
CERTIORARI AND MANDAMUS AND IN AFFIRMING THE ORDER OF THE HON. JUDGE ANTONIO N. ROSALES WHICH DISMISSED THE PETITIONERS ’ APPEAL IN CIVIL CASE NO.
C-36 OF THE REGIONAL TRIAL COURT, BRANCH 43, ROXAS, ORIENTAL MINDORO, EVEN AFTER THE PETITIONERS HAD PAID THE APPEAL DOCKET FEES. On July 7, 1945, Margarita Villarica filed an answer to said complaint, expressly admitting having sold said land to Fresdesvindo S. Alvero, for P100,000, in December, 1944, due to the
II imperative necessity of raising funds with which to provide for herself and family, and that she did not remember the previous sale; at the same time, offering to repurchase said land from
THE HONORABLE COURT OF APPEALS LIKEWISE ERRED IN RULING AND AFFIRMING THE DECISION OR ORDER OF THE RESPONDENT HON. ANTONIO M. ROSALES THAT Fredesvindo S. Alvero in the sum of P5,000, but that the latter refused to accept the offer.
PETITIONERS’ APPEAL WAS FILED OUT OF TIME WHEN PETITIONERS RECEIVED THE LAST OR FINAL ORDER OF THE COURT ON JULY 22, 1998 AND FILED THEIR NOTICE OF Respondent Counterclaim:
APPEAL ON JULY 27, 1998 AND PAID THE APPEAL DOCKET FEE ON AUGUST 3, 1998. On July 13, 1945, Fredesvindo S. Alvero, in answering said complaint, denied the allegations made therein, and claimed exclusive ownership of the land in question, and at the same time
III set up a counterclaim and crossclaim in his answer, demanding from Jose R. Victoriano a P200-monthly rent on said property, beginning from February, 1945, plus P2,000 as damages.
THE HONORABLE COURT OF APPEALS FURTHER ERRED IN RULING THAT THE WORDS "FINAL ORDER" IN SECTION 3, RULE 41, OF THE 1997 RULES OF CIVIL PROCEDURE
WILL REFER TO THE [FIRST] ORDER OF RESPONDENT JUDGE HON. ANTONIO M. MORALES DATED FEBRUARY 12, 1998 INSTEAD OF THE LAST AND FINAL ORDER DATED RTC Decision:
JULY 1, 1998 COPY OF WHICH WAS RECEIVED BY PETITIONERS THROUGH COUNSEL ON JULY 22, 1998.
IV. Considering that Jose R. Victoriano's document was older than that of Fredesvindo S. Alvero, and that he had taken possession of said property, since October 1, 1940
THE HONORABLE COURT OF APPEALS FINALLY ERRED IN FINDING THAT THE DECISION IN THE CASE OF DENSO, INC. V. IAC, 148 SCRA 280, IS APPLICABLE IN THE INSTANT The respondent judge rendered his decision in favor of Jose R. Victoriano, adjudging to him the title over the property in question, including all the improvements existing thereon, and
CASE THEREBY IGNORING THE PECULIAR FACTS AND CIRCUMSTANCES OF THIS CASE AND THE FACT THAT THE SAID DECISION WAS dismissed the counterclaim.
RENDERED PRIOR TO THE ENACTMENT OF THE 1997 RULES OF CIVIL PROCEDURE.
Subsequent Events:
On November 28, 1945, Fredesvindo S. Alvero was notified of said decision; and on December 27, 1945, he filed a petition for reconsideration and new trial, which was denied on January 3,
Note: 1946; and of said order he was notified on January 7, 1946.
The foregoing issues essentially revolve around the period within which petitioners should have filed their notice of appeal. On January 8, 1946, Fredesvindo S. Alvero filed his notice of appeal and record on appeal simultaneously in the lower court, without filing the P60-appeal bond.
On January 14, 1946, Jose R. Victoriano filed a petition to dismiss the appeal, and at the same time, asked for the execution of the judgment.
Ruling: On January 15, 1946, Fredesvindo S. Alvero filed an opposition to said motion to dismiss, alleging that on the very same day, January 15, 1946, said appeal bond for P60 had been actually
filed, and allege as an excuse, for not filing the said appeal bond, in due time, the illness of his lawyer's wife, who died on January 10, 1946, and buried the following day.
Supreme Court Granted the petition . On January 17, 1946, the respondent judge, Hon. Mariano L. de la Rosa, ordered the dismissal of the appeal, declaring that, although the notice of appeal and record on appeal had been
filed in due time, the P60-appeal bond was filed too late.
Based on the foregoing, an appeal should be taken within 15 days from the notice of judgment or final order appealed from. A final judgment or order is one that finally disposes of a case, On January 23, 1946, Fredesvindo S. Alvero filed a petition for the reconsideration of the said order dated January 17, 1946, dismissing his appeal; and said petition for reconsideration was
leaving nothing more for the court to do with respect to it. It is an adjudication on the merits which, considering the evidence presented at the trial, declares categorically what the rights and denied on January 29, 1946.
obligations of the parties are; or it may be an order or judgment that dismisses an action.

As already mentioned, petitioners argue that the order of July 1, 1998 denying their motion for reconsideration should be construed as the "final order," not the February 12, 1998 order SC:
which dismissed their complaint. Since they received their copy of the denial of their motion for reconsideration only on July 22, 1998, the 15-day reglementary period to appeal had not yet
lapsed when they filed their notice of appeal on July 27, 1998. Petioner filed petition for certiorari.

What therefore should be deemed as the "final order," receipt of which triggers the start of the 15-day reglementary period to appeal 3 ⁄4 the February 12, 1998 order dismissing the Respondent:
complaint or the July 1, 1998 order dismissing the MR? On February 11, 1946, the respondents filed their answer to the petition for certiorari, alleging (1) that said petition is defective in form as well as in substance; (2) that there has been no
excusable negligence, on the part of the petitioner, or grave abuse of discretion on the part of the respondent judge, in the instant case.
In the recent case of Quelnan v. VHF Philippines, Inc., the trial court declared petitioner Quelnan non-suited and accordingly dismissed his complaint. Upon receipt of the order of dismissal,
he filed an omnibus motion to set it aside. When the omnibus motion was filed, 12 days of the 15-day period to appeal the order had lapsed. He later on received another order, this time
dismissing his omnibus motion. He then filed his notice of appeal. But this was likewise dismissed ― for having been filed out of time. Issue: Whether petioner filed the appeal on time

The court a quo ruled that petitioner should have appealed within 15 days after the dismissal of his complaint since this was the final order that was appealable under the Rules. We Ruling:
reversed the trial court and declared that it was the denial of the motion for reconsideration of an order of dismissal of a complaint which constituted the final order as it was what ended the
issues raised there. No. As already stated, the decision rendered by the respondent judge, Hon. Mariano L. de la Rosa, was dated November 16, 1945, of which counsel for Fredesvindo S. Alvero was notified
on November 28, 1945; that his motion for reconsideration and new trial was filed on December 27, 1945, and denied on January 3, 1946, and that said counsel for Alvero was notified of
In de la Rosa v. Court of Appeals, we stated that, as a rule, periods which require litigants to do certain acts must be followed unless, under exceptional circumstances, a delay in the filing of said order on January 7, 1946; and that he filed his notice of appeal and record on appeal the following day, to wit, January 8, 1946, and that the P60-appeal bond was filed only on January
an appeal may be excused on grounds of substantial justice. There, we condoned the delay incurred by the appealing party due to strong considerations of fairness and justice. 15, 1946.
According to the computation erroneously made by the court, the last day for filing and perfecting the appeal, in this case, was January 8, 1946, or which date, Fredesvindo S. Alvero should
In setting aside technical infirmities and thereby giving due course to tardy appeals, we have not been oblivious to or unmindful of the extraordinary situations that merit liberal application of have filed his (1) notice of appeal, (2) record on appeal, and (3) appeal bond. But the P60-appeal bond was filed only on January 15, 1946.
the Rules. In those situations where technicalities were dispensed with, our decisions were not meant to undermine the force and effectivity of the periods set by law. But we hasten to add Failure to perfect the appeal, within the time prescribed by the rules of court, will cause the judgment to become final, and the certification of the record on appeal thereafter, cannot restore
that in those rare cases where procedural rules were not stringently applied, there always existed a clear need to prevent the commission of a grave injustice. Our judicial system and the the jurisdiction which has been lost. (Roman Catholic Bishop of Tuguegarao vs. Director of Lands, 34 Phil., 623; Estate of Cordoba and Zarate vs. Alabado, 34 Phil., 920; and Bermudez vs.
courts have always tried to maintain a healthy balance between the strict enforcement of procedural laws and the guarantee that every litigant be given the full opportunity for the just and Director of Lands, 36 Phil., 774.)
proper disposition of his cause.
Rules of courts, promulgated by authority of law, have the force and effect of law; and rules of court prescribing the time within which certain acts must be done, or certain proceedings taken,
are considered absolutely indispensable to the prevention of needless delays and to the orderly and speedy discharge of judicial business. (Shioji vs. Harvey, 43 Phil., 333.)
The Supreme Court may promulgate procedural rules in all courts.It has the sole prerogative to amend, repeal or even establish new rules for a more simplified and inexpensive process, Strict compliance with the rules of court has been held mandatory and imperative, so that failure to pay the docket fee in the Supreme Court, within the period fixed for that purpose, will
and the speedy disposition of cases. In the rules governing appeals to it and to the Court of Appeals, particularly Rules 42, 43 and 45, the Court allows extensions of time, based on cause the dismissal of the appeal. (Salaveria vs. Albindo, 39Phil., 922.) In the same manner, on failure of the appellant in a civil case to serve his brief, within the time prescribed by said
justifiable and compelling reasons, for parties to file their appeals. These extensions may consist of 15 days or more. rules, on motion of the appellee and notice to the appellant, or on its own motion, the court may dismiss the appeal. (Shioji vs. Harvey, 43 Phil., 333.)
To standardize the appeal periods provided in the Rules and to afford litigants fair opportunity to appeal their cases, the Court deems it practical to allow a fresh period of 15 days within
which to file the notice of appeal in the Regional Trial Court, counted from receipt of the order dismissing a motion for a new trial or motion for reconsideration. Henceforth, this "fresh period Held: Petition for certiorari dismissed.
rule" shall also apply to Rule 40 governing appeals from the Municipal Trial Courts to the Regional Trial Courts; Rule 42 on petitions for review from the Regional Trial Courts to the Court of
Appeals; Rule 43 on appeals from quasi-judicial agencies to the Court of Appeals and Rule 45 governing appeals by certiorari to the Supreme Court. The new rule aims to regiment or make
the appeal period uniform, to be counted from receipt of the order denying the motion for new trial, motion for reconsideration (whether full or partial) or any final order or resolution.
Jose v. Javellana, G.R. No. 158239, January 25, 2012
SC holds that petitioners seasonably filed their notice of appeal within the fresh period of 15 days, counted from July 22, 1998 (the date of receipt of notice denying their motion for
reconsideration). This pronouncement is not inconsistent with Rule 41, Section 3 of the Rules which states that the appeal shall be taken within 15 days from notice of judgment or final order TOPIC: Nature and purpose of procedural law
appealed from. The use of the disjunctive word "or" signifies disassociation and independence of one thing from another. It should, as a rule, be construed in the sense in which it ordinarily
implies. Hence, the use of "or" in the above provision supposes that the notice of appeal may be filed within 15 days from the notice of judgment or within 15 days from notice of the "final FACTS:
order," which we already determined to refer to the July 1, 1998 order denying the motion for a new trial or reconsideration. On September 8, 1979, Margarita Marquez Alma Jose (Margarita) sold for ₱160,000.00 to respondent Ramon Javellana by deed of conditional sale two parcels of land located in Barangay
Mallis, Guiguinto, Bulacan.

Agreement:
1. Javellana would pay ₱80,000.00 upon the execution of the deed and the balance of ₱80,000.00 upon the registration of the parcels of land under the Torrens System by
Margarita; The seemingly correct insistence of Priscilla cannot be upheld, however, considering that the Court meanwhile adopted the fresh period rule in Neypes v. Court of Appeals, by which an
2. Should Margarita become incapacitated, her son and attorney-in-fact, Juvenal M. Alma Jose (Juvenal), and her daughter, petitioner Priscilla M. Alma Jose, would receive the aggrieved party desirous of appealing an adverse judgment or final order is allowed a fresh period of 15 days within which to file the notice of appeal in the RTC reckoned from receipt of the
payment of the balance and proceed with the application for registration. order denying a motion for a new trial or motion for reconsideration, to wit:

The Supreme Court may promulgate procedural rules in all courts. It has the sole prerogative to amend, repeal or even establish new rules for a more simplified and inexpensive process,
After Margarita died and with Juvenal having predeceased Margarita without issue, the vendor’s undertaking fell on the shoulders of Priscilla, being Margarita’s sole surviving heir. and the speedy disposition of cases. In the rules governing appeals to it and to the Court of Appeals, particularly Rules 42, 43 and 45, the Court allows extensions of time, based on
justifiable and compelling reasons, for parties to file their appeals. These extensions may consist of 15 days or more.
Cause of the dispute:
Priscilla did not comply with the undertaking to cause the registration of the properties under the Torrens System, and, instead, began to improve the properties by dumping filling materials The fresh period rule may be applied to this case, for the Court has already retroactively extended the fresh period rule to "actions pending and undetermined at the time of their passage
therein with the intention of converting the parcels of land into a residential or industrial subdivision. and this will not violate any right of a person who may feel that he is adversely affected, inasmuch as there are no vested rights in rules of procedure." 27According to De los Santos v. Vda. de
Mangubat:28
Case originally filed:
Javellana commenced on February 10, 1997 an action for specific performance, injunction, and damages against Priscilla in the RTC in Malolos, Bulacan; entitled Ramon C. Javellana, Procedural law refers to the adjective law which prescribes rules and forms of procedure in order that courts may be able to administer justice. Procedural laws do not come
represented by Atty. Guillermo G. Blanco v. Priscilla Alma Jose. within the legal conception of a retroactive law, or the general rule against the retroactive operation of statues ― they may be given retroactive effect on actions pending and
undetermined at the time of their passage and this will not violate any right of a person who may feel that he is adversely affected, insomuch as there are no vested rights in
Plaintiff’s averments: rules of procedure.
Xxxxxxxx
1. Upon the execution of the deed of conditional sale, he had paid the initial amount of ₱80,000.00 and had taken possession of the parcels of land; WHEREFORE, the Court DENIES the petition for review on certiorari; AFFIRMS the decision promulgated on November 20, 2002; and ORDERS the petitioner to pay the costs of suit.
2. That he had paid the balance of the purchase price to Juvenal on different dates upon Juvenal’s representation that Margarita had needed funds for the expenses of
registration and payment of real estate tax; and
3. That in 1996, Priscilla had called to inquire about the mortgage constituted on the parcels of land; and
4. That he had told her then that the parcels of land had not been mortgaged but had been sold to him.
Panay Railways, Inc. v. Heva Management,

TOPIC: Retroactive application of procedural rules


Plaintiff’s prayer:
FACTS:
1. Issuance of a TRO or writ of preliminary injunction to restrain Priscilla from dumping filling materials in the parcels of land; and
Agreement:
2. That Priscilla be ordered to institute registration proceedings and then to execute a final deed of sale in his favor.
On 20 April 1982, petitioner Panay Railways Inc., a government-owned and controlled corporation, executed a Real Estate Mortgage Contract covering several parcels of lands, including
Lot No. 6153, in favor of Traders Royal Bank (TRB) to secure ₱ 20 million worth of loan and credit accommodations. Petitioner excluded certain portions of Lot No. 6153: that already sold to
Shell Co., Inc. referred to as 6153-B, a road referred to as 6153-C, and a squatter area known as 6153-D
Defendant’s answer:
Cause of the dispute:
Priscilla filed a motion to dismiss, stating that the complaint was already barred by prescription; and that the complaint did not state a cause of action.
Petitioner failed to pay its obligations to TRB, prompting the bank to extra-judicially foreclose the mortgaged properties including Lot No. 6153. On 20 January 1986, a Certificate of Sale was
issued in favor of the bank as the highest bidder and purchaser. Consequently, the sale of Lot No. 6153 was registered with the Register of Deeds on 28 January 1986 and annotated at the
RTC:
back of the transfer certificates of title (TCT) covering the mortgaged properties.
Thereafter, TRB caused the consolidation of the title in its name on the basis of a Deed of Sale and an Affidavit of Consolidation after petitioner failed to exercise the right to redeem the
1. The RTC initially denied Priscilla’s motion to dismiss. properties. The corresponding TCTs were subsequently issued in the name of the bank.
2. Upon her MR, the RTC on June 24, 1999 reversed itself and granted the motion to dismiss.
Basis/Reason: Javellana had no cause of action against her due to her not being bound to comply with the terms of the deed of conditional sale for not Case originally filed:
being a party thereto; that there was no evidence showing the payment of the balance; that he had never demanded the registration of the land from Margarita or Juvenal, or On 12 February 1990, TRB filed a Petition for Writ of Possession against petitioner. During the proceedings, petitioner, through its duly authorized manager and officer-in-charge and with the
brought a suit for specific performance against Margarita or Juvenal; and that his claim of paying the balance was not credible. assistance of counsel, filed a Manifestation and Motion to Withdraw Motion for Suspension of the Petition for the issuance of a writ of possession
3. Javellana moved for reconsideration -- contentions: It was only in 1994 that petitioner realized that the extrajudicial foreclosure included some excluded properties in the mortgage contract. Thus, on 19 August 1994, it filed a Complaint for
a. that the presentation of evidence of full payment was not necessary at that stage of the proceedings (Nonetheless, he attached to the motion for Partial Annulment of Contract to Sell and Deed of Absolute Sale with Addendum; Cancellation of Title No. T-89624; and Declaration of Ownership of Real Property with Reconveyance plus
reconsideration the receipts showing the payments made to Juvenal.); Damages.3
b. that in resolving a motion to dismiss on the ground of failure to state a cause of action, the facts alleged in the complaint were hypothetically admitted and It then filed an Amended Complaint4 on 1 January 1995 and again filed a Second Amended Complaint 5 on 8 December 1995.
only the allegations in the complaint should be considered in resolving the motion; and
c. Priscilla could no longer succeed to any rights respecting the parcels of land because he had meanwhile acquired absolute ownership of them; and that the Defendant’s answer:
only thing that she, as sole heir, had inherited from Margarita was the obligation to register them under the Torrens System. Meanwhile, respondents filed their respective Motions to Dismiss on these grounds: (1) petitioner had no legal capacity to sue; (2) there was a waiver, an abandonment and an
4. On June 21, 2000, the RTC denied the motion for reconsideration. extinguishment of petitioner’s claim or demand; (3) petitioner failed to state a cause of action; and (4) an indispensable party, namely TRB, was not impleaded.
Basis/reason: Lack of any reason to disturb the order of June 24, 1999.
5. Javellana filed a notice of appeal. RTC:
On 18 July 1997, the RTC issued an Order 6 granting the Motion to Dismiss of respondents. It held that the Manifestation and Motion filed by petitioner was a judicial admission of TRB’s
ownership of the disputed properties. The trial court pointed out that the Manifestation was executed by petitioner’s duly authorized representative with the assistance of counsel. This
CA: admission thus operated as a waiver barring petitioner from claiming otherwise.
1. Priscilla countered that the June 21, 2000 order was not appealable; that the appeal was not perfected on time ( Priscilla insists that Javellana filed his notice of appeal out of On 11 August 1997, petitioner filed a Notice of Appeal without paying the necessary docket fees. Immediately thereafter, respondents filed a Motion to Dismiss Appeal on the
time. She points out that he received a copy of the June 24, 1999 order on July 9, 1999, and filed his motion for reconsideration on July 21, 1999 (or after the lapse of 12 ground of nonpayment of docket fees.
days); that the RTC denied his motion for reconsideration through the order of June 21, 2000, a copy of which he received on July 13, 2000; that he had only three days
from July 13, 2000, or until July 16, 2000, within which to perfect an appeal; and that having filed his notice of appeal on July 19, 2000, his appeal should have been In its Opposition,7 petitioner alleged that its counsel was not yet familiar with the revisions of the Rules of Court that became effective only on 1 July 1997. Its representative was likewise not
dismissed for being tardy by three days beyond the expiration of the reglementary period).; and that Javellana was guilty of forum shopping. (It appears that pending the informed by the court personnel that docket fees needed to be paid upon the filing of the Notice of Appeal. Furthermore, it contended that the requirement for the payment of docket fees was
appeal, Javellana also filed a petition for certiorari in the CA to assail the June 24, 1999 and June 21, 2000 orders dismissing his complaint.) not mandatory. It therefore asked the RTC for a liberal interpretation of the procedural rules on appeals.
2. On August 6, 2001, however, the CA dismissed the petition for certiorari.
On 29 September 1997, the RTC issued an Order 8 dismissing the appeal citing Sec. 4 of Rule 41 9 of the Revised Rules of Court.
Basis/reason: RTC did not commit grave abuse of discretion in issuing the orders, and holding that it only committed, at most, an error of judgment
correctible by appeal in issuing the challenged orders. Petitioner thereafter moved for a reconsideration of the Order 10 alleging that the trial court lost jurisdiction over the case after the former had filed the Notice of Appeal. Petitioner also alleged
that the court erred in failing to relax procedural rules for the sake of substantial justice.
3. CA remanded the case back to the RTC for further proceedings.
4. On May 9, 2003, the CA denied the motion for reconsideration stating that it decided to give due course to the appeal even if filed out of time because Javellana had no On 25 November 1997, the RTC denied the Motion.
intention to delay the proceedings, as in fact he did not even seek an extension of time to file his appellant’s brief; that current jurisprudence afforded litigants the amplest
opportunity to present their cases free from the constraints of technicalities, such that even if an appeal was filed out of time, the appellate court was given the discretion to CA:
nonetheless allow the appeal for justifiable reasons. On 28 January 1998, petitioner filed with the Court of Appeals (CA) a Petition for Certiorari and Mandamus under Rule 65 alleging that the RTC had no jurisdiction to dismiss
the Notice of Appeal, and that the trial court had acted with grave abuse of discretion when it strictly applied procedural rules.

SC: On 29 November 2000, the CA rendered its Decision 12 on the Petition. It held that while the failure of petitioner to pay the docket and other lawful fees within the reglementary
Priscilla then brought this appeal, averring that the CA thereby erred in not outrightly dismissing Javellana’s appeal because: (a) the June 21, 2000 RTC order was not appealable XXXXXXX period was a ground for the dismissal of the appeal pursuant to Sec. 1 of Rule 50 of the Revised Rules of Court, the jurisdiction to do so belonged to the CA and not the trial
court. Thus, appellate court ruled that the RTC committed grave abuse of discretion in dismissing the appeal and set aside the latter’s assailed Order dated 29 September 1997.

On his part, Javellana countered that the errors being assigned by Priscilla involved questions of fact not proper for the Court to review through petition for review on certiorari; that the June Thereafter, respondents filed their respective Motions for Reconsideration.
21, 2000 RTC order, being a final order, was appealable; that his appeal was perfected on time; It appears that prior to the promulgation of the CA’s Decision, this Court issued Administrative Matter (A.M.) No. 00-2-10-SC which took effect on 1 May 2000, amending Rule 4, Sec. 7 and
Sec. 13 of Rule 41 of the 1997 Revised Rules of Court. The circular expressly provided that trial courts may, motu proprio or upon motion, dismiss an appeal for being filed out of
time or for nonpayment of docket and other lawful fees within the reglementary period. Subsequently, Circular No. 48-2000 13 was issued on 29 August 2000 and was addressed
ISSUE to all lower courts.
WHETHER OR NOT: THE JUNE 21, 2000 RTC ORDER WAS NO APPEALABLE
By virtue of the amendment to Sec. 41, the CA upheld the questioned Orders of the trial court by issuing the assailed Amended Decision 14 in the present Petition granting respondents’
Motion for Reconsideration.
RULING
YES IT IS APPEALABLE AND THE Appeal was made on time pursuant to Neypes v. CA The CA’s action prompted petitioner to file a Motion for Reconsideration alleging that SC Circular No. 48-2000 should not be given retroactive effect. It also alleged that the CA
Section 3 of Rule 41 of the Rules of Court provides: should consider the case as exceptionally meritorious. Petitioner’s counsel, Atty. Rexes V. Alejano, explained that he was yet to familiarize himself with the Revised Rules of
Section 3. Period of ordinary appeal. — The appeal shall be taken within fifteen (15) days from notice of the judgment or final order appealed from. Where a record on appeal is required, the Court, which became effective a little over a month before he filed the Notice of Appeal. He was thus not aware that the nonpayment of docket fees might lead to the dismissal
appellant shall file a notice of appeal and a record on appeal within thirty (30) days from notice of the judgment or final order. of the case.

The period of appeal shall be interrupted by a timely motion for new trial or reconsideration. No motion for extension of time to file a motion for new trial or reconsideration shall be allowed. On 30 May 2002, the CA issued the assailed Resolution 15 denying petitioner’s Motion for Reconsideration.
(n)
Hence, this Petition. The deposit of the amount of P116,032.00 made by plaintiff with the Office of Court s hereby considered full payment of the redemption price and the Clerk of Court is hereby ordered to
deliver said amount to herein defendants.
SC:
The Register of Deeds of Davao City x x x is hereby directed to cancel TCT No. T-134470 in the name of Jose Magdangal and Estrella Magdangal and, thereafter, to reinstate TCT No.
ISSUE 72067 in the name of Jaime C. Tan and Praxedes Valles Tan and to submit her compliance thereto within ten (10) days from receipt of this Order.
W/N the appeal was dismissible for failure to pay docket fees, substantial justice demands that procedural rules be relaxed in this case.
Explaining her action, the respondent judge wrote in the same order:
'Following the ruling of the Supreme Court in Cueto vs. Collantes, et al., 97 Phil. 325, the 120 days period for plaintiff to pay the amount of P59,200.00 plus interest x x x should be reckoned
RULING from the date of Entry of Judgment x x x which was March 13, 1996. The plaintiff made a deposit on April 17, 1996 well within the 120-day period mandated by the decision of this Court.'
The Petition has no merit.
Statutes and rules regulating the procedure of courts are considered applicable to actions pending and unresolved at the time of their passage. Procedural laws and rules are SC:
retroactive in that sense and to that extent. The effect of procedural statutes and rules on the rights of a litigant may not preclude their retroactive application to pending ISSUE
actions. This retroactive application does not violate any right of a person adversely affected. Neither is it constitutionally objectionable. The reason is that, as a general rule, ISSUE: w/n the section 1, Rule 39 of the 1997 Revised Rules of Procedure should not be given retroactive effect in this case
no vested right may attach to or arise from procedural laws and rules. It has been held that "a person has no vested right in any particular remedy, and a litigant cannot insist
on the application to the trial of his case, whether civil or criminal, of any other than the existing rules of procedure.”
RULING
More so when, as in this case, petitioner admits that it was not able to pay the docket fees on time. Clearly, there were no substantive rights to speak of when the RTC dismissed the Notice NO
of Appeal.
The clerk of court of the appellate court entered in the Book of Entries of Judgement the decision in CA-G.R. CV No. 33657 and issued the corresponding Entry of Judgment which, on its
As early as 1932, in Lazaro v. Endencia, 17 we have held that the payment of the full amount of the docket fees is an indispensable step for the perfection of an appeal. The Court acquires face, stated that the said decision "has on October 21, 1995 become final and executory." 6
jurisdiction over any case only upon the payment of the prescribed docket fees. 18
The respondents Magdangal filed in the trial court a Motion for Consolidation and Writ of Possession. 7 They alleged that the 120-day period of redemption of the petitioner has expired. They
Moreover, the right to appeal is not a natural right and is not part of due process. It is merely a statutory privilege, which may be exercised only in accordance with the law. 19 reckoned that the said period began 15 days after October 5, 1995, the date when the finality of the judgment of the trial court as affirmed by the appellate court commenced to run.

We have repeatedly stated that the term "substantial justice" is not a magic wand that would automatically compel this Court to suspend procedural rules. Procedural rules are On the other hand, petitioner filed on March 27, 1996 a motion for execution in the appellate court praying that it "direct the court a quo to issue the corresponding writ of execution in Civil
not to be belittled or dismissed simply because their non-observance may result in prejudice to a party’s substantive rights. Like all other rules, they are required to be Case No. 19049-88."8 On April 17, 1996, petitioner deposited with the clerk of court the repurchase price of the lot plus interest as ordered by the decision.
followed, except only for the most persuasive of reasons when they may be relaxed to relieve litigants of an injustice not commensurate with the degree of their
thoughtlessness in not complying with the procedure prescribed. 20 On June 10, 1996, the trial court allowed the petitioner to redeem the lot in question. It ruled that the 120-day redemption period should be reckoned from the date of Entry of Judgment in
the appellate court or from March 13, 1996. 9 The redemption price was deposited on April 17, 1996. As aforestated, the Court of Appeals set aside the ruling of the trial court.
We cannot consider counsel’s failure to familiarize himself with the Revised Rules of Court as a persuasive reason to relax the application of the Rules. It is well-settled that the
negligence of counsel binds the client. This principle is based on the rule that any act performed by lawyers within the scope of their general or implied authority is regarded as From 1991-1996, the years relevant to the case at bar, the rule that governs finality of judgment is Rule 51 of the Revised Rules of Court. Its sections 10 and 11 provide:
an act of the client. Consequently, the mistake or negligence of the counsel of petitioner may result in the rendition of an unfavorable judgment against it. "SEC. 10. Entry of judgments and final resolutions. - If no appeal or motion for new trial or reconsideration is filed within the time provided in these Rules, the judgment or final resolution
shall forthwith be entered by the clerk in the book of entries of judgments. The date when the judgment or final resolution becomes executory shall be deemed as the date of its entry. The
record shall contain the dispositive part of the judgment or final resolution and shall be signed by the clerk, with a certificate that such judgment or final resolution has become final and
executory. (2a, R36)
Tan v. Court of Appeals, G.R. No. 136368, January 16, 2002
SEC. 11. Execution of judgment. - Except where the judgment or final order or resolution, or a portion thereof, is ordered to be immediately executory, the motion for its execution may only
TOPIC: Exceptions to the retroactive application of procedural rules be filed in the proper court after its entry.
FACTS: In original actions in the Court of Appeals, its writ of execution shall be accompanied by a certified true copy of the entry of judgment or final resolution and addressed to any appropriate
This is a petition for review of the Decision of the Court of Appeals dated July 15, 1998 1 and its Resolution dated November 9, 1998 2 denying petitioner's motion for reconsideration in CA- officer for its enforcement.
G.R. SP-41738.
In appealed cases, where the motion for execution pending appeal is filed in the Court of Appeals at a time that it is in possession of the original record or the record on appeal, the
Agreement: resolution granting such motion shall be transmitted to the lower court from which the case originated, together with a certified true copy of the judgment or final order to be executed, with a
"Involved in this case is a parcel of land, designated as Lot No. 645-C, with an area of 34,829 square meters, more or less, situated in Bunawan, Davao City. The lot was once covered by directive for such court of origin to issue the proper writ for its enforcement."
TCT No. T-72067 of the Registry of Deeds of Davao City in the name of the late Jaime C. Tan (Tan, for short) married to Praxedes V. Tan.
On January 22, 1981, Tan, for a consideration of P59,200.00, executed a deed of absolute sale over the property in question in favor of spouses Jose Magdangal and Estrella Magdangal. It is evident that if we apply the old rule on finality of judgment, petitioner redeemed the subject property within the 120-day period of redemption reckoned from the appellate court's entry of
Simultaneous with the execution of this deed, the same contracting parties entered into another agreement whereunder Tan given one (1) year within which to redeem or repurchase the judgment. The appellate court, however, did not apply the old rule but the 1997 Revised Rules of Civil Procedure. In fine, it applied the new rule retroactively and we hold that given the facts
property. of the case at bar this is an error.
Tan failed to redeem the property until his death on January 4, 1988.
There is no dispute that rules of procedure can be given retroactive effect. This general rule, however, has well-delineated exceptions. We quote author Agpalo
****Procedural laws are adjective laws which prescribe rules and forms of procedure of enforcing rights or obtaining redress for their invasion; they refer to rules of procedure
by which courts applying laws of all kinds can properly administer justice. They include rules of pleadings, practice and evidence. As applied to criminal law, they provide or
regulate the steps by which one who commits a crime is to be punished.
Cause of the dispute:
and The general rule that statutes are prospective and not retroactive does not ordinarily apply to procedural laws. It has been held that "a retroactive law, in a legal sense, is one
Case originally filed: which takes away or impairs vested rights acquired under laws, or creates a new obligation and imposes a new duty, or attaches a new disability, in respect of transactions or
On May 2, 1988, Tan's heirs filed before the Regional Trial Court at Davao City a suit against the Magdangals for reformation of instrument. Docketed as CIVIL CASE NO. 19049-88, the considerations already past. Hence, remedial statutes or statutes relating to remedies or modes of procedure, which do not create new or take away vested rights, but only
complaint alleged that, while Tan and the Magdangals denominated their agreement as deed of absolute sale, their real intention was to conclude an equitable mortgage. operate in furtherance of the remedy or confirmation of rights already existing, do not come within the legal conception of a retroactive law, or the general rule against the
Barely hours after the complaint was stamped 'received,' the Magdangals were able to have Tan's title over the lot in question canceled and to secure in their names TCT No. T-134470. This retroactive operation of statutes." The general rule against giving statutes retroactive operation whose effect is to impair the obligations of contract or to disturb vested rights
development prompted the heirs of Tan, who were to be later substituted by Jaime V. Tan, Jr. (Tan, Jr.) as plaintiff, to file a supplemental complaint. does not prevent the application of statutes to proceedings pending at the time of their enactment where they neither create new nor take away vested rights. A new statute
which deals with procedure only is presumptively applicable to all actions - those which have accrued or are pending.
RTC:
On June 4, 1991, Branch 11 of the Regional Trial Court of Davao City rendered judgment: Statutes regulating the procedure of the courts will be construed as applicable to actions pending and undetermined at the time of their passage. Procedural laws are
1. The Deed of Absolute Sale (Exhibits B, B-1) is, in accordance with the true intention of the parties, hereby declared and reformed an equitable mortgage; retroactive in that sense and to that extent. The fact that procedural statutes may somehow affect the litigants' rights may not preclude their retroactive application to pending
2. The plaintiff is ordered to pay the defendants within 120 days after the finality of this decisionP59,200 plus interest at the rate of 12% per annum from May 2, 1988, the date the actions. The retroactive application of procedural laws is not violative of any right of a person who may feel that he is adversely affected. Nor is the retroactive application of
complaint was filed, until paid; procedural statutes constitutionally objectionable. The reason is that as a general rule no vested right may attach to, nor arise from, procedural laws. It has been held that "a
3. In order to avoid multiplicity of suits and to fully give effect to the true intention of the parties, upon the payment of the aforesaid amount, TCT No. T-134470 in the name of defendants person has no vested right in any particular remedy, and a litigant cannot insist on the application to the trial of his case, whether civil or criminal, of any other than the
Jose Magdangal and Estrella Magdangal (Exh. 13) and shall be deemed canceled and null and void and TCT No. T-72067 in the name of Jaime C. Tan and Praxedes Valles Tan (Exh. A) be existing rules of procedure."
reinstated).
From the above, the Magdangals appealed to this Court in CA Thus, the provision of Batas Bilang 129 in Section 39 thereof prescribing that "no record on appeal shall be required to take an appeal" is procedural in nature and should
therefore be applied retroactively to pending actions. Hence, the question as to whether an appeal from an adverse judgment should be dismissed for failure of appellant to file
CA: a record on appeal within thirty days as required under the old rules, which question is pending resolution at the time Batas Bilang 129 took effect, became academic upon the
Affirmed in toto the appealed decision of the lower court. Copy of this affirmatory judgment was each received by the Magdangals and Tan, Jr. on October 5, 1995. effectivity of said law because the law no longer requires the filing of a record on appeal and its retroactive application removed the legal obstacle to giving due course to the
On March 13, 1996, the Clerk of this Court entered in the Book of Entries of Judgment the Decision and issued the corresponding Entry of Judgment which, on its face, stated that the said appeal. A statute which transfers the jurisdiction to try certain cases from a court to a quasi-judicial tribunal is a remedial statute that is applicable to claims that accrued
Decision 'has become final and executory' before its enactment but formulated and filed after it took effect, for it does not create new nor take away vested rights. The court that has jurisdiction over a claim at the time it
accrued cannot validly try the claim where at the time the claim is formulated and filed the jurisdiction to try it has been transferred by law to a quasi-judicial tribunal, for even
On March 21, 1996, the Magdangals filed in the lower court a MOTION FOR CONSOLIDATION AND WRIT OF POSSESSION, therein alleging that they did not appeal from the aforesaid actions pending in one court may be validly taken away and transferred to another and no litigant can acquire a vested right to be heard by one particular court.
decision of this Court, adding '[T]hat the appealed judgment of the Court of Appeals has become final and executory 15 days from October 5, 1995 or up to October 20, 1995, which the 120
days redemption period commences. And noting that the redemption period has expired without Tan, Jr. exercising his option, the Magdangals thus prayed that the title 'in the name of Jaime ****Exceptions to the rule:****
C. Tan and Praxedes Tan be consolidated and confirmed in the name of the (Magdangals) x x x and pending such issuance, a writ of possession be ordered issued The rule that procedural laws are applicable to pending actions or proceedings admits certain exceptions. The rule does not apply where the statute itself expressly or by
necessary implication provides that pending actions are excepted from its operation, or where to apply it to pending proceedings would impair vested rights. Under
In opposition to this motion, Tan, Jr. alleged, among other things, that until an entry of judgment has been issued by the Court of Appeals and copy thereof furnished the parties, the appropriate circumstances, courts may deny the retroactive application of procedural laws in the event that to do so would not be feasible or would work injustice. Nor may
appealed decision of the court a quo in this case cannot be considered final and executory. Pressing the point, Tan, Jr., citing Cueto vs. Collantes, infra., would then assert that the period of procedural laws be applied retroactively to pending actions if to do so would involve intricate problems of due process or impair the independence of the courts."
redemption on his part commenced to run from receipt of entry of judgment in CA-G.R. CV No. 33657.
We hold that section 1, Rule 39 of the 1997 Revised Rules of Procedure should not be given retroactive effect in this case as it would result in great injustice to the petitioner.
Meanwhile, Tan, Jr. via a motion for execution dated March 27, 1996, which he filed directly with this court, prayed this court to direct the court a quo to issue the corresponding writ of Undoubtedly, petitioner has the right to redeem the subject lot and this right is a substantive right. Petitioner followed the procedural rule then existing as well as the decisions
execution in Civil Case No. 19049-88. In a related move, Tan, Jr. filed on April 16, 1996, a MANIFESTATION AND MOTION therein advising the court a quo of his intention to redeem the of this Court governing the reckoning date of the period of redemption when he redeemed the subject lot. Unfortunately for petitioner, the rule was changed by the 1997
property in question and of the fact that, on such date, he has deposited with its clerk of court the repurchase price, plus interest, as required by its original decision. By way of relief, Tan, Jr. Revised Rules of Procedure which if applied retroactively would result in his losing the right to redeem the subject lot. It is difficult to reconcile the retroactive application of
prayed that the Magdangals be ordered to claim the amount thus deposited and the Register of Deeds of Davao City, to reinstate the title of Jaime Tan and Praxedes Tan. this procedural rule with the rule of fairness. Petitioner cannot be penalized with the loss of the subject lot when he faithfully followed the laws and the rule on the period of
redemption when he made the redemption.
Jointly acting on the aforementioned MOTON FOR CONSOLIDATION AND WRIT OF POSSESION of the Magdangals, MANIFESTATION AND MOTION of Tan, Jr. he court a
quo presided by the respondent judge, came out with the first challenged order of June 10, 1996:
The Motion for Consolidation and a Writ of Possession is hereby DENIED for lack of merit.
Petitioner fought to recover this lot from 1988. To lose it because of a change of procedure on the date of reckoning of the period of redemption is inequitous. The manner of Agreement:
exercising the right cannot be changed and the change applied retroactively if to do so will defeat the right of redemption of the petitioner which is already vested
Cause of the dispute:
On March 15, 2004, Tagyamon,5 Luna,6 Badayos,7 Dela Cruz,8 and Comandao9 received a uniformly worded Memorandum of dismissal, to wit:
This is to inform you that in view of a slump in the market demand for our products due to the un-competitiveness of our price, the company is constrained to reduce the number of its
Neypes v. Court of Appeals, G.R. No. 141524, September 14, 2005 workforce. The long-term effects of September 11 and the war in the Middle East have greatly affected the viability of our business and we are left with no recourse but to reorganize and
downsize our organizational structure.
TOPIC: Fresh-Period Rule
We wish to inform you that we are implementing a retrenchment program in accordance with Article 283 of the Labor Code of the Philippines, as amended, and its implementing rules and
FACTS: regulations.
Petitioners filed an action for annulment of judgment and titles of land and/or reconveyance and/or reversion with preliminary injunction before the RTC against the private respondents
In this connection, we regret to advise you that you are one of those affected by the said exercise, and your employment shall be terminated effective at the close of working hours on April
RTC: 15, 2004.
Later, in an order, the trial court dismissed petitioners’ complaint on the ground that the action had already prescribed. Petitioners allegedly received a copy of the order of dismissal on
March 3, 1998 and, on the 15th day thereafter or on March 18, 1998, filed a motion for reconsideration. As to Marcos, Ilao, and Nemis, they claimed that they were dismissed effective March 31, 2004, together with fifteen (15) other employees on the ground of lack of market/slump in
On July 1, 1998, the trial court issued another order dismissing the motion for reconsideration which petitioners received on July 22, 1998. Five days later, on July 27, 1998, petitioners filed demand.11 PCMC, however, claimed that they availed of the company’s voluntary retirement program and, in fact, voluntarily executed their respective Deeds of Release, Waiver, and
a notice of appeal and paid the appeal fees on August 3, 1998. Quitclaim.
On August 4, 1998, the court a quo denied the notice of appeal, holding that it was filed eight days late. This was received by petitioners on July 31, 1998. Petitioners filed a motion for
reconsideration but this too was denied in an order dated September 3, 1998. Case originally filed:
Claiming that they were aggrieved by PCMC’s decision to terminate their employment, respondents filed separate complaints for illegal dismissal against PCMC, Pacific Carpet
CA: Manufacturing Corporation, Mr. Patricio Lim and Mr. David Lim. These cases were later consolidated

Via a petition for certiorari and mandamus under Rule 65, petitioners assailed the dismissal of the notice of appeal before the CA. In the appellate court, petitioners claimed Plaintiff’s (respondents) averments:
that they had seasonably filed their notice of appeal. Respondents primarily relied on the Supreme Court’s decision in Philippine Carpet Employees Association (PHILCEA) v. Hon. Sto. Tomas (Philcea case), 13 as to the validity of the
company’s retrenchment program. They further explained that PCMC did not, in fact, suffer losses shown by its acts prior to and subsequent to their termination. 14
They argued that the 15-day reglementary period to appeal started to run only on July 22, 1998 since this was the day they received the final order of the trial court denying
their motion for reconsideration. When they filed their notice of appeal on July 27, 1998, only five days had elapsed and they were well within the reglementary period for Plaintiff’s prayer:
appeal. They also insisted that their acceptance of separation pay and signing of quitclaim is not a bar to the pursuit of illegal dismissal case.

On September 16, 1999, the CA dismissed the petition. It ruled that the 15-day period to appeal should have been reckoned from March 3, 1998 or the day they received the Defendant’s (petitioner) answer:
February 12, 1998 order dismissing their complaint. According to the appellate court, the order was the “final order” appealable under the Rules. PCMC, for its part, defended its decision to terminate the services of respondents being a necessary management prerogative. It pointed out that as an employer, it had no obligation to keep
in its employ more workers than are necessary for the operation of his business. Thus, there was an authorized cause for dismissal. Petitioners also stressed that respondents belatedly filed
their complaint as they allowed almost three years to pass making the principle of laches applicable. Considering that respondents accepted their separation pay and voluntarily executed
SC: deeds of release, waiver and quitclaim, PCMC invoked the principle of estoppel on the part of respondents to question their separation from the service. Finally, as to Marcos, Ilao and
ISSUE Nemis, PCMC emphasized that they were not dismissed from employment, but in fact they voluntarily retired from employment to take advantage of the company’s program
(1) Whether or not receipt of a final order triggers the start of the 15-day reglmentary period to appeal, the February 12, 1998 order dismissing the complaint or the July 1, 1998 order
dismissing the Motion for Reconsideration. Labor Arbiter:
(2) Whether or not petitioners file their notice of appeal on time.
On August 23, 2007, Labor Arbiter (LA) Donato G. Quinto, Jr. rendered a Decision dismissing the complaint for lack of merit. 17 The LA found no flaw in respondents’ termination as they
voluntarily opted to retire and were subsequently re-employed on a contractual basis then regularized, terminated from employment and were paid separation benefits. 18 In view of
RULING respondents’ belated filing of the complaint, the LA concluded that such action is a mere afterthought designed primarily for respondents to collect more money, taking advantage of the 2006
(1) The July 1, 1998 order dismissing the motion for reconsideration should be deemed as the final order. Supreme Court decision
In the case of Quelnan v. VHF Philippines, Inc., the trial court declared petitioner non-suited and accordingly dismissed his complaint. Upon receipt of the order of dismissal, he filed an
omnibus motion to set it aside. When the omnibus motion was filed, 12 days of the 15-day period to appeal the order had lapsed. He later on received another order, this time dismissing his NCLR:
omnibus motion. He then filed his notice of appeal. But this was likewise dismissed ― for having been filed out of time. The court a quo ruled that petitioner should have appealed within 15 On appeal, the National Labor Relations Commission (NLRC) sustained the LA decision. 20 In addition to the LA ratiocination, the NLRC emphasized the application of the principle of laches
days after the dismissal of his complaint since this was the final order that was appealable under the Rules. The SC reversed the trial court and declared that it was the denial of the motion for respondents’ inaction for an unreasonable period.
for reconsideration of an order of dismissal of a complaint which constituted the final order as it was what ended the issues raised there.
Respondents elevated the matter to the CA in a petition for certiorari.
This pronouncement was reiterated in the more recent case of Apuyan v. Haldeman et al. where the SC again considered the order denying petitioner’s motion for reconsideration as the CA:
final order which finally disposed of the issues involved in the case. Based on the aforementioned cases, the SC sustained petitioners’ view that the order dated July 1, 1998 denying their In reversing the earlier decisions of the LA and the NLRC, the CA refused to apply the principle of laches, because the case was instituted prior to the expiration of the prescriptive period set
motion for reconsideration was the final order contemplated in the Rules. by law which is four years. It stressed that said principle cannot be invoked earlier than the expiration of the prescriptive period. 21 Citing the Court’s decision in the Philcea case, the CA
applied the doctrine of stare decisis, in view of the similar factual circumstances of the cases. As to Ilao, Nemis and Marcos, while acknowledging their voluntary resignation, the CA found
(2) YES. the same not a bar to the illegal dismissal case because they did so on the mistaken belief that PCMC was losing money. 22 With the foregoing findings, the CA ordered that respondents be
To standardize the appeal periods provided in the Rules and to afford litigants fair opportunity to appeal their cases, the Court deems it practical to allow a fresh period of 15 days within reinstated with full backwages less the amounts they received as separation pay. In case of impossibility of reinstatement, the CA ordered PCMC to pay respondents backwages and in lieu
which to file the notice of appeal in the Regional Trial Court, counted from receipt of the order dismissing a motion for a new trial or motion for reconsideration. of reinstatement, separation pay equal to one month pay or ½ month pay for every year of service whichever is higher, plus moral damages

Henceforth, this "fresh period rule" shall also apply to Rule 40 governing appeals from the Municipal Trial Courts to the Regional Trial Courts; Rule 42 on petitions for review from the SC:
Regional Trial Courts to the Court of Appeals; Rule 43 on appeals from quasi-judicial agencies to the Court of Appeals and Rule 45 governing appeals by certiorari to the Supreme Court. The Priscilla then brought this appeal, averring that the CA thereby erred in not outrightly dismissing Javellana’s appeal because: (a) the June 21, 2000 RTC order was not appealable XXXXXXX
new rule aims to regiment or make the appeal period uniform, to be counted from receipt of the order denying the motion for new trial, motion for reconsideration (whether full or partial) or
any final order or resolution.
On his part, Javellana countered that the errors being assigned by Priscilla involved questions of fact not proper for the Court to review through petition for review on certiorari; that the June
The SC thus held that petitioners seasonably filed their notice of appeal within the fresh period of 15 days, counted from July 22, 1998 (the date of receipt of notice denying their motion for 21, 2000 RTC order, being a final order, was appealable; that his appeal was perfected on time;
reconsideration). This pronouncement is not inconsistent with Rule 41, Section 3 of the Rules which states that the appeal shall be taken within 15 days from notice of judgment
or final order appealed from. The use of the disjunctive word “or” signifies disassociation and independence of one thing from another. It should, as a rule, be construed in the SC:
sense in which it ordinarily implies. Hence, the use of “or” in the above provision supposes that the notice of appeal may be filed within 15 days from the notice of judgment or IN RENDERING ITS DISPUTED DECISION AND RESOLUTION, THE COURT A QUO HAS DECIDED A QUESTION OF SUBSTANCE NOT IN ACCORD WITH LAW AND/OR
within 15 days from notice of the “final order,” which we already determined to refer to the July 1, 1998 order denying the motion for a new trial or reconsideration. ESTABLISHED JURISPRUDENCE.
Neither does this new rule run counter to the spirit of Section 39 of BP 129 which shortened the appeal period from 30 days to 15 days to hasten the disposition of cases . The
original period of appeal (in this case March 3-18, 1998) remains and the requirement for strict compliance still applies. The fresh period of 15 days becomes significant only when a party Petitioners contend that the Philcea case decided by this Court and relied upon by the CA in the assailed decision was based on erroneous factual findings, inapplicable financial statement,
opts to file a motion for new trial or motion for reconsideration. In this manner, the trial court which rendered the assailed decision is given another opportunity to review the case and, in the as well as erroneous analysis of such financial statements. 25 They, thus, implore the Court to revisit the cited case in order to dispense with substantial justice. 26 They explain that the Court
process, minimize and/or rectify any error of judgment. While we aim to resolve cases with dispatch and to have judgments of courts become final at some definite time, we likewise aspire to made conclusions based on erroneous information. Petitioners also insist that the doctrines of res judicata and law of the case are not applicable, considering that this case does not involve
deliver justice fairly. the same parties as the Philcea case.27 They likewise point out that not all respondents were involuntarily separated on the ground of redundancy as some of them voluntarily availed of the
company’s Voluntary Separation Program. 28They further contend that respondents are guilty not only of laches but also of estoppel in view of their inaction for an unreasonable length of time
In this case, the new period of 15 days eradicates the confusion as to when the 15-day appeal period should be counted – from receipt of notice of judgment (March 3, 1998) or from receipt to assail the alleged illegal dismissal and in voluntarily executing a release, quitclaim and waiver
of notice of "final order" appealed from (July 22, 1998).

To recapitulate, a party litigant may either file his notice of appeal within 15 days from receipt of the RTC’s decision or file it within 15 days from receipt of the order (the “final ISSUE
order”) denying his motion for new trial or motion for reconsideration. Obviously, the new 15-day period may be availed of only if either motion is filed; otherwise, the decision The main issue sought to be determined in this case is the validity of respondents’ dismissal from employment.
becomes final and executory after the lapse of the original appeal period provided in Rule 41, Section 3.
Petitioners contend that they either voluntarily retired from the service or terminated from employment based on an authorized cause. The LA and the NLRC are one in saying that the
Petitioners here filed their notice of appeal on July 27, 1998 or five days from receipt of the order denying their motion for reconsideration on July 22, 1998. Hence, the notice of appeal was dismissal was legal. The CA, however, no longer discussed the validity of the ground of termination. Rather, it applied the Court’s decision in the Philcea case where the same ground was
well within the fresh appeal period of 15 days, as already discussed. thoroughly discussed. In other words, the appellate court applied the doctrine of stare decisis and reached the same conclusion as the earlier case.

Note: The “FRESH PERIOD RULE” do not apply to Rule 64 (Review of Judgments and Final Orders or Resolutions of the Commission on Elections and the Commission on Audit) because Under the doctrine of stare decisis, when a court has laid down a principle of law as applicable to a certain state of facts, it will adhere to that principle and apply it to all future cases in which
Rule 64 is derived from the Constitution the facts are substantially the same, even though the parties may be different. 36 Where the facts are essentially different, however, stare decisis does not apply, for a perfectly sound principle
as applied to one set of facts might be entirely inappropriate when a factual variant is introduced. 37

Phil. Carpet Manufacturing v. Tagyamon, G.R. No. 191475, December 11, 2013 The question, therefore, is whether the factual circumstances of this present case are substantially the same as the Philcea case.

TOPIC: Philippine Courts; Nature


RULING
FACTS: We answer in the affirmative.
Petitioner Philippine Carpet Manufacturing Corporation (PCMC) is a corporation registered in the Philippines engaged in the business of manufacturing wool and yarn carpets and
rugs.4 Respondents were its regular and permanent employees, but were affected by petitioner’s retrenchment and voluntary retirement programs.
This case and the Philcea case involve the same period which is March to April 2004; the issuance of Memorandum to employees informing them of the implementation of the cost reduction Petitioners are former employees of NEA who were terminated from their employment with the implementation of the assailed resolutions.
program; the implementation of the voluntary retirement program and retrenchment program, except that this case involves different employees; the execution of deeds of release, waiver,
and quitclaim, and the acceptance of separation pay by the affected employees. Respondent NEA is a government-owned and/or controlled corporation created in accordance with Presidential Decree No. (PD) 269 issued on August 6, 1973. Under PD 269, Section
5(a)(5), the NEA Board is empowered to organize or reorganize NEAs staffing structure.
The illegality of the basis of the implementation of both voluntary retirement and retrenchment programs of petitioners had been thoroughly ruled upon by the Court in the Philcea case. It
discussed the requisites of both retrenchment and redundancy as authorized causes of termination and that petitioners failed to substantiate them. In ascertaining the bases of the
termination of employees, it took into consideration petitioners’ claim of business losses Thereafter, in order to enhance and accelerate the electrification of the whole country, including the privatization of the National Power Corporation, Republic Act No. (RA) 9136,
otherwise known as the Electric Power Industry Reform Act of 2001 (EPIRA Law), was enacted, taking effect on June 26, 2001. The law imposed upon NEA additional mandates in relation
In arriving at its conclusions, the Court took note of petitioners’ net sales, gross and net profits, as well as net income. The Court, thus, reached the conclusion that the retrenchment effected to the promotion of the role of rural electric cooperatives to achieve national electrification.
by PCMC is invalid due to a substantive defect.
Meanwhile, on August 28, 2002, former President Gloria Macapagal- Arroyo issued Executive Order No. 119 directing the NEA Board to submit a reorganization plan. Thus,
The retrenchment effected by respondent Corporation is invalid due to a substantive defect, non-compliance with the substantial requirements to effect a valid retrenchment; it necessarily the NEA Board issued the assailed resolutions.
follows that the termination of the employment of petitioner Union's members on such ground is, likewise, illegal. As such, they (petitioner Union's members) are entitled to reinstatement with
full backwages. On September 17, 2003, the Department of Budget and Management approved the NEA Termination Pay Plan.

We find no reason to depart from the above conclusions which are based on the Court’s examination of the evidence presented by the parties therein. As the respondents here were similarly Thereafter, the NEA implemented an early retirement program denominated as the Early Leavers Program, giving incentives to those who availed of it and left NEA before
situated as the union members in the Philcea case, and considering that the questioned dismissal from the service was based on the same grounds under the same circumstances, there is the effectivity of the reorganization plan. The other employees of NEA were terminated effective December 31, 2003.
no need to relitigate the issues presented herein. In short, we adopt the Court’s earlier findings that there was no valid ground to terminate the employees.
ISSUE : Whether or Not the Court has jurisdiction over the instance case.
A closer look at petitioners’ arguments would show that they want the Court to re-examine our decision in the Philcea case allegedly on the ground that the conclusions therein were based
on erroneous interpretation of the evidence presented. RULING : YES. THE COURT HAS JURISDICTION OVER THE CASE.

Indeed, in Abaria v. National Labor Relations Commission, 39 although the Court was confronted with the same issue of the legality of a strike that has already been determined in a previous Evidently, the instant petition should have been filed with the RTC. However, as an exception to this general rule, the principle of hierarchy of courts may be set aside for special and
case, the Court refused to apply the doctrine of stare decisis insofar as the award of backwages was concerned because of the clear erroneous application of the law. We held therein that important reasons. Such reason exists in the instant case involving as it does the employment of the entire plantilla of NEA, more than 700 employees all told, who were effectively
the Court abandons or overrules precedents whenever it realizes that it erred in the prior decision. 40 The Court’s pronouncement in that case is instructive: dismissed from employment in one swift stroke. This to the mind of the Court entails its attention.
The doctrine though is not cast in stone for upon a showing that circumstances attendant in a particular case override the great benefits derived by our judicial system from
the doctrine of stare decisis, the Court is justified in setting it aside. For the Court, as the highest court of the land, may be guided but is not controlled by precedent. Thus, the Moreover, the Court has made a similar ruling in National Power Corporation Drivers and Mechanics Association (NPC-DAMA) v. National Power Corporation (NPC) .In that
Court, especially with a new membership, is not obliged to follow blindly a particular decision that it determines, after re-examination, to call for a rectification . case, the NPC-DAMA also filed a petition for injunction directly with this Court assailing NPC Board Resolution Nos. 2002-124 and 2002-125, both dated November 18, 2002, directing the
(TOPIC: Philippine Courts; Nature) termination of all employees of the NPC on January 31, 2003. Despite such apparent disregard of the principle of hierarchy of courts, the petition was given due course. We perceive no
compelling reason to treat the instant case differently.
The Abaria case, however, is not applicable in this case.1âwphi1 There is no reason to abandon the Court’s ruling in the Philcea case.
Do we apply the aforesaid decision to all the respondents herein? Again, we answer in the affirmative.
ERNESTO DY VS. HON. PALAMOS
Just like the union members in the Philcea case, respondents Tagyamon, Luna, Badayos, Dela Cruz, and Comandao received similarly worded memorandum of dismissal effective April 15,
CITATION : G.R. No. 187107, JANUARY 31. 2012
2004 based on the same ground of slump in the market demand for the company’s products. As such, they are similarly situated in all aspects as the union members. With respect to
respondents Marcos, Nemis and Ilao, although they applied for voluntary retirement, the same was not accepted by petitioner. Instead, it issued notice of termination dated March 6, 2004 to
FACTS : This petition for Certiorari under Rule 65 of the 1997 Revised Rules of Civil Procedure questions the December 13, 2010
these same employees.42 And while it is true that petitioner paid them separation pay, the payment was in the nature of separation and not retirement pay. In other words, payment was
and March 7, 2011Ordersof the Regional Trial Court of Makati, Branch 64 (RTC), in Civil Case No. 92-2311, granting the motion for execution of petitioner, but denying his prayer for the
made because of the implementation of the retrenchment program and not because of retirement. 43 As their application for availing of the company’s voluntary retirement program was
return of his cargo vessel in the condition when the possession thereof was seized from him.
based on the wrong premise, the intent to retire was not clearly established, or rather that the retirement is involuntary. Thus, they shall be considered discharged from
employment.44 Consequently, they shall be treated as if they are in the same footing as the other respondents herein and the union members in the Philcea case.
Petitioner Ernesto Dy (petitioner) and his wife, Lourdes Dy (Lourdes), were the proprietors of Limchia Enterprises which was engaged in the shipping business. In 1990, Limchia
Enterprises, with Lourdes as co-maker, obtained a loan from Orix Metro Leasing and Finance Corporation (respondent) to fund its acquisition of M/V Pilar-I, a cargo vessel. As additional
RAYOS VS CITY OF MANILA security for the loan, Limchia Enterprises executed the Deed of Chattel Mortgage over M/V Pilar-I.

CITATION : G.R. No. 196063, December 14, 2011


Due to financial losses suffered when M/V Pilar-I was attacked by pirates, Spouses Dy failed to make the scheduled payments as required in their promissory note. After receiving
FACTS : This petition, captioned as a petition for review on Certiorari and declaratory relief, assails the Order of 6 January 2011of the several demand letters from respondent, Spouses Dy applied for the restructuring of their loan. Meanwhile, Lourdes issued several checks to cover the remainder of their loan but the same
Regional Trial Court of Manila, Branch 49, denying reconsideration of the trial courts Order of 11 March 2010which denied the motion to dismiss filed by petitioners Orlando A. Rayos, Fe were dishonored by the bank, prompting respondent to institute a criminal complaint for violation of the Bouncing Checks Law. Lourdes appealed to respondent with a new proposal to
A. Rayos Dela Paz, and Engr. Manuel A. Rayos.The present case originated from a complaint for eminent domain filed by respondent City of Manila against Remedios V. De Caronongan, update their outstanding loan obligations.
Patria R. Serrano, Laureano M. Reyes, Paz B. Sison, Teofila B. Sison, Leticia R. Ventanilla, Rosalinda R. Barrozo (defendants), docketed as Civil Case No. 03108154.

Consequently, on August 17, 2010, petitioner filed a motion for execution of judgment with the RTC. In the intervening period, Colorado filed its Manifestation/Motion, dated July 29,
2010, informing the RTC that M/V Pilar-I, which was in its possession, had sustained severe damage and deterioration and had sunk in its shipyard because of its exposure to the elements.
In its Complaint,the City of Manila alleged that it passed Ordinance No. 7949 authorizing the City Mayor to acquire by expropriation, negotiation or by any other legal means the
For this reason, it sought permission from the court to cut the sunken vessel into pieces, sell its parts and deposit the proceeds in escrow.In his Comment/Objection, petitioner insisted that
parcel of land co-owned by defendants, which is covered by TCT No. 227512 and with an area of 1,182.20 square meters. The City of Manila offered to purchase the property
he had the right to require that the vessel be returned to him in the same condition that it had been at the time it was wrongfully seized by respondent or, should it no longer be possible, that
at P1,000.00 per square meter.
another vessel of the same tonnage, length and beam similar to that of M/V Pilar-I be delivered. Colorado, however, responded that the vessel had suffered severe damage and deterioration
In their Answer,defendants conveyed their willingness to sell the property to the City of Manila, but at the price of P50,000.00 per square meter which they claimed was the fair market
that refloating or restoring it to its former condition would be futile, impossible and very costly; and should petitioner persist in his demand that the ship be refloated, it should be done at the
value of the land at the time.
expense of the party adjudged by the court to pay the same.
On 11 March 2010, the trial court denied the motion to dismiss. The trial court ruled that the motion to dismiss did not show any compelling reason to convince the court that the doctrine
of stare decisis applies. Petitioners failed to demonstrate how or why the facts in this case are similar with the cited cases in order that the issue in this case be resolved in the same manner.
ISSUE : Whether petitioner was justified in resorting directly to this Court via a petition for certiorari under Rule 65.
ISSUE : Is Motion to dismiss appealable?

RULING : An order denying a motion to dismiss is interlocutory and not appealable. An order denying a motion to dismiss does not finally dispose of the case, and in RULING : Hierarchy of Courts; Direct ResortTo The Supreme Court Justified.
effect, allows the case to proceed until the final adjudication thereof by the court. As such, it is merely interlocutory in nature and thus, not appealable. Petitioner argues that his situation calls for the direct invocation of this Court’s jurisdiction in the interest of justice. Moreover, as pointed out by the RTC, what is
Clearly, no appeal, under Rule 45 of the Rules of Court, may be taken from an interlocutory order. In case of denial of an interlocutory order, the immediate remedy available to involved is a judgment of the Court which the lower courts cannot modify. Hence, petitioner deemed it proper to bring this case immediately to the attention of this Court.
the aggrieved party is to file a special civil action for certiorari under Rule 65 of the Rules of Court. Lastly, petitioner claims that the present case involves a novel issue of law – that is, whether in an action to recover, a defendant in wrongful possession of the subject
In this case, since the trial courts order denying the motion to dismiss is not appealable, petitioners should have filed a petition for certiorari under Rule 65 to matter in litigation may be allowed to return the same in a deteriorated condition without any liability.
assail such order, and not a petition for review on certiorari under Rule 45 of the Rules of Court. For being a wrong remedy, the present petition deserves outright dismissal.
Even if the Court treats the present petition as a petition for certiorari under Rule 65, which is the proper remedy to challenge the order denying the motion to dismiss, the RATIO : Under the principle of hierarchy of courts, direct recourse to this Court is improper because the Supreme Court is a court of last resort and must remain to be
same must be dismissed for violation of the principle of hierarchy of courts. This well-settled principle dictates that petitioners should file the petition for certiorari with the Court of Appeals, so in order for it to satisfactorily perform its constitutional functions, thereby allowing it to devote its time and attention to matters within its exclusive jurisdiction and preventing the
and not directly with this Court. overcrowding of its docket. 16 Nonetheless, the invocation of this Court’s original jurisdiction to issue writs of certiorari has been allowed in certain instances on the ground of special and
important reasons clearly stated in the petition, such as,(1) when dictated by the public welfare and the advancement of public policy; (2) when demanded by the broader interest of justice;
RATIONALE : This Courts original jurisdiction to issue writs of certiorari is not exclusive. It is shared by this Court with Regional Trial Courts and with the Court of (3) when the challenged orders were patent nullities; or (4) when analogous exceptional and compelling circumstances called for and justified the immediate and direct handling of the
Appeals. This concurrence of jurisdiction is not,however, to be taken as according to parties seeking any of the writs an absolute, unrestrained freedom of choice of the court to which case.17
application therefor will be directed. There is after all a hierarchy of courts. That hierarchy is determinative of the venue of appeals, and also serves as a general determinant of the This case falls under one of the exceptions to the principle of hierarchy of courts. Justice demands that this Court take cognizance of this case to put an end to the controversy and resolve
appropriate forum for petitions for the extraordinary writs. A becoming regard for that judicial hierarchy most certainly indicates that petitions for the issuance of extraordinary writs against the matter which has been dragging on for more than twenty (20) years. Moreover, in light of the fact that what is involved is a final judgment promulgated by this Court, it is but proper for
first level (inferior) courts should be filed with the Regional Trial Court, and those against the latter, with the Court of Appeals. A direct invocation of the Supreme Courts original petitioner to call upon its original jurisdiction and seek final clarification.
jurisdiction to issue these writs should be allowed only when there are special and important reasons therefor, clearly and specifically set out in the petition . This is [an]
established policy. It is a policy necessary to prevent inordinate demands upon the Courts time and attention which are better devoted to those matters within its exclusive jurisdiction, and to
HOME GUARANTY CORP. VS. R-11 BUILDERS
prevent further over-crowding of the Courts docket.
CITATION : G.R. NO. 192649, MARCH 9, 2011

UNITED CLAIMANTS ASSOC. VS. NEA


FACTS : In urging the reversal of the Court's decision, R-II Builders argues that it filed its complaint with the Manila RTC which is
undoubtedly vested with jurisdiction over actions where the subject matter is incapable of pecuniary estimation; that through no fault of its own, said complaint was raffled to Branch 24, the
CITATION : G.R. No. 187107, JANUARY 31. 2012 designated Special Commercial Court (SCC) tasked to hear intra-corporate controversies; that despite the determination subsequently made by Branch 24 of the Manila RTC that the case
did not involve an intra-corporate dispute, the Manila RTC did not lose jurisdiction over the same and its Executive Judge correctly directed its re-raffling to Branch 22 of the same Court; that
the re-raffle and/or amendment of pleadings do not affect a court's jurisdiction which, once acquired, continues until the case is finally terminated; that since its original Complaint, Amended
and Supplemental Complaint and Second Amended Complaint all primarily sought the nullification of the Deed of Assignment and Conveyance (DAC) transferring the Asset Pool in favor of
FACTS : This is an original action for Injunction to restrain and/or prevent the implementation of Resolution Nos. 46 and 59, dated petitioner Home Guaranty Corporation (HGC), the subject matter of the case is clearly one which is incapable of pecuniary estimation; and, that the court erred in holding that the case was a
July 10, 2003 and September 3, 2003, respectively, otherwise known as the National Electrification Administration (NEA) Termination Pay Plan, issued by respondent NEA Board of real action and that it evaded the payment of the correct docket fees computed on the basis of the assessed value of the realties in the Asset Pool.
Administrators (NEA Board).
The record shows that, with the raffle of R-II Builders’ complaint before Branch 24 of the Manila RTC and said court’s grant of the It is settled that jurisdiction is conferred by law based on the facts alleged in the complaint since the latter comprises a concise statement of the ultimate facts constituting the plaintiff's
application for temporary restraining order incorporated therein, HGC sought a preliminary hearing of its affirmative defenses which included, among other grounds, lack of jurisdiction and causes of action.[11] In the case at bar, after examining the original complaint, we find that the RTC acquired jurisdiction over the case when the case was filed before it. From the allegations
improper venue. It appears that, at said preliminary hearing, it was established that R-II Builders’ complaint did not involve an intra-corporate dispute and that, even if it is, venue was thereof, respondents cause of action is for damages arising from libel, the jurisdiction of which is vested with the RTC. Article 360 of the Revised Penal Code provides that it is a Court of
improperly laid since none of the parties maintained its principal office in Manila. While it is true, therefore, that R-II Builders had no hand in the raffling of the case, it cannot be gainsaid that First Instance.that is specifically designated to try a libel case
Branch 24 of the RTC Manila had no jurisdiction over the case. Rather than ordering the dismissal of the complaint, however, said court issued the 2 January 2008 order erroneously
ordering the re-raffle of the case. In Atwel v. Concepcion Progressive Association, Inc. and Reyes v. Hon. Regional Trial Court of Makati, Branch 142 which involved SCCs trying and/or Petitioners are confusing jurisdiction with venue. A former colleague, the Hon. Florenz D. Regalado, differentiated jurisdiction and venue as follows: (a) Jurisdiction is the authority to
deciding cases which were found to be civil in nature, this Court significantly ordered the dismissal of the complaint for lack of jurisdiction instead of simply directing the re-raffle of the case hear and determine a case; venue is the place where the case is to be heard or tried; (b) Jurisdiction is a matter of substantive law; venue, of procedural law; (c) Jurisdiction establishes a
to another branch. relation between the court and the subject matter; venue, a relation between plaintiff and defendant, or petitioner and respondent; and, (d) Jurisdiction is fixed by law and cannot be
conferred by the parties; venue may be conferred by the act or agreement of the parties.
ISSUE : Whether or not the Court had no jurisdiction over the instant case.
RULING : Even then, the question of the Manila RTC's jurisdiction over the case is tied up with R-II Builder's payment of the correct docket fees which should be paid In the case at bar, the additional allegations in the Amended Complaint that the article and the caricature were printed and first published in the City of Makati referred only to the
in full upon the filing of the pleading or other application which initiates an action or proceeding. While it is, consequently, true that jurisdiction, once acquired, cannot be easily ousted, it is question of venue and not jurisdiction. These additional allegations would neither confer jurisdiction on the RTC nor would respondents failure to include the same in the original complaint
equally settled that a court acquires jurisdiction over a case only upon the payment of the prescribed filing and docket fees. Already implicit from the filing of the complaint in the City of divest the lower court of its jurisdiction over the case. Respondents failure to allege these allegations gave the lower court the power, upon motion by a party, to dismiss the complaint on the
Manila where the realties comprising the Asset Pool are located, the fact that the case is a real action is evident from the allegations of R-II Builders’ original Complaint, Amended and ground that venue was not properly laid.
Supplemental Complaint and Second Amended Complaint which not only sought the nullification of the DAC in favor of HGC but, more importantly, prayed for the transfer of possession of
and/or control of the properties in the Asset Pool. Its current protestations to the contrary notwithstanding, no less than R-II Builders – in its opposition to HGC’s motion to dismiss – admitted
that the case is a real action as it affects title to or possession of real property or an interest therein. Having only paid docket fees corresponding to an action where the subject matter is
incapable of pecuniary estimation, R-II Builders cannot expediently claim that jurisdiction over the case had already attached.
In De Leon v. Court of Appeals, this Court had, of course, ruled that a case for rescission or annulment of contract is not susceptible of pecuniary estimation although it may ARMAND NOCUM and THE PHILIPPINE DAILY INQUIRER, INC. vs. LUCIO TAN
eventually result in the recovery of real property. Taking into consideration the allegations and the nature of the relief sought in the complaint in the subsequent case of Serrano v. Delica,
however, this Court determined the existence of a real action and ordered the payment of the appropriate docket fees for a complaint for cancellation of sale which prayed for both G.R. No. 145022 September 23, 2005
permanent and preliminary injunction aimed at the restoration of possession of the land in litigation is a real action. In discounting the apparent conflict in said rulings, the Court went on to
rule as follows in Ruby Shelter Builders and Realty Development Corporation v. Hon. Pablo C, Formaran, to wit: Topic: Jurisdiction vs. Venue
The Court x x x does not perceive a contradiction between Serrano and the Spouses De Leon. The Court calls attention to the following statement in Spouses De Leon: "A
review of the jurisprudence of this Court indicates that in determining whether an action is one the subject matter of which is not capable of pecuniary estimation, this Court has adopted the Facts:
criterion of first ascertaining the nature of the principal action or remedy sought." Necessarily, the determination must be done on a case-to-case basis, depending on the facts and Lucio Tan filed a complaint against reporter Armand Nocum, Capt. Florendo Umali, ALPAP and Inquirer with the Regional Trial Court of Makati, seeking moral and exemplary damages for
circumstances of each. What petitioner conveniently ignores is that in Spouses De Leon, the action therein that private respondents instituted before the RTC was "solely for annulment or the alleged malicious and defamatory imputations contained in a news article. INQUIRER and NOCUM filed their joint answer, dated October 27, 1998. ALPAP and UMALI likewise filed their
rescission" of the contract of sale over a real property. joint answer and alleged therein that the venue was improperly laid, among others. It appeared that the complaint failed to state the residence of the complainant at the time of the alleged
commission of the offense and the place where the libelous article was printed and first published. Thus, the Regional Trial Court of Makati dismissed the complaint without prejudice on the
ground of improper venue.

Aggrieved, respondent Lucio Tan filed an Omnibus Motion seeking reconsideration of the dismissal and admission of the amended complaint. In par. 2.01.1 of the amended complaint, it is
DAVAO LIGHT VS. COURT OF APPEALS alleged that "This article was printed and first published in the City of Makati".

RTC admitted the amended complaint and deemed set aside the previous order of dismissal stating that the mistake or deficiency in the original complaint appears now to have been cured
CITATION : G.R. NO. 111685, AUGUST 20, 2001 in the Amended Complaint. Besides, there is no substantial amendment in the Amended Complaint which would affect the defendants’ defenses and their Answers as the Amendment is
merely formal.
FACTS : The Davao Light and Power Co., Inc. ("Davao Light") filed a collection suit against Queensland Hotel ("Queensland") and
Teodorico Adarna ("Adarna") with an ex parte application for a writ of preliminary attachment. On 3 May 1989, the trial court issued an Order of Attachment, and the corresponding Writ of Dissatisfied, petitioners appealed the RTC decision to the Court of Appeals. Two petitions for certiorari were filed, one filed by petitioners and the other by defendants. The two petitions were
Attachment on 11 May 1989. On 12 May 1989, the summons, a copy of the complaint, and the writ of attachment was served upon Queensland and Adarna. Queensland and Adarna filed a consolidated. CA affirmed the decision of the RTC. The motions for reconsideration filed by petitioners and by defendants Umali and ALPAP were likewise denied in a resolution dated 15
motion to discharge the attachment on the ground that at the time the Order of Attachment and Writ of Attachment were issued, the trial court has yet to acquire jurisdiction over the cause of September 2000. Hence, this petition.
action and over the persons of the defendants.
This Order of September 19, 1989 was successfully challenged by Queensland and Adarna in a special civil Issue: Whether or not the lower court acquire jurisdiction over the civil case upon the filing of the original complaint for damages.
action of certiorari instituted by them in the Court of Appeals. The Order was, as aforestated, annulled by the Court of Appeals in its Decision of May 4, 1990.
Ruling:
ISSUE : The question is whether or not a writ of preliminary attachment may issue ex parte against a defendant before acquisition of It is settled that jurisdiction is conferred by law based on the facts alleged in the complaint since the latter comprises a concise statement of the ultimate facts constituting the plaintiff's
jurisdiction of the latter's person by service of summons or his voluntary submission to the Court's authority. causes of action. In the case at bar, after examining the original complaint, we find that the RTC acquired jurisdiction over the case when the case was filed before it. From the allegations
thereof, respondent’s cause of action is for damages arising from libel, the jurisdiction of which is vested with the RTC. Article 360 of the Revised Penal Code provides that it is a Court of
RULING : The Court rules that the question must be answered in the affirmative and that consequently, the petition for review will have First Instance that is specifically designated to try a libel case.
to be granted.
The court may validly issue a writ of preliminary injunction prior to the acquisition of jurisdiction over the person of the defendant. There is an appreciable period of time between Petitioners are confusing jurisdiction with venue. A former colleague, the Hon. Florenz D. Regalado, differentiated jurisdiction and venue as follows: (a) Jurisdiction is the authority to hear
the commencement of the action (takes place upon the filing of an initiatory pleading) and the service of summons to the defendant. In the meanwhile, there are a number of actions which and determine a case; venue is the place where the case is to be heard or tried; (b) Jurisdiction is a matter of substantive law; venue, of procedural law; (c) Jurisdiction
the plaintiff or the court may validly take, including the application for and grant of the provisional remedy of preliminary attachment. There is nothing in the law which prohibits the court from establishes a relation between the court and the subject matter; venue, a relation between plaintiff and defendant, or petitioner and respondent; and, (d) Jurisdiction is fixed by
granting the remedy prior to the acquisition of jurisdiction over the person of the defendant. In fact, Rule 57 of the Rules of Court allows the granting of a writ of preliminary injunction at the law and cannot be conferred by the parties; venue may be conferred by the act or agreement of the parties.
commencement of the suit. In the cases of Toledo v. Burgos and Filinvest Credit Corporation v. Relova, it was held that notice and hearing are not prerequisites to the issuance of a writ of
preliminary attachment. Further, in the case of Mindanao Savings & Loan Association, Inc. v. Court of Appeals , it was ruled that giving notice to the defendant would defeat the purpose of In the case at bar, the additional allegations in the Amended Complaint that the article and the caricature were printed and first published in the City of Makati referred only to the question of
the remedy by affording him or her the opportunity to dispose of his properties before the writ can be issued. venue and not jurisdiction. These additional allegations would neither confer jurisdiction on the RTC nor would respondent’s failure to include the same in the original complaint divest the
A preliminary attachment may be discharged with the same ease as obtaining it. In any case, the ease of availing the provisional remedy of preliminary attachment is matched by the lower court of its jurisdiction over the case. Respondent’s failure to allege these allegations gave the lower court the power, upon motion by a party, to dismiss the complaint on the ground
ease with which it can be remedied by either the posting of a counterbond, or by a showing of its improper or irregular issuance. The second means of defeating a preliminary attachement, that venue was not properly laid.
however, may not be availed of if the writ was issued upon a ground which is at the same time the applicant's cause of action.
Preliminary attachment not binding until jurisdiction over the person of the defendant is acquired. The writ of preliminary attachment, however, even though validly issued, is not binding In Laquian v. Baltazar, this Court construed the term "jurisdiction" in Article 360 of the Revised Penal Code as referring to the place where actions for libel shall be filed or "venue."
upon the defendant until jurisdiction over his person is first acquired.
We note that the amended complaint or amendment to the complaint was not intended to vest jurisdiction to the lower court, where originally it had none. The amendment was merely to
establish the proper venue for the action. It is a well-established rule that venue has nothing to do with jurisdiction, except in criminal actions. Assuming that venue were
properly laid in the court where the action was instituted, that would be procedural, not a jurisdictional impediment. In fact, in civil cases, venue may be waived.
NOCUM VS. LUCIO TAN
CITATION : G.R. N0. 145022, SEPTEMBER 23, 2005 Consequently, by dismissing the case on the ground of improper venue, the lower court had jurisdiction over the case. Apparently, the herein petitioners recognized this jurisdiction by filing
FACTS : On September 27, 1998, Lucio Tan filed a complaint against reporter Armand Nocum, Capt. Florendo their answers to the complaint, albeit, questioning the propriety of venue, instead of a motion to dismiss.
Umali, ALPAP and Inquirer with the Regional Trial Court of Makati, docketed as Civil Case No. 98-2288, seeking moral and exemplary damages for the alleged malicious
and defamatory imputations contained in a news article. We so hold that dismissal of the complaint by the lower court was proper considering that the complaint, indeed, on its face, failed to allege neither the residence of the complainant nor the
place where the libelous article was printed and first published. Nevertheless, before the finality of the dismissal, the same may still be amended as in fact the amended complaint was
INQUIRER and NOCUM filed their joint answer, dated October 27, 1998, wherein they alleged that: (1) the complaint failed to state a cause of action; (2) the admitted, in view of the court a quo’s jurisdiction, of which it was never divested. In so doing, the court acted properly and without any grave abuse of discretion.
defamatory statements alleged in the complaint were general conclusions without factual premises; (3) the questioned news report constituted fair and true report on the
matters of public interest concerning a public figure and therefore, was privileged in nature; and (4) malice on their part was negated by the publication in the same article of
plaintiffs or PALs side of the dispute with the pilots union. SPOUSES JESUS FAJARDO and EMER FAJARDO vs. ANITA R. FLORES, assisted by her husband, BIENVENIDO FLORES
G.R. No. 167891 January 15, 2010
ALPAP and UMALI likewise filed their joint answer, dated October 31, 1998, and alleged therein that: (1) the complaint stated no cause of action; (2) venue was
improperly laid; and (3) plaintiff Lucio Tan was not a real party in interest. It appeared that the complaint failed to state the residence of the complainant at the time of the Topic: Doctrine of primary jurisdiction
alleged commission of the offense and the place where the libelous article was printed and first published.
Facts:
Thus, the Regional Trial Court of Makati issued an Order dated February 10, 1999, dismissing the complaint without prejudice on the ground of improper venue. Leopoldo delos Reyes owned a parcel of land located in Barangay Sumandig in Hacienda Buenavista, San Ildefonso, Bulacan. In 1963, he allowed petitioner Jesus Fajardo to cultivate said
land. The net harvests were divided equally between the two until 1975 when the relationship was converted to leasehold tenancy. From the time petitioner cultivated the land, he was
Aggrieved by the dismissal of the complaint, respondent Lucio Tan filed an Omnibus Motion dated February 24, 1999, seeking reconsideration of the dismissal and allowed by Leopoldo delos Reyes to erect a house for his family on the stony part of the land, which is the subject of controversy.
admission of the amended complaint. In par. 2.01.1 of the amended complaint, it is alleged that This article was printed and first published in the City of Makati (p. 53,
Rollo, CA-G.R. SP No. 55192), and in par. 2.04.1, that This caricature was printed and first published in the City of Makati (p. 55, id.). On January 26, 1988, Leopoldo delos Reyes died. His daughter and sole heir, herein respondent Anita Flores, inherited the property. On June 28, 1991, Anita Flores and Jesus Fajardo
The lower court, after having the case dismissed for improper venue, admitted the amended complaint and deemed set aside the previous order of dismissal . executed an agreement, denominated as “KASUNDUAN NG PAGHAHATI NG LUPA AT PAGTATALAGA NG DAAN UKOL SA MAGKABILANG PANIG followed by another agreement,
“KASUNDUAN SA HATIAN SA LUPA,” executed on July 10, 1991, wherein the parties agreed to deduct from Lot No. 2351 an area of 10,923 sq m, allotting the same to petitioner.
Apparently, there was a conflict of claims in the interpretation of the Kasunduan between Anita Flores and Jesus Fajardo, which was referred to the DAR, Provincial Agrarian Reform Office,
Baliuag, Bulacan. The case was referred to the Department of Agrarian Reform Adjudication Board (DARAB), Malolos, Bulacan.
ISSUE : The question to be resolved is: Did the lower court acquire jurisdiction over the civil case upon the filing of the original complaint for
damages? On December 22, 2000, a complaint for ejectment was filed by herein respondent Anita Flores, assisted by her husband Bienvenido Flores, against petitioners with the Municipal Trial Court
(MTC), San Ildefonso, Bulacan. Petitioners filed a Motion to Dismiss, alleging that Lot No. 2351, was agricultural land; that they had been continuously, uninterruptedly, and personally
RULING : The Court rule in the affirmative. cultivating the same since 1960 up to the present; that the MTC had no jurisdiction over the case, considering that the dispute between the parties, regarding the Kasunduan, was referred to
the DARAB; and that the assumption by the DARAB of jurisdiction over the controversy involving the lot in question therefore precluded the MTC from exercising jurisdiction over the case.
Resolving the Motion to Dismiss, the MTC ruled that, while at first glance, the court did not have jurisdiction over the case, considering that it was admitted that petitioner was allowed to
cultivate the land, a closer look at the Kasunduan, however, revealed that what was divided was only the portion being tilled. By contrast, the subject matter of the complaint was the stony
portion where petitioners’ house was erected. Thus, the court ruled that it had jurisdiction over the subject matter. On April 25, 2001, the MTC rendered judgment in favor of respondent.

On appeal, the Regional Trial Court (RTC), Branch 16, Third Judicial Region, Malolos, Bulacan, affirmed the MTC Decision in toto upon a finding that no reversible error was committed by
the court a quo in its Decision dated August 29, 2002. On motion for reconsideration, however, the RTC issued an Order on December 10, 2002, reversing its decision dated August 29,
2002. The RTC found that the issue involved appeared to be an agrarian dispute, which fell within the contemplation of Republic Act (R.A.) No. 6657, otherwise known as the
Comprehensive Agrarian Reform Law of 1988, and thus ordered the dismissal of the case for lack of jurisdiction.

A petition for review was then filed by respondents with the CA to annul the Order of the RTC dated December 10, 2002. On October 28, 2004, the CA rendered the assailed decision, which
reinstated the MTC decision. It disagreed with the findings of the RTC and ruled that the part of Lot No. 2351 where petitioners’ house stood was stony and residential in nature, one that
may not be made to fall within the ambit of the operation of Philippine agrarian laws, owing to its non-agriculture character. The subsequent motion for reconsideration was denied; hence,
this petition.

Issue:
Whether or not it is MTC or the DARAB which has jurisdiction over the case.

Ruling:
We agree with the RTC when it clearly pointed out in its Order dated December 10, 2002 that the resolution of this case hinges on the correct interpretation of the contracts executed by the
parties. The issue of who has a better right of possession over the subject land cannot be determined without resolving first the matter as to whom the subject property was allotted. Thus,
this is not simply a case for unlawful detainer, but one that is incapable of pecuniary estimation, definitely beyond the competence of the MTC.

More importantly, the controversy involves an agricultural land, which petitioners have continuously and personally cultivated since the 1960s. In the Kasunduan, it was admitted that Jesus
Fajardo was the tiller of the land. Being agricultural lessees, petitioners have a right to a home lot and a right to exclusive possession thereof by virtue of Section 24, R.A. No. 3844 of the
Agricultural Land Reform Code. Logically, therefore, the case involves an agrarian dispute, which falls within the contemplation of R.A. No. 6657, or the Comprehensive Agrarian Reform
Law.

An agrarian dispute refers to any controversy relating to tenurial arrangements, whether leasehold, tenancy, stewardship, or otherwise, over lands devoted to agriculture, including disputes
concerning farmworkers’ associations or representation of persons in negotiating, fixing, maintaining, changing, or seeking to arrange terms or conditions of such tenurial arrangements. It
includes any controversy relating to compensation of lands acquired under this Act and other terms and conditions of transfer of ownership from landowner to farmworkers, tenants, and
other agrarian reform beneficiaries, whether the disputants stand in the proximate relation of farm operator and beneficiary, landowner and tenant, or lessor and lessee. It relates to any
controversy relating to, inter alia, tenancy over lands devoted to agriculture.

Undeniably, the instant case involves a controversy regarding tenurial arrangements. The contention that the Kasunduans, which allegedly terminated the tenancy relationship between the
parties and, therefore, removed the case from the ambit of R.A. No. 6657, is untenable. There still exists an agrarian dispute because the controversy involves the home lot of petitioners, an
incident arising from the landlord-tenant relationship.

In the case at bar, petitioners’ claim that the tenancy relationship has been terminated by the Kasulatan is of no moment. As long as the subject matter of the dispute is the legality of the
termination of the relationship, or if the dispute originates from such relationship, the case is cognizable by the DAR, through the DARAB. The severance of the tenurial arrangement will not
render the action beyond the ambit of an agrarian dispute.

Furthermore, the records disclose that the dispute between the parties, regarding the interpretation of the Kasunduan, was, in fact, raised and referred to the DAR, which in turn referred the
case to the DARAB. In view of the foregoing, we reiterate Hilario v. Prudente, that:
The doctrine of primary jurisdiction precludes the courts from resolving a controversy over which jurisdiction has initially been lodged with an administrative body of
special competence. For agrarian reform cases, jurisdiction is vested in the Department of Agrarian Reform (DAR); more specifically, in the Department of Agrarian
Reform Adjudication Board (DARAB).
TOPIC: ADHERENCE OF JURISDICTION
RULING:

JOSE E. ARUEGO VS. COURT OF APPEALS NO.

CITATION: G.R. NO. 112193 MARCH 13, 1996 As ruled in Tayag vs. Court of Appeals:

CASE: On March 7, 1983, a Complaint for Compulsory Recognition and Enforcement of Successional Rights was filed before Branch 30 of the Regional Trial Court of Manila by the minors, Under the circumstances obtaining in the case at bar, we hold that the right of action of the minor child has been vested by the filing of the complaint in court under the
private respondent Antonia F. Aruego and her alleged sister Evelyn F. Aruego, represented by their mother and natural guardian, Luz M. Fabian. Named defendants therein were Jose E. regime of the Civil Code and prior to the effectivity of the Family Code. We herein adopt our ruling in the recent case of Republic of the Philippines vs. Court of Appeals, et.
Aruego, Jr. and the five (5) minor children of the deceased Gloria A. Torres, represented by their father and natural guardian, Justo P. Torres, Jr. al. 7 where we held that the fact of filing of the petition already vested in the petitioner her right to file it and to have the same proceed to final adjudication in accordance with
the law in force at the time, and such right can no longer be prejudiced or impaired by the enactment of a new law.
FACTS: The late Jose M. Aruego, Sr., a married man, had an amorous relationship with Luz M. Fabian in 1959 until his death on March 30, 1982. Out of this relationship were born Antonia
F. Aruego and Evelyn F. Aruego on October 5, 1962 and September 3, 1963, respectively. xxx xxx xxx

Jose M. Aruego, Sr acknowledged and recognized the herein private respondents as his children verbally among respondent’s and their mother's family friends, as well as by myriad different Accordingly, Article 175 of the Family Code finds no proper application to the instant case since it will ineluctably affect adversely a right of private respondent and,
paternal ways, including but not limited to the following: consequentially, of the minor child she represents, both of which have been vested with the filing of the complaint in court . The trial court is, therefore, correct in applying the
provisions of Article 285 of the Civil Code and in holding that private respondent's cause of action has not yet prescribed.
1. Regular support and educational expenses;
2. Allowance to use his surname; Tayag applies four-square with the case at bench. The action brought by private respondent Antonia Aruego for compulsory recognition and enforcement of successional rights which was
3. Payment of maternal bills; filed prior to the advent of the Family Code, must be governed by Article 285 of the Civil Code and not by Article 175, paragraph 2 of the Family Code. The present law cannot be given
4. Payment of baptismal expenses and attendance therein; retroactive effect insofar as the instant case is concerned, as its application will prejudice the vested right of private respondent to have her case decided under Article 285 of
5. Taking them to restaurants and department stores on occasions of family rejoicing; the Civil Code. The right was vested to her by the fact that she filed her action under the regime of the Civil Code. Prescinding from this, the conclusion then ought to be that the
6. Attendance to school problems of plaintiffs; action was not yet barred, notwithstanding the fact that it was brought when the putative father was already deceased, since private respondent was then still a minor when it was filed, an
7. Calling and allowing plaintiffs to his office every now and then; exception to the general rule provided under Article 285 of the Civil Code. Hence, the trial court, which acquired jurisdiction over the case by the filing of the complaint, never lost
8. Introducing them as such children to family friends. jurisdiction over the same despite the passage of E.O. No. 209, also known as the Family Code of the Philippines.

The respondents are thus, in continuous possession of the status of (illegitimate) children of the deceased Jose M. Aruego who showered them, with the continuous and clear manifestations The jurisdiction of a court, whether in criminal or civil cases, once attached cannot be ousted by subsequent happenings or events, although of a character which would have prevented
of paternal care and affection as above outlined. jurisdiction from attaching in the first instance, and it retains jurisdiction until it finally disposes of the case.

Basis of Respondent’s action for compulsory recognition as an illegitimate child:

Book I, Title VIII of the Civil Code on PERSONS, specifically Article 285 thereof, which state the manner by which illegitimate children may prove their TOPIC: ADHERENCE OF JURISDICTION
filiation, to wit:

Art. 285. The action for the recognition of natural children may be brought only during the lifetime of the presumed parents, except in the following cases: LUCIA BARRAMEDA VS. RURAL BANK OF CANAMAN
1. If the father or mother died during the minority of the child, in which case the latter may file the action before the expiration of four years from the CITATION: G.R. NO. 176260 NOVEMBER 24, 2010
attainment of his majority; . . . .
CASE originally filed: On March 17, 2000, petitioner Lucia Barrameda Vda. DeBallesteros filed a complaint for Annulment of Deed of Extrajudicial Partition, Deed of Mortgage and
Damages with prayer for Preliminary Injunction against her children, Roy, Rito, Amy, Arabel, Rico, Abe, Ponce Rex and Adden, all surnamed Ballesteros, and the Rural Bank of Canaman,
Inc., Baao Branch (RBCI) before the RTC-Irig.

Petitioner’s argument: FACTS:


With the advent of the New Family Code on August 3, 1988, the trial court lost jurisdiction over the complaint of private respondent on the ground of prescription, considering that under Lucia’s deceased husband, Eugenio, left two (2) parcels of land located in San Nicolas, Baao, Camarines Sur, each with an area of 357 square meters. On March 6, 1995, without her
Article 175, paragraph 2, in relation to Article 172 of the New Family Code, it is provided that an action for compulsory recognition of illegitimate filiation, if based on the "open and continuous knowledge and consent, her children executed a deed of extrajudicial partition and waiver of the estate of her husband wherein all the heirs, including Lucia, agreed to allot the two parcels
possession of the status of an illegitimate child," must be brought during the lifetime of the alleged parent without any exception, otherwise the action will be barred by prescription. to Rico Ballesteros. Still, without her knowledge and consent, Rico mortgaged Parcel B of the estate in favor of RBCI which mortgage was being foreclosed for failure to settle the loan
secured by the lot; and that Lucia was occupying Parcel B and had no other place to live.
Basis of Petitioners’ argument:
She prayed that the deed of extrajudicial partition and waiver, and the subsequent mortgage in favor of RBCI be declared null and void having been executed without her knowledge and
Art. 172. The filiation of legitimate children is established by any of the following: consent. She also prayed for damages.
1. The record of birth appearing in the civil register or a final judgment; or RBCI claimed that in 1979, Lucia sold one of the two parcels to Rico which represented her share in the estate of her husband. The extrajudicial partition, waiver and mortgage were all
2. An admission of legitimate filiation in a public document or a private handwritten instrument and signed by the parent concerned. executed with the knowledge and consent of Lucia although she was not able to sign the document. RBCI further claimed that Parcel B had already been foreclosed way back in 1999 which
fact was known to Lucia through the auctioning notary public.
In the absence of the foregoing evidence, the legitimate filiation shall be proved by:
Attorney’s fees were pleaded as counterclaim.
1. The open and continuous possession of the status of a legitimate child; or
2. Any other means allowed by the Rules of Court and special laws. During the pre-trial, RBCIs counsel filed a motion to withdraw after being informed that Philippine Deposit Insurance Corporation (PDIC) would handle the case as RBCI had already been
closed and placed under the receivership of the PDIC. Consequently, on February 4, 2002, the lawyers of PDIC took over the case of RBCI.
Art. 175. Illegitimate children may establish their illegitimate filiation in the same way and on the same evidence as legitimate children.
On May 9, 2003, RBCI, through PDIC, filed a motion to dismiss on the ground that the RTC-Iriga has no jurisdiction over the subject matter of the action. RBCI stated that pursuant to
The action must be brought within the same period specified in Article 173 [during the lifetime of the child], except when the action is based on the second paragraph of Section 30, Republic Act No. 7653, otherwise known as the New Central Bank Act, the RTC-Makati, already constituted itself, per its Order dated August 10, 2001, as the liquidation court to
Article 172, in which case the action may be brought during the lifetime of the alleged parent. assist PDIC in undertaking the liquidation of RBCI. Thus, the subject matter of Civil Case No. IR-3128 fell within the exclusive jurisdiction of such liquidation court.

Petitioners point out that, since the complaint of private respondent and her alleged sister was filed on March 7, 1983, or almost one (1) year after the death of their presumed father on Lucia opposed the motion.
March 30, 1982, the action has clearly prescribed under the new rule as provided in the Family Code. Petitioners, further, maintain that even if the action was filed prior to the effectivity of
the Family Code, this new law must be applied to the instant case pursuant to Article 256 of the Family Code which provides: Argument: Lucia contends that the RTC-Iriga is vested with jurisdiction over Civil Case No. 3128, the constitution of the liquidation court notwithstanding. She contends that:

This Code shall, have retroactive effect insofar as it does not prejudice or impair vested of acquired rights in accordance with the Civil Code or other laws. Since the RTC-Iriga has already obtained jurisdiction over the case it should continue exercising such jurisdiction until the final termination of the case. The jurisdiction of a
court once attached cannot be ousted by subsequent happenings or events, although of a character which would have prevented jurisdiction from attaching in the first
instance, and the Court retains jurisdiction until it finally disposes of the case (Aruego Jr. v. Court of Appeals, 254 SCRA 711).
RTC
When a court has already obtained and is exercising jurisdiction over a controversy, its jurisdiction to proceed to final determination of the case is not affected by a new
Decision: The court declared Antonia Aruego as illegitimate daughter of Jose Aruego and Luz Fabian. The court further declared that Evelyn Fabian is not an illegitimate legislation transferring jurisdiction over such proceedings to another tribunal. (Alindao v. Joson, 264 SCRA 211). Once jurisdiction is vested, the same is retained up to the
daughter of Jose Aruego with Luz Fabian end of the litigation (Bernate v. Court of Appeals, 263 SCRA 323).

Motion for Partial Reconsideration was filed of the decision alleging loss of jurisdiction on the part of the trial court over the complaint by virtue of the passage of Executive RTC
Order No. 209 (as amended by Executive Order No. 227), otherwise known as the Family Code of the Philippines which took effect on August 3, 1988. This motion was
denied by the lower court in the Order, dated January 14, 1993. Decision: Granted the Motion to dismiss.

Petitioners interposed an appeal but the lower court refused to give it due course on the ground that it was filed out of time. Basis: Pronouncement of the Supreme Court in Ong v. C.A. 253 SCRA 105 and in the case of Hernandez v. Rural Bank of Lucena, Inc., G.R. No. L-29791 dated January
10, 1978, wherein it was held that the liquidation court shall have jurisdiction to adjudicate all claims against the bank whether they be against assets of the insolvent bank,
CA for Specific Performance, Breach of Contract, Damages or whatever.

Petitioners filed a Petition for Prohibition and Certiorari with prayer for a Writ of Preliminary Injunction. CA

Decision: CA dismissed the petition for lack of merit. Lucia appealed the RTC ruling to the CA on the ground that the RTC-Iriga erred in dismissing the case because it had jurisdiction over Civil Case No. IR-3128 under the rule
on adherence of jurisdiction.
A motion for reconsideration was filed. It was denied by CA in a minute resolution dated Oct 13, 1993.
CA ordered the consolidation of the Civil Case No. IR-3128 and the liquidation case pending before RTC-Makat to prevent confusion, to avoid multiplicity of suits and to save
ISSUE: WON RTC lost jurisdiction of the case on the ground of prescription. unnecessary cost and expense.
Judge was aware that he was acting on matters pertaining to the execution phase of a final decision of a co-equal and coordinate court since he even quoted MSUs allegations in his April 8,
Lucia filed a motion for reconsideration but it was denied by the CA in its Resolution dated December 14, 2006. 2009 Order

ISSUE: WON RTC IS VESTED WITH JURISDICTION TO CONTINUE TRYING AND ULTIMATELY DECIDE CIVIL CASE NO. IR-3128 The respondent Judge should have refrained from acting on the petition because Branch 6 of the Iligan City RTC retains jurisdiction to rule on any question on the enforcement of the writ of
execution. Section 16, Rule 39 of the Rules of Court (terceria), cited in the course of the Courts deliberations, finds no application to this case since this provision applies to claims made by
a third person, other than the judgment obligor or his agent; a third-party claimant of a property under execution may file a claim with another court which, in the exercise of its own
RULING: jurisdiction, may issue a temporary restraining order. In this case, the petition for injunction before the respondent Judge was filed by MSU itself, the judgment obligor. If Sheriff Gaje
committed any irregularity or exceeded his authority in the enforcement of the writ, the proper recourse for MSU was to file a motion with, or an application for relief from, the same court
NO. which issued the decision, not from any other court, or to elevate the matter to the CA on a petition for certiorari. In this case, MSU filed the proper motion with the Iligan City RTC (the
issuing court), but, upon denial, proceeded to seek recourse through another co-equal court presided over by the respondent Judge.
The Court recognizes the doctrine on adherence of jurisdiction. However, such principle is not without exceptions. It is well to quote the ruling of the CA on this matter, thus:
It is not a viable legal position to claim that a TRO against a writ of execution is issued against an erring sheriff, not against the issuing Judge. A TRO enjoining the enforceability of a writ
This Court is not unmindful nor unaware of the doctrine on the adherence of jurisdiction. However, the rule on adherence of jurisdiction is not absolute and has exceptions. addresses the writ itself, not merely the executing sheriff. The duty of a sheriff in enforcing writs is ministerial and not discretionary.
One of the exceptions is that when the change in jurisdiction is curative in character.
As already mentioned above, the appropriate action is to assail the implementation of the writ before the issuing court in whose behalf the sheriff acts, and, upon failure, to seek redress
For sure, Section 30, R.A. 7653 is curative in character when it declared that the liquidation court shall have jurisdiction in the same proceedings to assist in the adjudication through a higher judicial body. Significantly, MSU did file its opposition before the issuing court Iligan City RTC which denied this opposition.
of the disputed claims against the Bank. The interpretation of this Section (formerly Section 29, R.A. 265) becomes more obvious in the light of its intent. In Manalo v. Court
of Appeals (366 SCRA 752, [2001]), the Supreme Court says: That the respondent Judge subsequently rectified his error by eventually dismissing the petition before him for lack of jurisdiction is not a defense that the respondent Judge can use. His
lack of familiarity with the rules in interfering with the acts of a co-equal court undermines public confidence in the judiciary through his demonstrated incompetence. In this case, he
xxx The requirement that all claims against the bank be pursued in the liquidation proceedings filed by the Central Bank is intended to prevent multiplicity of actions against impressed upon the Iligan public that the kind of interference he exhibited can be done, even if only temporarily, i.e., that an official act of the Iligan City RTC can be thwarted by going to the
the insolvent bank and designed to establish due process and orderliness in the liquidation of the bank, to obviate the proliferation of litigations and to avoid injustice and Marawi City RTC although they are co-equal courts. That the complaining lawyer, Atty. Tomas Ong Cabili, subsequently reversed course and manifested that the respondent Judge is
arbitrariness. The lawmaking body contemplated that for convenience, only one court, if possible, should pass upon the claims against the insolvent bank and that the basically a good Judge, and should only be reprimanded, cannot affect the respondent Judges liability. This liability and the commensurate penalty do not depend on the complainant’s
liquidation court should assist the Superintendents of Banks and regulate his operations. personal opinion but on the facts he alleged and proved, and on the applicable law and jurisprudence. When the law is sufficiently basic, a judge owes it to his office to know and to simply
apply it. Anything less would be constitutive of gross ignorance of the law.

It is clear, therefore, that the liquidation court has jurisdiction over all claims, including that of Lucia against the insolvent bank. As declared in Miranda v. Philippine Deposit Insurance
TOPIC: DOCTRINE OF NON-INTERFERENCE OR JUDICIAL STABILITY
Corporation, regular courts do not have jurisdiction over actions filed by claimants against an insolvent bank, unless there is a clear showing that the action taken by the BSP, through the
Monetary Board, in the closure of financial institutions was in excess of jurisdiction, or with grave abuse of discretion. The same is not obtaining in this present case.
HON. JUDGE ADRIANO R. VILLAMOR VS. HON. JUDGE BERNARDO LL. SALAS
TOPIC: DOCTRINE OF NON-INTERFERENCE OR JUDICIAL STABILITY
CITATION: G.R. NO. 101041 NOVEMBER 13, 1991

ATTY. TOMAS ONG CABILI VS. JUDGE RASAD G. BALINDONG FACTS: In 1977, Civil Case No. B-398 (Gloria Naval vs. George Carlos) for recovery of ownership of a parcel of coconut land was filed and subsequently raffled to the sala of the petitioner,
Judge Adriano Villamor. While the civil case was pending there, respondent Carlos filed Criminal Cases Nos. N-989, N-990, N-991, N-992 and N-993 for qualified theft against Gloria Naval
CITATION: A.M. NO. RTJ-10-2225 SEPTEMBER 6, 2011 and her helpers. The criminal cases were also assigned to the sala of Judge Villamor.

CASE: Administrative complaint against respondent Acting Presiding Judge Rasad G. Balindong of the Regional Trial Court of Marawi City, Branch 8, for Gross Ignorance of the Law, Grave Due to the pendency of Civil Case No. B-398, the criminal cases were temporarily archived. After trial in Civil Case No. B-398, a decision was rendered in favor of Naval who was declared
Abuse of Authority, Abuse of Discretion, and/or Grave Misconduct Prejudicial to the Interest of the Judicial Service. the lawful owner and possessor of the disputed land. Carlos was ordered to vacate the land.

FACTS: Civil Case No. 06-2954 is an action for damages in Branch 6 of the Iligan City RTC against the Mindanao State University, et al., arising from a vehicular accident that caused the Thereafter, respondent Carlos, through counsel, moved to activate the archived criminal cases. Having declared Naval the lawful owner and possessor of the contested land in Civil Case
death of Jesus Ledesma and physical injuries to several others. No. B-398, Judge Villamor dismissed the criminal cases against her and her co-accused.

On November 29, 1997, the Iligan City RTC rendered a Decision, holding the MSU liable for damages amounting to P2,726,189.90. The Court of Appeals affirmed the Iligan City RTC Judge Villamor likewise granted execution pending appeal of his decision in Civil Case No. B-398. This order was challenged by Carlos in the Court of Appeals and in this Court, both without
decision and the CA decision subsequently lapsed to finality. On January 19, 2009, Entry of Judgment was made. success.

On March 10, 2009, the Iligan City RTC issued a writ of execution. The MSU, however, failed to comply with the writ; thus, on March 24, 2009, Sheriff Gerard Peter Gaje served a Notice of Administrative case
Garnishment on the MSUs depository bank, the Land Bank of the Philippines (LBP), Marawi City Branch.
Carlos filed an administrative case, A.M. No. RTJ-87-105, against Judge Villamor, charging him with having issued illegal orders and an unjust decision in Civil Case No. B-398.
Office of the Solicitor General opposed the motion for execution. The Iligan City RTC denied the opposition. MSU responded to the denial by filing on April 1, 2009 a petition with the Marawi
City RTC, for prohibition and mandamus with an application for the issuance of a temporary restraining order (TRO) and/or preliminary injunction against the LBP and Sheriff Gaje. The Decision: SC, in an En Banc resolution, summarily dismissed the administrative case on November 21, 1988.
petition of MSU was raffled to the RTC, Marawi City, Branch 8, presided by respondent Judge.
Civil Action for Damages
The respondent Judge set the hearing for the application for the issuance of a TRO on April 8, 2009. After which he issued a TRO restraining Sheriff Gaje from garnishing P2,726,189.90
from MSUs LBP-Marawi City Branch account. Civil Case No. CEB-6478

On April 17, 2009, the respondent Judge conducted a hearing on the application for the issuance of a writ of preliminary injunction. Thereafter, he required MSU to file a memorandum in Cause of action: For knowingly rendering an unjust judgment when he dismissed the five (5) criminal cases against Naval, et al.
support of its application for the issuance of a writ of preliminary injunction. On April 21, 2009, Sheriff Gaje moved to dismiss the case on the ground of lack of jurisdiction. The respondent
Judge thereafter granted the motion and dismissed the case. The summons in Civil Case No. CEB-6478 was served upon Judge Villamor on December 10,1987.

On May 8, 2009, complainant Atty. Tomas Ong Cabili, counsel of the private plaintiffs in Civil Case No. 06-2954, filed the instant complaint charging the respondent Judge with Gross The next day, instead of answering the complaint, Judge Villamor issued in Criminal Cases Nos. N-0989 to 0993 an order of direct contempt against Carlos
Ignorance of the Law, Grave Abuse of Authority, Abuse of Discretion, and/or Grave Misconduct Prejudicial to the Interest of the Judicial Service for interfering with the order of a co-equal and his lawyer. Attorney Antonio T. Guerrero, "for degrading the respect and dignity of the court through the use of derogatory and contemptous language
court, Branch 6 of the Iligan City RTC, by issuing the TRO to enjoin Sheriff Gaje from garnishing P2,726,189.90 from MSUs LBP-Marawi City Branch account. before the court," and sentenced each of them to suffer the penalty of imprisonment for five (5) days and to pay a fine of P500.

The respondent Judge denied that he interfered with the order of Branch 6 of the Iligan City RTC. He explained that he merely gave the parties the opportunity to be heard and eventually Carlos immediately filed in SC a petition for certiorari with a prayer for the issuance of a writ of preliminary injunction against the Judge.
dismissed the petition for lack of jurisdiction.
Decision: SC promptly restrained Judge Villamor from enforcing his Order of Contempt against Carlos and Attorney Guerrero.

Office of the Court Administrator (OCA)


Decision: Found the respondent Judge guilty of gross ignorance of the law for violating the elementary rule of non-interference with the proceedings of a court of co-equal Judge Villamor filed a motion to dismiss the complaint for lack of jurisdiction. The trial court granted the motion. The order of dismissal was affirmed by the Court of Appeals.
jurisdiction. It recommended a fine of P40,000.00, noting that this is the respondent Judges second offense. Carlos appealed to this Court which also denied the petition.

Unfazed by these setbacks, Carlos and his counsel, Attorney Antonio Guerrero, filed separate complaints for damages against Judge Villamor for knowingly rendering an unjust order of
ISSUE: WON respondent judge violates the rule on non-interference with the proceedings of a court of co-equal jurisdiction contempt.

RULING:
Civil Case No. CEB-8802
YES.
Atty Guerrero filed a complaint for damages was raffled to Branch 21, Regional Trial Court, Cebu City, presided over by Judge Peary G. Aleonar. Carlos' complaint for
The doctrine of judicial stability or non-interference in the regular orders or judgments of a co-equal court is an elementary principle in the administration of justice: no court can interfere by damages was docketed as Civil Case No. CEB-8823 and raffled to Branch 8, Regional Trial Court of Cebu City presided over by Judge Bernardo LL. Salas.
injunction with the judgments or orders of another court of concurrent jurisdiction having the power to grant the relief sought by the injunction. The rationale for the rule is founded on the
concept of jurisdiction: a court that acquires jurisdiction over the case and renders judgment therein has jurisdiction over its judgment, to the exclusion of all other coordinate courts, for its Judge Villamor filed a motion to dismiss but it was denied.
execution and over all its incidents, and to control, in furtherance of justice, the conduct of ministerial officers acting in connection with this judgment.
1st SC Case: Petition for certiorari and prohibition with restraining order (G.R. No. 101296)
Thus, we have repeatedly held that a case where an execution order has been issued is considered as still pending, so that all the proceedings on the execution are still proceedings in the
suit. A court which issued a writ of execution has the inherent power, for the advancement of justice, to correct errors of its ministerial officers and to control its own processes. To hold Decision: SC issued a TRO against Judge Aleonar to stop him from proceeding in Civil Case No. CEB8802
otherwise would be to divide the jurisdiction of the appropriate forum in the resolution of incidents arising in execution proceedings. Splitting of jurisdiction is obnoxious to the orderly
administration of justice. On May 20, 1991, a Manifestation was filed by Judge Villamor praying Judge Salas to dismiss Civil Case No. CEB- 8823 but the motion was denied by respondent Judge on
July 2, 1991.
Jurisprudence shows that a violation of this rule warrants the imposition of administrative sanctions.
2nd SC Case: Petition for certiorari and prohibition with restraining order (G.R. No. 101041)
In the present case, the respondent Judge clearly ignored the principle of judicial stability by issuing a TRO to temporarily restrain Sheriff Gaje from enforcing the writ of execution issued by
a co-equal court, Branch 6 of the Iligan City RTC, and from pursuing the garnishment of the amount of P2,726,189.90 from MSUs account with the LBP, Marawi City Branch. The respondent Court Resolution:
1. Temporarily restraining Judge Salas from further proceeding in Civil Case No. CEB-8823;
Petitioners share in Alicia’s estate was sold in a public auction on February 26, 2003. Based on the Commissioners Report on the Auction Sale, there were two bidders, Apolonio Marasigan
2. Granting the petitioner's prayer that this case be consolidated with G.R. No. 101296 (pp. 37-39, Rollo of G.R. No. 101041). and Amado Lazaro. Apolonio, with a bid of P701,000.00 per hectare, won over Amado Lazaro, whose bid was P700,000.00 per hectare. Petitioners 1/7 share as Cesars heirs in Alicias
estate was sold in the public auction for P3,777,689.00. This amount is lower than the P4,724,726.30 price of the 1/7 share in Alicia’s estate as earlier determined by the Commissioners due
allegedly to the acquisition by the Department of Agrarian Reform (DAR) of a portion of Hacienda Sta. Rita located in Minilabac, Camarines Sur which was placed under Republic Act No.
ISSUE: WON Judges Aleonar and Salas may take cognizance of the actions for damagesagainst Judge Villamor for allegedly having rendered an unjust order of direct contempt against 6657, or the Comprehensive Agrarian Reform Law, with 100.00 hectares thereof compulsorily acquired.
Carlos and Attorney Guerrero which this Court subsequently annulled.
Petitioners filed with the RTC a Motion to Declare Failure of Bidding and to Annul Public Auction Sale on March 24, 2003.
RULING:
On May 5, 2003, however, the RTC released an Omnibus Order ruling, among other things, that the objection of petitioners as to the difference of the value of their 1/7 share as determined
NO. by the Commissioners vis--vis the winning bid was no longer an issue since Apolonio Marasigan indicated his willingness to pay for the deficiency.

No Regional Trial Court can pass upon and scrutinize, and much less declare as unjust a judgment of another Regional Trial Court and sentence the judge thereof liable for damages without Following the public auction and sale of their 1/7 share in the property, petitioners filed a Notice of Appeal with the RTC indicating that they were appealing the May 5, 2003 Omnibus Order
running afoul with the principle that only the higher appellate courts, namely, the Court of Appeals and the Supreme Court, are vested with authority to review and correct errors of the trial of the RTC to the Court of Appeals. Thereafter, petitioners filed a Record on Appeal pursuant to Section 3, Rule 41 of the Rules of Court, praying that it be approved and transmitted to the
courts. Court of Appeals.

To allow respondent Judges Aleonar and Salas to proceed with the trial of the actions for damages against the petitioner, a co-equal judge of a co-equal court, would in effect permit a court The RTC issued an Order denying due course to petitioners Notice of Appeal on the ground that the proper remedy is not appeal, but certiorari. Petitioners then filed another Petition before
to review and interfere with the judgment of a co-equal court over which it has no appellate jurisdiction or power of review. The various branches of a Court of First Instance (now the the Court of Appeals for Certiorari and Mandamus, praying that the RTC be directed to approve their Notice of Appeal and Record on Appeal, and to forward the same to the appellate court.
Regional Trial Court) being co-equal, may not interfere with each other's cases, judgments and orders.
In a Resolution dated October 10, 2003, the Court of Appeals dismissed CA-G.R. SP No. 78912 outright on the ground that the verification and certificate of non-forum shopping of the
It is only after the Appellate Court, in a final judgment, has found that a trial judge's errors were committed deliberately and in bad faith may a charge of knowingly rendering an unjust petition was signed by only Cesar Marasigan, Jr., without any accompanying document to prove his authority to sign on behalf of the other petitioners. Petitioners filed a Motion for
decision be levelled against the latter. Reconsideration but it was denied by the Court of Appeals in a Resolution dated July 12, 2004.

TOPIC: LACK OF JURISDICTION – RAISED AT EARLIEST POSSIBLE OPPORTUNITY ISSUE: WON the Court of Appeals erred in affirming in toto the RTC Order adopting the Commissioners’ recommendation on the manner of partition of the estate of Alicia Marasigan

HEIRS OF CESAR MARASIGAN VS. APOLONIO MARASIGAN, ET AL.


RULING:
CITATION: G.R. NO. 156078 MARCH 14, 2008
YES.
FACTS: Alicia was survived by her siblings: Cesar, Apolonio, Lilia, and Benito; Marissa, a sister-in-law; and the children of her brothers who predeceased her: Francisco, Horacio, and
Octavio. She died intestate and without issue on 21 January 1995. First, it bears to point out that Cesar, petitioner’s predecessor, did not file any motion to dismiss, and his answer before the RTC did not bear the defenses/objections of lack of jurisdiction or
cause of action on these grounds; consequently, these must be considered waived. The exception that the court may still dismiss a case for lack of jurisdiction over the subject matter,
On December 17, 1997, a Complaint for Judicial Partition of the Estate of Alicia Marasigan was filed before the RTC by several of her heirs and private respondents herein, namely, although the same is not pleaded, but is apparent in the pleadings or evidence on record, does not find application to the present petition. Second, petitioner’s arguments on the lack of
Apolonio, Lilia, Octavio, Jr., Horacio, Benito, Jr., and Marissa, against Cesar, docketed as Special Civil Action No. P-77-97. jurisdiction of the RTC over the case more appropriately pertain to venue, rather than jurisdiction over the subject matter, and are, moreover, not apparent from the pleadings and evidence
on record. Third, the property subject of partition is only the 47.2 hectare pro-indiviso area representing the estate of Alicia. It does not include the entire 496 hectares of land comprising
According to private respondents, Alicia owned in common with her siblings 13 parcels of land called Hacienda Sta. Rita in Pili and Minalabac, Camarines Sur, with an aggregate area of Hacienda Sta. Rita.
4,960,963 square meters or 496 hectares.
Even petitioners argument that non-payment of appropriate docket fees by private respondents deprived the RTC of jurisdiction to partition the entire Hacienda Sta. Rita deserves scant
Alicia left behind her 2/21 shares in the 13 parcels of land. consideration. In National Steel Corporation v. Court of Appeals, the Court ruled:

In answer to the private respondents Complaint, Cesar enumerated Alicia’s several other properties and assets which he also wanted included in the action for partition. x x x while the lack of jurisdiction of a court may be raised at any stage of an action, nevertheless, the party raising such question may be estopped if he has actively taken
part in the very proceedings which he questions and he only objects to the court’s jurisdiction because the judgment or the order subsequently rendered is adverse to him.
Cesar’s request for inclusion was contested by private respondents on the ground that the properties he enumerated had already been previously partitioned and distributed to the
appropriate parties.
Irrefragably, petitioners raised the issues of jurisdiction for lack of payment of appropriate docket fees and lack of cause of action belatedly in their Memorandum before this Court. Cesar and
RTC petitioners’ were noticeably mum about these in the proceedings before. In fact, Cesar actively participated in the proceedings conducted before the RTC by seeking affirmative reliefs
Decided in favor of private respondents and issued an Order of Partition of the Estate of Alicia Marasigan therefrom, such as the inclusion of more properties in the partition. Hence, petitioners are already estopped from assailing the jurisdiction of the RTC on this ground.

As the parties could not agree on how they shall physically partition among themselves Alicias estate, private respondents filed a Motion to Appoint Commissioners following the procedure It is conceded that this Court adheres to the policy that where the court itself clearly has no jurisdiction over the subject matter or the nature of the action, the invocation of this defense may
outlined in Sections 4, 5, 6 and 7 of Rule 69 of the Rules of Court. be done at any time. While it is the general rule that neither waiver nor estoppel shall apply to confer jurisdiction upon a court, the Court may rule otherwise under meritorious and
exceptional circumstances. One such exception is Tijam v. Sibonghanoy, which finds application in this case. This Court held in Tijam that after voluntarily submitting a cause and
The RTC granted the Motion and appointed Myrna V. Badiong, Assistant Provincial Assessor of Camarines Sur, as Chairman of the Board of Commissioners. encountering an adverse decision on the merits, it is too late for the loser to question the jurisdiction or power of the court."

Private respondents nominated Sandie B. Dacara as the second commissioner. Cesar failed to nominate a third commissioner despite due notice. Upon lapse of the period given, only two After an exhaustive study of the merits of the case and the pleadings submitted by the parties, this Court is convinced that the Court of Appeals did not err in affirming the Order of the RTC
commissioners were appointed which approved the Commissioners recommendations as to the manner of implementing the Order of Partition of Alicia’s estate. There is no reason to reverse the Court of Appeals dismissal
of petitioners Petition for Certiorari and Prohibition and ruling that the RTC acted well-within its jurisdiction in issuing the assailed Order. Nowhere is it shown that the RTC committed such
Cesar opposed the foregoing findings and prayed for the disapproval of the Commissioners Report. patent, gross and prejudicial errors of law or fact, or a capricious disregard of settled law and jurisprudence, as to amount to a grave abuse of discretion or lack of jurisdiction on its part, in
adopting and confirming the recommendations submitted by the Commissioners, and which would have warranted the issuance of a writ of certiorari.
Petitioner’s argument:
The estate is not indivisible just because of the different locations and conditions of the parcels of land constituting the same. Section 5, Rule 69 of the Rules of Court can TOPIC: LACK OF JURISDICTION – RAISED AT EARLIEST POSSIBLE OPPORTUNITY
only be availed of if the partition or division of the real properties involved would be prejudicial to the interest of any of the parties. He asserted that despite the segregation of
his share, the remaining parcels of land would still be serviceable for the planting of rice, corn, and sugarcane, thus evidencing that no prejudice would be caused to the
interests of his co-heirs.
SERAFIN TIJAM VS. MAGDALENO SIBONGHANOY
Respondents’ answer:
The physical division is impossible because Alicia’s estate is equivalent to 2/21 shares in Hacienda Sta. Rita, which is composed of 13 parcels under different titles and tax CITATION: G.R. NO. L-21450 APRIL 15, 1968
declarations, situated in different barangays and municipalities, and covers an area of 496 hectares.
FACTS: On July 19, 1948, spouses Serafin Tijam and Felicitas Tagalog commenced Civil Case No. R-660 in the Court of First Instance of Cebu against the spouses Magdaleno
Sibonghanoy and Lucia Baguio to recover from them the sum of P1,908.00, with legal interest thereon from the date of the filing of the complaint until the whole obligation is paid, plus costs.
RTC Decision: Issued an Order dated June 22, 2001 approving in toto the recommendations embodied in the Commissioners Report, particularly, the recommendation that the property be
assigned to one of the heirs at P700,000.00 per hectare or a total amount of P4,724,726.00, after finding the same to be in accordance with the Rules of Court and the New Civil Code. As prayed for in the complaint, a writ of attachment was issued by the court against defendants' properties, but the same was soon dissolved upon the filing of a counter-bond by defendants
and the Manila Surety and Fidelity Co., Inc. hereinafter referred to as the Surety, on the 31st of the same month.
Cesar filed a Motion for Reconsideration which was denied by RTC for lack of merit.
After being duly served with summons the defendants filed their answer in which, after making some admissions and denials of the material averments of the complaint, they interposed a
CA counterclaim. This counterclaim was answered by the plaintiffs.

(Cesar died on 25 October 2001. He was substituted by his heirs and herein petitioners, namely, Luz Regina, Cesar, Jr., Benito, Santiago, Renato, Jose, Geraldo, Orlando, Peter, Paul, RTC
Mauricio, Rommel, Michael, Gabriel, and Maria Luz, all surnamed Marasigan.)
Decision: Rendered judgment in favor of the petitioner and issued a writ of execution against the defendants.
Action: Petition for Certiorari and Prohibition under Rule 65 of the Rules of Court
The writ having been returned unsatisfied, the plaintiffs moved for the issuance of a writ of execution against the Surety's bond,
CA decision: Dismissed petition and ruled that the RTC acted within its authority in issuing the Order of June 22, 2001. The Court of Appeals found that petitioners failed to discharge the
burden of proving that the proceedings before the Board of Commissioners were unfair and prejudicial. It likewise found that the petitioners were not denied due process considering that Surety’s Action: filed a written opposition upon two grounds, namely:
they were afforded the opportunity to be heard during the hearing for approval of the Commissioners Report on January 18, 2001. According to the appellate court, whether or not the
physical division of the estate will cause prejudice to the interests of the parties is an issue addressed to the discretion of the Commissioners. 1. Failure to prosecute
2. Absence of a demand upon the Surety for the payment of the amount due under the judgment.
A motion for Reconsideration was filed by the petitioners but the same was denied by CA.

Petitioners filed a petition for review in SC under Rule 45 of the Revised Rules of Court. Pending resolution of the instant Petition by this Court, the RTC granted private respondents Urgent Surety’s Prayer: Prayed the Court not only to deny the motion for execution against its counter-bond but also the following affirmative relief : "to relieve the herein bonding company
Motion for Execution on December 26, 2002. The RTC ordered the sale of petitioners 1/7 pro-indiviso share in Alicia’s estate upon the urgent motion of private respondents dated September of its liability, if any, under the bond in question"
27, 2002 for the partial execution of the judgment of the Court approving the Commissioners report pending certiorari.
Court’s Decision: The Court denied this motion on the ground solely that no previous demand had been made on the Surety for the satisfaction of the judgment.
Thereafter the necessary demand was made, and upon failure of the Surety to satisfy the judgment, the plaintiffs filed a second motion for execution against the counterbond. On the date On July 18, 1997, the RTC issued an Order setting the case for initial hearing on October 22, 1997. On August 7, 1997, it issued a second Order setting the initial hearing on November 4,
set for the hearing thereon, the Court, upon motion of the Surety's counsel, granted the latter a period of five days within which to answer the motion. Upon its failure to file such answer, the 1997.
Court granted the motion for execution and the corresponding writ was issued.
Petitioner Republic filed its Opposition to the application for registration on January 8, 1998 while the records were still with the RTC. On March 31, 1998, the RTC Clerk of Court transmitted
Surety’s Action: moved to quash the writ on the ground that the same was issued without the required summary hearing provided for in Section 17 of Rule 59 of the Rules of motu proprio the records of the case to the MTC of San Juan, because the assessed value of the property was allegedly less than ₱100,000.
Court.
Thereafter, the MTC entered an Order of General Default and commenced with the reception of evidence. Among the documents presented by respondent in support of its application are
Court’s Decision: Denied the Motion Tax Declarations, a Deed of Absolute Sale in its favor, and a Certification from the Department of Environment and Natural Resources (DENR) Community Environment and Natural
Resources Office (CENRO) of Batangas City that the lot in question is within the alienable and disposable zone. Thereafter, it awarded the land to respondent Corporation.
CA
Acting on an appeal filed by the Republic, the CA ruled that since the former had actively participated in the proceedings before the lower court, but failed to raise the jurisdictional challenge
Surety appealed to the CA from such order of denial and from the one denying its motion for reconsideration therein, petitioner is thereby estopped from questioning the jurisdiction of the lower court on appeal. The CA further found that respondent Corporation had sufficiently established the latter’s
registrable title over the subject property after having proven open, continuous, exclusive and notorious possession and occupation of the subject land by itself and its predecessors-in-
CA Decision: Affirmed the orders appealed from. interest even before the outbreak of World War II.

Surety filed a Motion to Dismiss alleging substantially that appellees action was filed in the Court of First Instance of Cebu on July 19, 1948 for the recovery of the sum of P1,908.00 only; Dissatisfied with the CA’s ruling, petitioner Republic filed this instant Rule 45 Petition.
that a month before that date Republic Act No. 296, otherwise known as the Judiciary Act of 1948, had already become effective, Section 88 of which placed within the original exclusive
jurisdiction of inferior courts all civil actions where the value of the subject-matter or the amount of the demand does not exceed P2,000.00, exclusive of interest and costs. The Court of First Issue:
Instance therefore had no jurisdiction to try and decide the case. Whether or not petitioner Republic is not estopped from questioning the jurisdiction of the Municipal Trial Court over the application for original registration of land title even for the first time
on appeal.
Surety’s prayer: Prayed the Court of Appeals to set aside its decision and to dismiss the case.
Ruling:
CA’s resolution: CA required the appellees to answer the motion to dismiss, but they failed to do so. At the outset, we rule that petitioner Republic is not estopped from questioning the jurisdiction of the lower court, even if the former raised the jurisdictional question only on appeal. The rule
is settled that lack of jurisdiction over the subject matter may be raised at any stage of the proceedings. Jurisdiction over the subject matter is conferred only by the
Decision: Constitution or the law. It cannot be acquired through a waiver or enlarged by the omission of the parties or conferred by the acquiescence of the court. Consequently,
questions of jurisdiction may be cognizable even if raised for the first time on appeal.
It would indeed appear from the record that the action at bar, which is a suit for collection of money in the sum of exactly P1,908.00 exclusive of interest, was originally
instituted in the Court of First Instance of Cebu on July 19, 1948. But about a month prior to the filing of the complaint, more specifically on June 17, 1948, the Judiciary Act The ruling of the Court of Appeals that "a party may be estopped from raising such [jurisdictional] question if he has actively taken part in the very proceeding which he questions, belatedly
of 1948 took effect, depriving the Court of First Instance of original jurisdiction over cases in which the demand, exclusive of interest, is not more than P2,000.00. objecting to the court’s jurisdiction in the event that the judgment or order subsequently rendered is adverse to him" is based on the doctrine of estoppel by laches. We are aware of that
doctrine first enunciated by this Court in Tijam v. Sibonghanoy. In Tijam, the party-litigant actively participated in the proceedings before the lower court and filed pleadings therein. Only 15
Xxxx years thereafter, and after receiving an adverse Decision on the merits from the appellate court, did the party-litigant question the lower court’s jurisdiction. Considering the unique facts in
that case, we held that estoppel by laches had already precluded the party-litigant from raising the question of lack of jurisdiction on appeal. In Figueroa v. People, we cautioned that Tijam
Considering, however, that the Supreme Court has the "exclusive" appellate jurisdiction over "all cases in which the jurisdiction of any inferior court is in issue" (See. 1, Par. must be construed as an exception to the general rule and applied only in the most exceptional cases whose factual milieu is similar to that in the latter case.
3[3], Judiciary Act of 1948, as amended), we have no choice but to certify, as we hereby do certify, this case to the Supreme Court
The facts are starkly different in this case, making the exceptional rule in Tijam inapplicable. Here, petitioner Republic filed its Opposition to the application for registration when the records
ACCORDINGLY, pursuant to Section 31 of the Judiciary Act of 1948 as amended, let the record of this case be forwarded to the Supreme Court. were still with the RTC. At that point, petitioner could not have questioned the delegated jurisdiction of the MTC, simply because the case was not yet with that court. When the records were
transferred to the MTC, petitioner neither filed pleadings nor requested affirmative relief from that court. On appeal, petitioner immediately raised the jurisdictional question in its Brief.
Clearly, the exceptional doctrine of estoppel by laches is inapplicable to the instant appeal.

ISSUE: WON the motion to dismiss on the ground of lack of jurisdiction of the Court of First Instance during the pendency of the appeal will prosper. Laches has been defined as the "failure or neglect, for an unreasonable and unexplained length of time, to do that which, by exercising due diligence, could or should have been done
earlier; it is negligence or omission to assert a right within a reasonable time, warranting the presumption that the party entitled to assert it either has abandoned or declined to assert it." In
RULING: this case, petitioner Republic has not displayed such unreasonable failure or neglect that would lead us to conclude that it has abandoned or declined to assert its right to question the lower
court's jurisdiction.
NO.
We uphold the jurisdiction of the MTC, but remand the case to the court a quo for further proceedings in order to determine if the property in question forms part of the alienable and
It is an undisputed fact that the action commenced by appellees in the Court of First Instance of Cebu against the Sibonghanoy spouses was for the recovery of the sum of P1,908.00 only disposable land of the public domain.
— an amount within the original exclusive jurisdiction of inferior courts in accordance with the provisions of the Judiciary Act of 1948 which had taken effect about a month prior to the date
when the action was commenced. True also is the rule that jurisdiction over the subject matter is conferred upon the courts exclusively by law, and as the lack of it affects the very authority
SERAFIN TIJAM, ET AL. vs. MAGDALENO SIBONGHANOY alias GAVINO SIBONGHANOY and LUCIA BAGUIO, MANILA SURETY AND FIDELITY CO., INC. (CEBU BRANCH)
of the court to take cognizance of the case, the objection may be raised at any stage of the proceedings. However, considering the facts and circumstances of the present case — which
G.R. No. L-21450 April 15, 1968
shall forthwith be set forth — We are of the opinion that the Surety is now barred by laches from invoking this plea at this late hour for the purpose of annulling everything done heretofore in
the case with its active participation.
Topic: Lack of jurisdiction over subject matter may be raised at any stage
As already stated, the action was commenced in the Court of First Instance of Cebu on July 19, 1948, that is, almost fifteen years before the Surety filed its motion to dismiss on January 12,
Facts:
1963 raising the question of lack of jurisdiction for the first time.
Barely one month after the effectivity of the Judiciary Act of 1948, spouses Tijam commenced Civil Case No. R-660 in the Court of First Instance of Cebu against the spouses Sibonghanoy
to recover from them the sum of P1,908.00, with legal interest thereon from the date of the filing of the complaint until the whole obligation is paid, plus costs. A writ of attachment was issued
A party may be estopped or barred from raising a question in different ways and for different reasons. Thus we speak of estoppel in pais, or estoppel by deed or by record, and of estoppel
by the court against defendants' properties, but the same was soon dissolved upon the filing of a counter-bond by defendants and the Manila Surety and Fidelity Co., Inc. After being duly
by laches.
served with summons the defendants filed their answer in which, after making some admissions and denials of the material averments of the complaint, they interposed a counterclaim. This
counterclaim was answered by the plaintiffs.
Laches, in a general sense is failure or neglect, for an unreasonable and unexplained length of time, to do that which, by exercising due diligence, could or should have been done earlier; it
is negligence or omission to assert a right within a reasonable time, warranting a presumption that the party entitled to assert it either has abandoned it or declined to assert it.
The Court rendered judgment in favor of the plaintiffs and, after the same had become final and executory, upon motion of the latter, the Court issued a writ of execution against the
defendants. The writ having been returned unsatisfied, the plaintiffs moved for the issuance of a writ of execution against the Surety's bond, against which the Surety filed a written
The doctrine of laches or of "stale demands" is based upon grounds of public policy which requires, for the peace of society, the discouragement of stale claims and, unlike the statute of
opposition upon two grounds, namely, (1) Failure to prosecute and (2) Absence of a demand upon the Surety for the payment of the amount due under the judgment.
limitations, is not a mere question of time but is principally a question of the inequity or unfairness of permitting a right or claim to be enforced or asserted.
The Court denied this motion on the ground solely that no previous demand had been made on the Surety for the satisfaction of the judgment. Thereafter the necessary demand was made,
It has been held that a party cannot invoke the jurisdiction of a court to sure affirmative relief against his opponent and, after obtaining or failing to obtain such relief, repudiate or question
and upon failure of the Surety to satisfy the judgment, the plaintiffs filed a second motion for execution against the counterbond. On the date set for the hearing thereon, the Court, upon
that same jurisdiction. In the case just cited, by way of explaining the rule, it was further said that the question whether the court had jurisdiction either of the subject-matter of the action or of
motion of the Surety's counsel, granted the latter a period of five days within which to answer the motion. Upon its failure to file such answer, the Court granted the motion for execution and
the parties was not important in such cases because the party is barred from such conduct not because the judgment or order of the court is valid and conclusive as an adjudication, but for
the corresponding writ was issued.
the reason that such a practice cannot be tolerated — obviously for reasons of public policy.
Subsequently, the Surety moved to quash the writ on the ground that the same was issued without the required summary hearing provided for in Section 17 of Rule 59 of the Rules of Court.
Furthermore, it has also been held that after voluntarily submitting a cause and encountering an adverse decision on the merits, it is too late for the loser to question the jurisdiction or power
As the Court denied the motion, the Surety appealed to the Court of Appeals from such order of denial and from the one denying its motion for reconsideration. Although the appellees failed
of the court (Pease vs. Rathbun-Jones etc). And in Littletonvs. Burgess, the Court said that it is not right for a party who has affirmed and invoked the jurisdiction of a court in a particular
to file their brief, the Court of Appeals, on December 11, 1962, decided the case affirming the orders appealed from.
matter to secure an affirmative relief, to afterwards deny that same jurisdiction to escape a penalty.
On January 8, 1963, the Surety filed a motion asking for extension of time within which to file a motion for reconsideration. The Court of Appeals granted the motion in its resolution of
Upon this same principle is what We said in the three cases mentioned in the resolution of the Court of Appeals of May 20, 1963 ( supra) — to the effect that we frown upon the "undesirable
January 10 of the same year. Thereafter, the Surety filed a pleading entitled MOTION TO DISMISS, alleging, among others, that the Court of First Instance had no jurisdiction to try and
practice" of a party submitting his case for decision and then accepting the judgment, only if favorable, and attacking it for lack of jurisdiction, when adverse.
decide the case and prayed the Court of Appeals to set aside its decision and to dismiss the case. The Court of Appeals required the appellees to answer the motion to dismiss, but failed to
do so. On May 20 of the same year, the Court resolved to set aside its decision and to certify the case to Us. Hence, this petition.
From the time the Surety became a quasi-party on July 31, 1948, it could have raised the question of the lack of jurisdiction of the Court of First Instance of Cebu to take cognizance of the
present action by reason of the sum of money involved which, according to the law then in force, was within the original exclusive jurisdiction of inferior courts. It failed to do so. Instead, at
Issue:
several stages of the proceedings in the court a quo as well as in the Court of Appeals, it invoked the jurisdiction of said courts to obtain affirmative relief and submitted its case for a final
Whether or not the surety bond is estopped from questioning the jurisdiction of the CFI Cebu for the first time upon appeal.
adjudication on the merits. It was only after an adverse decision was rendered by the Court of Appeals that it finally woke up to raise the question of jurisdiction. Were we to sanction such
conduct on its part, We would in effect be declaring as useless all the proceedings had in the present case since it was commenced on July 19, 1948 and compel the judgment creditors to
Ruling:
go up their Calvary once more. The inequity and unfairness of this is not only patent but revolting.
It is an undisputed fact that the action commenced by appellees in the Court of First Instance of Cebu against the Sibonghanoy spouses was for the recovery of the sum of P1,908.00 only
— an amount within the original exclusive jurisdiction of inferior courts in accordance with the provisions of the Judiciary Act of 1948 which had taken effect about a month prior to the date
when the action was commenced. True also is the rule that jurisdiction over the subject matter is conferred upon the courts exclusively by law, and as the lack of it affects the very authority
REPUBLIC OF THE PHILIPPINES vs. BANTIGUE POINT DEVELOPMENT CORPORATION of the court to take cognizance of the case, the objection may be raised at any stage of the proceedings. However, considering the facts and circumstances of the present case — which
G. R. No. 162322 March 14, 2012 shall forthwith be set forth — We are of the opinion that the Surety is now barred by laches from invoking this plea at this late hour for the purpose of annuling everything done
heretofore in the case with its active participation.
Topic: Lack of jurisdiction over subject matter may be raised at any stage
As already stated, the action was commenced in the Court of First Instance of Cebu on July 19, 1948, that is, almost fifteen years before the Surety filed its motion to dismiss on
Facts: January 12, 1963 raising the question of lack of jurisdiction for the first time.
On July 17, 1997, respondent Bantigue Point Development Corporation filed with the Regional Trial Court (RTC) of Rosario, Batangas an application for original registration of title over Lot
8060 of Cad 453-D, San Juan Cadastre, with an area of more or less 10,732 square meters and with a total assessed value of ₱14,920. It must be remembered that although the action, originally, was exclusively against the Sibonghanoy spouses the Surety became a quasi-party therein since July 31, 1948 when it filed a
counter-bond for the dissolution of the writ of attachment issued by the court of origin (Record on Appeal, pp. 15-19). Since then, it acquired certain rights and assumed specific obligations in Laches is defined as the "failure or neglect for an unreasonable and unexplained length of time, to do that which, by exercising due diligence, could or should have been done
connection with the pending case, in accordance with sections 12 and 17, Rule 57, Rules of Court. earlier, it is negligence or omission to assert a right within a reasonable length of time, warranting a presumption that the party entitled to assert it either has abandoned it or declined
to assert it."
Upon the filing of the first motion for execution against the counter-bond the Surety not only filed a written opposition thereto praying for its denial but also asked for an additional affirmative
relief — that it be relieved of its liability under the counter-bond upon the grounds relied upon in support of its opposition — lack of jurisdiction of the court a quo not being one of them. In Sibonghanoy, the defense of lack of jurisdiction was raised for the first time in a motion to dismiss filed by the Surety almost 15 years after the questioned ruling had been rendered. At
several stages of the proceedings, in the court a quo as well as in the Court of Appeals, the Surety invoked the jurisdiction of the said courts to obtain affirmative relief and submitted its case
Then, at the hearing on the second motion for execution against the counter-bond, the Surety appeared, through counsel, to ask for time within which to file an answer or opposition thereto. for final adjudication on the merits. It was only when the adverse decision was rendered by the Court of Appeals that it finally woke up to raise the question of jurisdiction.
This motion was granted, but instead of such answer or opposition, the Surety filed the motion to dismiss mentioned heretofore.
The factual settings attendant in Sibonghanoy are not present in the case at bar that would justify the application of estoppel by laches against the petitioner. Here, petitioner assailed the
A party may be estopped or barred from raising a question in different ways and for different reasons. Thus we speak of estoppel in pais, or estoppel by deed or by record, and of estoppel jurisdiction of the COSLAP when she appealed the case to the CA and at that time, no considerable period had yet elapsed for laches to attach. Therefore, petitioner is not
by laches. estopped from assailing the jurisdiction of the COSLAP. Additionally, no laches will even attach because the judgment is null and void for want of jurisdiction .

Laches, in a general sense is failure or neglect, for an unreasonable and unexplained length of time, to do that which, by exercising due diligence, could or should have been done earlier; it
MEGAN SUGAR CORPORATION vs. REGIONAL TRIAL COURT of ILOILO, Branch 68, Dumangas, Iloilo; New Frontier Sugar Corporation and EQUITABLE PCI BANK
is negligence or omission to assert a right within a reasonable time, warranting a presumption that the party entitled to assert it either has abandoned it or declined to assert it.
G.R. No. 170352 June 1, 2011
The doctrine of laches or of "stale demands" is based upon grounds of public policy which requires, for the peace of society, the discouragement of stale claims and, unlike the statute of
Topic: Objection to jurisdiction over a party; holding out theory or doctrine of ostensible agency
limitations, is not a mere question of time but is principally a question of the inequity or unfairness of permitting a right or claim to be enforced or asserted.
Facts:
It has been held that a party can not invoke the jurisdiction of a court to sure affirmative relief against his opponent and, after obtaining or failing to obtain such relief, repudiate or question
On July 23, 1993, respondent New Frontier Sugar Corporation (NFSC) obtained a loan from respondent Equitable PCI Bank (EPCIB) secured by a real estate mortgage over NFSC’s land
that same jurisdiction. In the case just cited, by way of explaining the rule, it was further said that the question whether the court had jurisdiction either of the subject-matter of the action or of
and a chattel mortgage over NFSC’s sugar mill. Due to liquidity problems, NFSC entered into a Memorandum of Agreement (MOA) with Central Iloilo Milling Corporation (CIMICO), whereby
the parties was not important in such cases because the party is barred from such conduct not because the judgment or order of the court is valid and conclusive as an adjudication, but for
the latter agreed to take-over the operation and management of the NFSC raw sugar factory and facilities for the period covering crop years 2000 to 2003.
the reason that such a practice can not be tolerated — obviously for reasons of public policy.
On April 19, 2002, NFSC filed a compliant for specific performance and collection against CIMICO for the latter's failure to pay its obligations under the MOA. CIMICO countered by filing with
Furthermore, it has also been held that after voluntarily submitting a cause and encountering an adverse decision on the merits, it is too late for the loser to question the jurisdiction or power
the Regional Trial Court (RTC) of Dumangas, Iloilo, Branch 68, a case against NFSC for sum of money and/or breach of contract.
of the court. And in Littleton vs. Burgess, 16 Wyo. 58, the Court said that it is not right for a party who has affirmed and invoked the jurisdiction of a court in a particular matter to secure an
affirmative relief, to afterwards deny that same jurisdiction to escape a penalty.
On May 10, 2002, EPCIB instituted extra-judicial foreclosure proceedings due to NFSC's failure to pay. During public auction, EPCIB was the sole bidder and was thus able to buy the entire
property and consolidate the titles in its name. EPCIB then hired Philippine Industrial Security Agency (PISA) to secure the land and the sugar mill.
The facts of this case show that from the time the Surety became a quasi-party on July 31, 1948, it could have raised the question of the lack of jurisdiction of the Court of First Instance of
Cebu to take cognizance of the present action by reason of the sum of money involved which, according to the law then in force, was within the original exclusive jurisdiction of inferior
On September 16, 2002, CIMICO filed with the RTC an Amended Complaint where it impleaded PISA and EPCIB. As a result, upon the motion of CIMICO, the RTC issued a restraining
courts. It failed to do so. Instead, at several stages of the proceedings in the court a quo as well as in the Court of Appeals, it invoked the jurisdiction of said courts to obtain affirmative relief
order, directing EPCIB and PISA to desist from taking possession over the property in dispute. Hence, CIMICO was able to continue its possession over the property.
and submitted its case for a final adjudication on the merits. It was only after an adverse decision was rendered by the Court of Appeals that it finally woke up to raise the question of
jurisdiction. Were we to sanction such conduct on its part, We would in effect be declaring as useless all the proceedings had in the present case since it was commenced on July 19, 1948
On October 3, 2002, CIMICO and petitioner Megan Sugar Corporation (MEGAN) entered into a MOA whereby MEGAN assumed CIMICO's rights, interests and obligations over the property.
and compel the judgment creditors to go up their Calvary once more. The inequity and unfairness of this is not only patent but revolting.
Thus, MEGAN started operating the sugar mill on November 18, 2002.
Coming now to the merits of the appeal: after going over the entire record, We have become persuaded that We can do nothing better than to quote in toto, with approval, the decision
Thereafter, Passi Iloilo Sugar Central, Inc. (Passi Sugar) filed with the RTC a Motion for Intervention claiming to be the vendee of EPCIB. During the hearing on the said motion, Atty. Reuben
rendered by the Court of Appeals on December 11, 1962 x x x granting plaintiffs' motion for execution against the surety x x x.
Mikhail Sabig (Atty. Sabig) appeared before the RTC as counsel for MEGAN. Several counsels objected to Atty. Sabig's appearance since MEGAN was not a party to the proceedings;
however, Atty. Sabig manifested that his statements would bind MEGAN.
CELIA S. VDA. DE HERRERA vs. EMELITA BERNARDO, EVELYN BERNARDO as Guardian of Erlyn, Crislyn and Crisanto Bernardo
G.R. No. 170251 June 1, 2011 Several motions were filed by EPCIB to hold in escrow the sugar quedans or the proceeds therefrom which was granted by the RTC. Atty. Sabig filed an Omnibus Motion for Reconsideration
and Clarification which was denied. Aggrieved by the orders issued by the RTC, MEGAN filed before the CA a petition for certiorari arguing mainly on two points; first, that the RTC erred
Topic: Lack of jurisdiction over subject matter may be raised at any stage when it determined that MEGAN was subrogated to the obligations of CIMICO and; second, that the RTC had no jurisdiction over MEGAN. CA denied such motion, ruling that since Atty.
Sabig had actively participated before the RTC, MEGAN was already estopped from assailing the RTCs jurisdiction. Aggrieved, MEGAN then filed a Motion for Reconsideration, which was,
Facts: however, denied by the CA. Hence, this petition.
Respondent heirs of Crisanto S. Bernardo, represented by Emelita Bernardo, filed a complaint before the Commission on the Settlement of Land Problems (COSLAP) against Alfredo
Herrera (Alfredo) for interference, disturbance, unlawful claim, harassment and trespassing over a portion of a parcel of land situated at Barangay Dalig, Cardona, Rizal, with an area of Issue:
7,993 square meters. The complaint was docketed as COSLAP Case No. 99-221. Whether or not MEGAN is already estopped from assailing the jurisdiction of the RTC.
Ruling:
Respondents claimed that said land was originally owned by their predecessor-in-interest, Crisanto Bernardo, and was later on acquired by Crisanto S. Bernardo. This was later on covered After a judicial examination of the records pertinent to the case at bar, this Court agrees with the finding of the CA that MEGAN is already estopped from assailing the jurisdiction of
by Tax Declaration No. CD-006-0828 under the name of the respondents. the RTC.

Petitioner, on the other hand, alleged that the portion of the subject property consisting of about 700 square meters was bought by Diosdado Herrera, Alfredo's father, from a certain The doctrine of estoppel is based upon the grounds of public policy, fair dealing, good faith and justice, and its purpose is to forbid one to speak against his own act, representations, or
Domingo Villaran. Upon the death of Diosdado Herrera, Alfredo inherited the 700-square-meter lot. commitments to the injury of one to whom they were directed and who reasonably relied thereon. The doctrine of estoppel springs from equitable principles and the equities in the case. It is
designed to aid the law in the administration of justice where without its aid injustice might result. It has been applied by this Court wherever and whenever special circumstances of a case
The COSLAP, in a Resolution dated December 6, 1999, ruled that respondents have a rightful claim over the subject property. Consequently, a motion for reconsideration and/or reopening so demand.
of the proceedings was filed by Alfredo. The COSLAP, in an Order dated August 21, 2002, denied the motion and reiterated its Order dated December 6, 1999. Aggrieved, petitioner Celia S.
Vda. de Herrera, as the surviving spouse of Alfredo, filed a petition for certiorari with the CA. The CA, Twelfth Division, in its April 28, 2005 Decision, dismissed the petition and affirmed the Based on the events and circumstances surrounding the issuance of the assailed orders, this Court rules that MEGAN is estopped from assailing both the authority of Atty. Sabig and the
resolution of the COSLAP. The CA ruled that the COSLAP has exclusive jurisdiction over the present case and, even assuming that the COSLAP has no jurisdiction over the land dispute of jurisdiction of the RTC. While it is true, as claimed by MEGAN, that Atty. Sabig said in court that he was only appearing for the hearing of Passi Sugars motion for intervention and not for the
the parties herein, petitioner is already estopped from raising the issue of jurisdiction because Alfredo failed to raise the issue of lack of jurisdiction before the COSLAP and he actively case itself, his subsequent acts, coupled with MEGANs inaction and negligence to repudiate his authority, effectively bars MEGAN from assailing the validity of the RTC proceedings under
participated in the proceedings before the said body. Petitioner filed a motion for reconsideration, which was denied by the CA in a Resolution dated October 17, 2005. Hence, petitioner the principle of estoppel.
elevated the case to this Court via Petition for Review on Certiorari under Rule 45 of the Rules of Court.
MEGAN can no longer deny the authority of Atty. Sabig as they have already clothed him with apparent authority to act in their behalf . It must be remembered that when Atty.
Petitioner averred that the COSLAP has no adjudicatory powers to settle and decide the question of ownership over the subject land. Further, the present case cannot be classified as Sabig entered his appearance, he was accompanied by Concha, MEGAN’s director and general manager. Concha himself attended several court hearings, and on December 17, 2002,
explosive in nature as the parties never resorted to violence in resolving the controversy. Petitioner submits that it is the Regional Trial Court which has jurisdiction over controversies relative even sent a letter to the RTC asking for the status of the case. A corporation may be held in estoppel from denying as against innocent third persons the authority of its officers or agents who
to ownership of the subject property. have been clothed by it with ostensible or apparent authority. Atty. Sabig may not have been armed with a board resolution, but the appearance of Concha made the parties assume
that MEGAN had knowledge of Atty. Sabig’s actions and, thus, clothed Atty. Sabig with apparent authority such that the parties were made to believe that the proper person and
Respondents, on the other hand, alleged that the COSLAP has jurisdiction over the present case. Further, respondents argued that petitioner is estopped from questioning the jurisdiction of entity to address was Atty. Sabig. Apparent authority, or what is sometimes referred to as the "holding out" theory, or doctrine of ostensible agency, imposes liability, not as the
the COSLAP by reason of laches due to Alfredo's active participation in the actual proceedings before the COSLAP. Respondents said that Alfredo's filing of the Motion for Reconsideration result of the reality of a contractual relationship, but rather because of the actions of a principal or an employer in somehow misleading the public into believing that the
and/or Reopening of the proceedings before the COSLAP is indicative of his conformity with the questioned resolution of the COSLAP. relationship or the authority exists.

Issue: One of the instances of estoppel is when the principal has clothed the agent with indicia of authority as to lead a reasonably prudent person to believe that the agent actually has such
Whether or not petitioner is estopped from questioning the jurisdiction of the COSLAP by reason of laches due to Alfredo's active participation in the actual proceedings before the COSLAP. authority. With the case of MEGAN, it had all the opportunity to repudiate the authority of Atty. Sabig since all motions, pleadings and court orders were sent to MEGAN office. However,
Ruling: MEGAN never questioned the acts of Atty. Sabig and even took time and effort to forward all the court documents to him.
In the instant case, the COSLAP has no jurisdiction over the subject matter of respondents' complaint. The present case does not fall under any of the cases enumerated under
Section 3, paragraph 2 (a) to (e) of E.O. No. 561. The dispute between the parties is not critical and explosive in nature, nor does it involve a large number of parties, nor is there a presence To this Court mind, MEGAN cannot feign knowledge of the acts of Atty. Sabig, as MEGAN was aware from the very beginning that CIMICO was involved in an on-going litigation.
or emergence of social tension or unrest.
The rule is that the active participation of the party against whom the action was brought, coupled with his failure to object to the jurisdiction of the court or administrative
It is axiomatic that the jurisdiction of a tribunal, including a quasi-judicial officer or government agency, over the nature and subject matter of a petition or complaint is determined by the body where the action is pending, is tantamount to an invocation of that jurisdiction and a willingness to abide by the resolution of the case and will bar said party from later on
material allegations therein and the character of the relief prayed for, irrespective of whether the petitioner or complainant is entitled to any or all such reliefs. impugning the court or body’s jurisdiction. Based on the preceding discussion, this Court holds that MEGAN's challenge to Atty. Sabig’s authority and the RTCs jurisdiction was a
mere afterthought after having received an unfavorable decision from the RTC . Certainly, it would be unjust and inequitable to the other parties if this Court were to grant such a
Respondents' cause of action before the COSLAP pertains to their claim of ownership over the subject property, which is an action involving title to or possession of real belated jurisdictional challenge.
property, or any interest therein, the jurisdiction of which is vested with the Regional Trial Courts or the Municipal Trial Courts depending on the assessed value of the subject
property.
Manchester Development v. Court of Appeals
G.R. No. 75919, May 7, 1987
Since the COSLAP has no jurisdiction over the action, all the proceedings therein, including the decision rendered, are null and void . A judgment issued by a quasi-judicial body
TOPIC: Payment of filing/docket fees – jurisdictional
without jurisdiction is void. It cannot be the source of any right or create any obligation. All acts performed pursuant to it and all claims emanating from it have no legal effect. Having no legal
effect, the situation is the same as it would be as if there was no judgment at all. It leaves the parties in the position they were before the proceedings.
NATURE OF THE CASE: Motion for Reconsideration
Respondents’ allegation that petitioner is estopped from questioning the jurisdiction of the COSLAP by reason of laches does not hold water. Petitioner is not estopped from
FACTS:
raising the jurisdictional issue, because it may be raised at any stage of the proceedings, even on appeal, and is not lost by waiver or by estoppel . The fact that a person attempts
This was originally a case of an action for torts and damages and specific performance with a prayer for temporary restraining order. The damages were not specifically stated in the prayer
to invoke unauthorized jurisdiction of a court does not estop him from thereafter challenging its jurisdiction over the subject matter, since such jurisdiction must arise by law and not by mere
but the body of the complaint assessed a P78.75M damages suffered by the petitioner. The amount of docket fees paid was only P410.00. The petitioner then amended the complaint and
consent of the parties.
reduced the damages to P10M only.
In Regalado v. Go, the Court held that laches should be clearly present for the Sibonghanoy doctrine to apply, thus:
Petitioners in support of their contention that the �ling fee must be assessed on the basis of the amended complaint cite the case of Magaspi vs. Ramolete. They contend that the Court of Petitioner is a corporation engaged in the manufacture and distribution of all Nestle products nationwide. Respondent is a corporation engaged in trading, marketing, selling and distributing
Appeals erred in ruling that the filing fee should be levied by considering the amount of damages sought in the original complaint. food items to restaurants and food service outlets. On December 23, 1998, Nestlé and FY Sons entered into a distributorship agreement (agreement) whereby petitioner would supply its
products for respondent to distribute to its food service outlets. FY Sons executed a deed of assignment in favor of Nestlé, assigning a time deposit of Calixto Laureano in the amount of
P500k to secure FY Sons’ credit purchases from Nestlé. Laureano also executed a special power of attorney, authorizing FY Sons to use the time deposit as collateral.
Differences: The areas covered by the agreement were Baguio, Dagupan, Angeles, Bulacan, Pampanga, Urdaneta, La Union, Tarlac and Olongapo. At the end of 1989, the agreement expired and the
parties executed a renewal agreement on January 22, 1990. A supplemental agreement was executed on June 27, 1990, to take effect on July 1, 1990.
MAGASPI CASE MANCHESTER CASE (present case)
On July 2, 1990, Nestlé fined FY Sons P20,000 for allegedly selling 50 Krem-Top liquid coffee creamer cases to Lu Hing Market (Tarlac), as this was purportedly proscribed by the
It was an action for recovery of ownership and possession of a parcel of land with damages An action for torts and damages and specific performance with prayer for temporary agreement. FY Sons paid the fine. When the same product was sold to Augustus Bakery and Grocery, another fine (P40,000) was imposed for the same reason.
restraining order, etc.
On October 19, 1990, FY Sons (through counsel) wrote Nestlé to demand damages and to complain about the latter’s breaches of their agreement and the various acts of bad faith. On 5
The prayer in the complaint seeks not only the annulment of title of the defendant to the The prayer is for the issuance of a writ of preliminary prohibitory injunction during the Nov, Nestlé sent FY Sons a demand letter and notice of termination, alleging that FY Sons had outstanding accounts of P995k~. When the alleged accounts were not settled, Nestlé applied
property, the declaration of ownership and delivery of possession thereof to plaintiffs but pendency of the action against the defendants' announced forfeiture of the sum of P3 Million the P500k time deposit as partial payment.
also asks for the payment of actual, moral, exemplary damages and attorney's fees arising paid by the plaintiffs for the property in question, to attach such property of defendants that
therefrom in the amounts specified therein may be sufficient to satisfy any judgment that may be rendered, and after hearing, to order FY Sons filed a complaint for damages against Nestlé, alleging bad faith.
defendants to execute a contract of purchase and sale of the subject property and annul
defendants' illegal forfeiture of the money of plaintiff, ordering defendants jointly and  Nestlé made representations of rendering support, so FY Sons was lured into executing an agreement.
severally to pay plaintiff actual, compensatory and exemplary damages as well as 25% of  FY Sons invested money, time, and effort, only for Nestlé to breach the agreement by committing various acts of bad faith.
said amounts as may be proved during the trial as attorney's fees and declaring the tender
 By the agreement’s termination, Nestlé would obtain market gains made by FY Sons at the latter’s efforts and expenses.
of payment of the purchase price of plaintiff valid and producing the effect of payment and to
make the injunction permanent. The amount of damages sought is not specified in the
prayer although the body of the complaint alleges the total amount of over P78 Million as
Respondent’s Prayer:
damages suffered by plaintiff.
Respondent sought actual damages of P1,000,000, moral damages of P200,000, exemplary damages of P100,000, attorney's fees of P100,000, plus the return of the P500,000 time deposit
and costs of suit.
Upon the �ling of the complaint there was an honest difference of opinion as to the nature There can be no such honest difference of opinion. As may be gleaned from the allegations
of the action in the Magaspi case. The complaint was considered as primarily an action for of the complaint as well as the designation thereof, it is both an action Petitioner’s Answer:
recovery of ownership and possession of a parcel of land. The damages stated for damages and specific performance. The docket fee paid upon filing of complaint in the
Petitioner interposed a counterclaim for P495,319.81 representing the balance of respondent's overdue accounts, with interest of 2% per month from the date of default until fully paid, moral
were treated as merely ancillary to the main cause of action. Thus, the docket fee of only amount only of P410.00 by considering the action to be merely one for specific performance damages of P100,000, exemplary damages of P200,000, attorney's fees of P120,000 and costs of suit.
P60.00 and P10.00 for the sheriff's fee were paid. where the amount involved is not capable of pecuniary estimation is obviously erroneous.
Although the total amount of damages sought is not stated in the prayer of the complaint yet RTC:
it is spelled out in the body of the complaint totaling in the amount of P78,750,000.00 which
RTC ruled in favor of the respondent and ordered the petitioner to pay P1,000,000 as actual damages, P100,000 as exemplary damages, P100,000 as attorney’s fees and P53,214.26, the
should be the basis of assessment of the filing fee. amount entitled from the petitioner.
The trial court ordered the plaintiffs to pay the amount of P3,104.00 as filing fee covering Plaintiff through another counsel with leave of court filed an amended complaint on
the damages alleged in the original complaint as it did not consider the damages to be September 12, 1985 for the inclusion of Philips Wire and Cable Corporation as co-plaintiff  Nestlé failed to provide support; it unjustifiably refused to deliver stocks; the imposition of the P20k fine was void for having no basis; Nestlé terminated the agreement
merely ancillary or incidental to the action for recovery of ownership and possession of real and by eliminating any mention of the amount of damages in the body of the complaint. The
without sufficient basis and in bad faith.
property. An amended complaint was filed by plaintiff with leave of court to include the prayer in the original complaint was maintained. After this Court issued an order on October
 Nestlé failed to prove FY Sons’ alleged outstanding obligation.
government of the Republic as defendant and reducing the amount of damages, and 15, 1985 ordering the re-assessment of the docket fee in the present case and other cases
o Nestle’s Statement of Account showing the alleged unpaid balance is undated, and it does not show receipt thereof by FY Sons, and when, if such indeed
attorney's fees prayed for to P100,000.00. Said amended complaint was also admitted. that were investigated, on November 12, 1985 the trial court directed plaintiffs to rectify the
amended complaint by stating the amounts which they are asking for. It was only then that was received.
plaintiffs specified the amount of damages in the body of the complaint in the reduced o There are no supporting documents to sustain such unpaid accounts.
amount of P10,000,000.00. Still no amount of damages were specified in the prayer. Said
amended complaint was admitted.
CA:
The action was considered not only one for recovery of ownership but also for damages, so No such honest difference of opinion was possible as the allegations of the complaint, the Rendered a decision affirming the RTC's decision with modification: (1) the actual damages is INCREASED from P1,000,000.00 to P1,500,000.00; and (2) the amount of P53,214.26
that the filing fee for the damages should be the basis of assessment. Although the designation and the prayer show clearly that it is an action for damages and specific payable by the petitioner to the is DELETED.
payment of the docketing fee of P60.00 was found to be insufficient, nevertheless, it was performance. The docketing fee should be assessed by considering the amount of damages
held that since the payment was the result of an "honest difference of opinion as to the as alleged in the original complaint. ISSUES:
correct amount to be paid as docket fee" the court "had acquired jurisdiction over the case
and the proceedings thereafter had were proper and regular." Hence, as the amended (1) Whether or not petitioner’s witness (Rayos) is competent to testify.
complaint superseded the original complaint, the allegations of damages in the amended (2) Whether or not the CA erred in awarding damages and refund of the P500,000 time deposit.
complaint should be the basis of the computation of the filing fee.

EVIDENCE FOR FY SONS EVIDENCE FOR NESTLÉ


CA Ruling:
The basis of assessment of the docket fee should be the amount of damages sought in the original complaint and not in the amended complaint. Testimony of Cristina Rayos
Testimony of Florentino Yue, Jr.
ISSUE:  Prepared statement of account on the basis of the invoices and delivery
Whether or not the court acquired jurisdiction over the case when the correct and proper docket fees has not been paid.  Director and officer
orders corresponding to the alleged overdue accounts of FY Sons

SC Ruling:
A case is deemed filed only upon payment of the docket fee regardless of the actual date of filing in court." Thus, in the present case the trial court did not acquire jurisdiction over the case
by the payment of only P410.00 as docket fee. Neither can the amendment of the complaint thereby vest jurisdiction upon the Court. For all legal purposes there is no such original Re: Cristina Rayos
complaint that was duly filed which could be amended. Consequently, the order admitting the amended complaint and all subsequent proceedings and actions taken by the trial court are null
and void.  She admitted that the invoices corresponding to the alleged overdue accounts are not signed, because that there were delivery orders covering the transactions. However,
she did not identify the signatures on the delivery orders as the persons who received the goods for Nestlé.
The Court cannot close this case without making the observation that it frowns at the practice of counsel who filed the original complaint in this case of omitting any specification of the
 She could not have identified the same, for she was not involved in the delivery, as she is only in charge of the records and documents on all accounts receivables as part of
amount of damages in the prayer although the amount of over P78 million is alleged in the body of the complaint. This is clearly intended for no other purpose than to evade the payment of
the correct filing fees if not to mislead the docket clerk in the assessment of the filing fee. This fraudulent practice was compounded when, even as this Court had taken cognizance of the her duties as Credit and Collection Manager.
anomaly and ordered an investigation, petitioner through another counsel filed an amended complaint, deleting all mention of the amount of damages being asked for in the body of the
complaint. It was only when in obedience to the order of this Court of October 18, 1985, the trial court directed that the amount of damages be speci fied in the amended complaint, that
petitioners' counsel wrote the damages sought in the much reduced amount of P10,000,000.00 in the body of the complaint but not in the prayer thereof. The design to avoid payment of the Re: FY Sons’ witness Yue, Jr.
required docket fee is obvious.
 Nestlé: Yue admitted in open court that FY Sons had an unpaid obligation to Nestlé of around P900k.
To put a stop to this irregularity, henceforth all complaints, petitions, answers and other similar pleadings should specify the amount of damages being prayed for not only in the body of the  FY Sons: This statement was merely in response to the judge’s question on what ground Nestlé supposedly terminated the agreement. Yue was not being asked, nor was
pleading but also in the prayer, and said damages shall be considered in the assessment of the filing fees in any case. Any pleading that fails to comply with this requirement shall not be he addressing, the truth of such ground.
accepted nor admitted, or shall otherwise be expunged from the record.  SC: Nestlé is WRONG, as it took Yue’s statement out of context; cannot be considered a judicial admission.

The Court acquires jurisdiction over any case only upon the payment of the prescribed docket fee. An amendment of the complaint or similar pleading will not thereby vest jurisdiction in the
Court, much less the payment of the docket fee based on the amounts sought in the amended pleading. The ruling in the Magaspi case in so far as it is inconsistent with this pronouncement SC RULING:
is overturned and reversed. Rayos testified on a statement of account she prepared on the basis of invoices and delivery orders which she, however, knew nothing about. She had no personal knowledge of the facts on
which the accounts were based since, admittedly, she was not involved in the delivery of goods and was merely in charge of the records and documents of all accounts receivable as part of
WHEREFORE, the motion for reconsideration is denied for lack of merit. her duties as credit and collection manager. She thus knew nothing of the truth or falsity of the facts stated in the invoices and delivery orders, i.e., whether such deliveries were in fact made
in the amounts and on the dates stated, or whether they were actually received by respondent. She was not even the credit and collection manager during the period the agreement was in
effect. This can only mean that she merely obtained these documents from another without any personal knowledge of their contents. Thus, she was incompetent to testify on whether or not
Nestle Phils. v. FY Sons, G.R. No. 150780, May 5, 2006
the invoices and delivery orders turned over to her correctly reflected the details of the deliveries made.
TOPIC: Payment of filing/docket fees – jurisdictional
The CA declared that petitioner was not able to prove that respondent had unpaid accounts, thus debunking the claim of a valid termination. The CA also held petitioner guilty of various acts
NATURE OF THE CASE: petition for review on certiorari under Rule 45 of the Rules of Court assailing the decision of the CA which in turn affirmed with modification the decision of the
which violated the provisions of the agreement. Consequently, for petitioner's breach of the agreement, the CA awarded actual damages to
RTC, as well as the CA's resolution which denied petitioner's motion for reconsideration.
respondent in the amount of P1,000,000. Petitioner, other than claiming that it validly terminated the agreement, did not challenge the findings of the CA that it committed various violations
of the agreement. Hence, there was legal basis for the grant of actual damages.
FACTS:
Petitioner’s contention: the expiration of said period. Consequently, this Court held that the date of such payment must be deemed to be the real date of filing of aforesaid petition and not the date when it was
mailed.
 documentary evidence presented by respondent to prove actual damages in the amount of P4,246,015.60 should not have been considered because respondent's
complaint only prayed for an award of P1,000,000. Again, in Garica vs, Vasquez, this Court reiterated the rule that the docket fee must be paid before a court will act on a petition or complaint. However, we also held that said rule is not
 the court acquires jurisdiction over the claim only upon payment of the prescribed docket fee. applicable when petitioner seeks the probate of several wills of the same decedent as he is not required to file a separate action for each will but instead he may have other wills probated in
the same special proceeding then pending before the same court.

Indeed, a court acquires jurisdiction over the claim of damages upon payment of the correct docket fees. In this case, it is not disputed that respondent paid docket fees based on the Then in Magaspi, this Court reiterated the ruling in Malimit and Lee that a case is deemed filed only upon payment of the correct docket fee regardless of the actual date of its filing in court.
amounts prayed for in its complaint. Respondent adduced evidence to prove its losses. It was proper for the CA and the RTC to consider this evidence and award the sum of P1,000,000. In the said case, there was an honest difference of opinion as to the correct amount to be paid as docket fee because the action appears to be one for the recovery of property the docket
Had the courts below awarded a sum more than P1,000,000, which was the amount prayed for, an additional filing fee would have been assessed and imposed as a lien on the judgment. fee of P60.00 was correct; and that as the action is also one, for damages, the SC upheld the assessment of the additional docket fee based on the damages alleged in the amended
However, the courts limited their award to the amount prayed for. complaint as against the assessment of the trial court which was based on the damages alleged in the original complaint.

Given that petitioner was not able to prove that respondent had unpaid accounts in the amount of P995,319.81, the seizure of the P500,000 time deposit was improper. As a result, the However, SC overturned Magaspi in Manchester. Manchester involves an action for torts and damages and specific performance with a prayer for the issuance of a temporary restraining
refund of this amount with interest is also called for. order, etc.. The amount of damages sought is not specified in the prayer although the body of the complaint alleges the total amount of over P78 Millon allegedly suffered by plaintiff.
Applying the principle in Magaspi that "the case is deemed filed only upon payment of the docket fee regardless of the actual date of filing in court," this Court held that the trial court did not
Petitioner's counterclaims are necessarily without merit. It failed to prove the alleged outstanding accounts of respondent. Accordingly, it is not entitled to the supposed unpaid balance of acquire jurisdiction over the case by payment of only P410.00 for the docket fee. Neither can the amendment of the complaint thereby vest jurisdiction upon the Court. For all legal purposes
P495,319.81 with interest. Petitioner, being at fault and in bad faith, and there being no proof that respondent was guilty of any wrongdoing, cannot claim moral and exemplary damages and there was no such original complaint duly filed which could be amended. Consequently, the order admitting the amended complaint and all subsequent proceedings and actions taken by the
attorney's fees from respondent. trial court were declared null and void.

WHEREFORE, the petition is hereby DENIED for lack of merit. The decision of the Court of Appeals dated January 11, 2001 and resolution dated November 14, 2001 in CA-G.R. CV No. The facts and circumstances of the present case are similar to Manchester. The principle in Manchester could very well be applied in the present case. The pattern and the intent to defraud
57299 are hereby AFFIRMED. the government of the docket fee due is obvious not only in the filing of the original complaint but also in the filing of the second amended complaint.

However, in Manchester, petitioner did not pay any additional docket fee until the case was decided by the SC on May 7, 1987. Thus, in Manchester, due to the fraud committed on the
Sun Insurance v. Asuncion government, this Court held that the court a quo did not acquire jurisdiction over the case and that the amended complaint could not have been admitted inasmuch as the original complaint
G.R. Nos. 79937-38, February 13, 1989 was null and void.
TOPIC: Payment of filing/docket fees – jurisdictional
In the present case, a more liberal interpretation of the rules is called for considering that, unlike Manchester, private respondent demonstrated his willingness to abide by the rules by paying
FACTS: the additional docket fees as required.
On February 28, 1984, petitioner Sun Insurance filed a complaint with the RTC Makati for the consignation of a premium refund on a fire insurance policy with a prayer for the judicial
declaration of its nullity against private respondent Manuel Uy Po Tiong. Private respondent as declared in default for failure to file the required answer within the reglementary period. Nevertheless, petitioners contend that the docket fee that was paid is still insufficient considering the total amount of the claim. This is a matter which the clerk of court of the lower court
and/or his duly authorized docket clerk or clerk in-charge should determine and, thereafter, if any amount is found due, he must require the private respondent to pay the same.
On the other hand, on March 28, 1984, private respondent filed a complaint in the RTC QC for the refund of premiums and the issuance of a writ of preliminary attachment, initially against
petitioner Sun Insurance, and thereafter including E.B. Philipps and D.J. Warby as additional defendants. The complaint docketed as Civil Case Q-41177 sought, among others, the payment Thus, the Court rules as follows:
of damages. Although the prayer in the complaint did not quantify the amount of damages sought said amount may be inferred from the body of the complaint to be about P50 Million. 1. It is not simply the filing of the complaint or appropriate initiatory pleading, but the payment of the prescribed docket fee, that vests a trial court with jurisdiction over the subject matter or
nature of the action. Where the filing of the initiatory pleading is not accompanied by payment of the docket fee, the court may allow payment of the fee within a reasonable time but in no
Only the amount of P210.00 was paid by private respondent as docket fee which prompted petitioners' counsel to raise his objection. Said objection was disregarded by respondent Judge case beyond the applicable prescriptive or reglementary period.
Jose P. Castro who was then presiding over said case. Upon the order of this Court, the records of said case together with 22 other cases assigned to different branches of the RTC QC
which were under investigation for under-assessment of docket fees were transmitted to the SC. The SC ordered that the cases be re-raffled, the judges in said cases to reassess the docket 2. The same rule applies to permissive counterclaims, third party claims and similar pleadings, which shall not be considered filed until and unless the filing fee prescribed therefor is paid.
fees and that in case of deficiency, to order its payment. The Resolution also requires all clerks of court to issue certificates of re-assessment of docket fees. All litigants were likewise The court may also allow payment of said fee within a reasonable time but also in no case beyond its applicable prescriptive or reglementary period.
required to specify in their pleadings the amount sought to be recovered in their complaints.
3. Where the trial court acquires jurisdiction over a claim by the filing of the appropriate pleading and payment of the prescribed filing fee but, subsequently, the judgment awards a claim not
Thus, Judge Solano, to whose sala Civil Case Q-41177 was temporarily assigned, instructed the Clerk of Court to issue a certificate of assessment of the docket fee paid by private specified in the pleading, or if specified the same has been left for determination by the court, the additional filing fee therefor shall constitute a lien on the judgment. It shall be the
respondent and, in case of deficiency, to include the same in said certificate. On January 7, 1984, to forestall a default, a cautionary answer was filed by petitioners. On August 30,1984, an responsibility of the Clerk of Court or his duly authorized deputy to enforce said lien and assess and collect the additional fee.
amended complaint was filed by private respondent including the two additional defendants aforestated.
WHEREFORE, the petition is DISMISSED for lack of merit. The Clerk of Court of the court a quo is hereby instructed to reassess and determine the additional filing fee that should be paid
Respondent Judge Asuncion, to whom Civil Case No. Q41177 was thereafter assigned, after his assumption into office on January 16, 1986, issued a Supplemental Order requiring the by private respondent considering the total amount of the claim sought in the original complaint and the supplemental complaint as may be gleaned from the allegations and the prayer
parties in the case to comment on the Clerk of Court's letter-report signifying her difficulty in complying with the Resolution of the SC since the pleadings filed by private respondent did not thereof and to require private respondent to pay the deficiency, if any, without pronouncement as to costs.
indicate the exact amount sought to be recovered. Private respondent filed a "Compliance" and a "Re-Amended Complaint" stating therein a claim of "not less than P10 Million as actual
compensatory damages" in the prayer. In the body of the said second amended complaint however, private respondent alleges actual and compensatory damages and attorney's fees in the
total amount of about P44,601,623.70.
Phil. First Insurance v. Paramount Gen. Insurance
On January 24, 1986, Judge Asuncion issued another Order admitting the second amended complaint and stating therein that the same constituted proper compliance with the SC G.R. No. 165147, July 9, 2008
Resolution and that a copy thereof should be furnished the Clerk of Court for the reassessment of the docket fees. The reassessment by the Clerk of Court based on private respondent's TOPIC: Payment of filing/docket fees – jurisdictional
claim of "not less than P10 M as actual and compensatory damages" amounted to P39,786.00 as docket fee. This was subsequently paid by private respondent.
FACTS:
Petitioners then filed a petition for certiorari with the CA questioning the said order of Judge Asuncion. On November 8, 2000, the delivery van of Pyramid bearing license plate number PHL-545 which was loaded with goods belonging to California Manufacturing Corporation (CMC) valued at
On April 24, 1986, private respondent filed a supplemental complaint alleging an additional claim of P20 M as damages so the total claim amounts to about P65 Million. Seven months after P907,149.07 left the CMC Bicutan Warehouse but the van, together with the goods, failed to reach its destination and its driver and helper were nowhere to be found, to its damage and
filing the supplemental complaint, the private respondent paid the additional docket fee of P80,396.00. prejudice; that it filed a criminal complaint against the driver and the helper for qualified theft, and a claim with herein petitioners as co-insurers of the lost goods but, in violation of
petitioners’ undertaking under the insurance policies, they refused without just and valid reasons to compensate it for the loss; and that as a direct consequence of petitioners’ failure,
On August 13, 1987, the CA denied the petition insofar as it seeks annulment of the order, and petitioner’s motion to dismiss the amended complaint. Hence, the instant petition. despite repeated demands, to comply with their respective undertakings under the Insurance Policies by compensating for the value of the lost goods, it suffered damages and was
constrained to engage the services of counsel to enforce and protect its right to recover compensation under said policies, for which services it obligated itself to pay the sum equivalent to
During the pendency of this petition and in conformity with the said judgment of respondent court, private respondent paid the additional docket fee of P62,432.90 on April 28, 1988. twenty-five (25%) of any amount recovered as and for attorney’s fees and legal expenses. Pyramid was assessed P610 docket fee, apparently on the basis of the amount of P50,000
specified in the prayer representing attorney’s fees, which it duly paid.
ISSUE:
Whether or not the RTC acquire jurisdiction over the case even if there was nonpayment of the correct and proper docket fee. Petitioners filed a Motion to Dismiss on the ground of, inter alia, lack of jurisdiction, Pyramid not having paid the docket fees in full arguing that in the prayer in the Complaint, plaintiff
deliberately omitted to specify what these damages are in order to evade the payment of the docket fees. To the Motion to Dismiss Pyramid filed its Opposition, alleging that if there was a
Petitioners’ contention: mistake in the assessment of the docket fees, the trial court was not precluded from acquiring jurisdiction over the complaint as “it has the authority to direct the mistaken party to complete
Considering that the total amount sought to be recovered in the amended and supplemental complaint is P64,601,623.70 the docket fee that should be paid by private respondent is the docket fees in the course of the proceedings.”
P257,810.49, more or less. Not having paid the same, petitioners contend that the complaint should be dismissed and all incidents arising therefrom should be annulled. As basis, petitioners
cite Manchester Development Corporation vs. CA: RTC Makati:
The Court acquires jurisdiction over any case only upon the payment of the prescribed docket fee. An amendment of the complaint or similar pleading will not thereby vest Dismissed, saying that the case being for specific performance, it is not dismissible on that ground but unless proper docket fees are paid, the RTC can only grant what was prayed for in the
jurisdiction in the Court, much less the payment of the docket fee based on the amounts sought in the amended pleading. The ruling in the Magaspi Case in so far as it is Complaint.
inconsistent with this pronouncement is overturned and reversed.
CA:
Respondent’s contention: Partially granted, ordering Pyramid to pay the correct docket fees on the basis of the losses alleged in the body of the complaint, plus the attorney’s fees mentioned in the prayer, within a
Manchester cannot apply retroactively for at the time said civil case was filed in court there was no such ruling as yet. Magaspi v. Ramolete applies wherein it was held that the trial court reasonable time which should not go beyond the applicable prescriptive or reglementary period.
acquired jurisdiction over the case even if the docket fee paid was insufficient.
Petitioners' Argument:
RULING: They invoke the doctrine in Manchester Development Corporation v. Court of Appeals that a pleading which does not specify in the prayer the amount sought shall not be admitted or shall
YES, the court acquired jurisdiction over the case. otherwise be expunged, and that the court acquires jurisdiction only upon the payment of the prescribed docket fee.

Nevertheless, the contention that Manchester cannot retroactively apply is untenable. Statutes regulating the procedure of the courts will be construed as applicable to actions pending and Respondent's Argument:
undetermined at the time of their passage. Procedural laws are retrospective in that sense and to that extent. They invoke the application of Sun Insurance Office, Ltd. (SIOL) v. Asuncion and subsequent rulings relaxing the Manchester ruling by allowing payment of the docket fee within a
reasonable time, in no case beyond the applicable prescriptive or reglementary period, where the filing of the initiatory pleading is not accompanied by the payment of the prescribed docket
In Lazaro vs. Endencia and Andres, this Court held that the payment of the full amount of the docket fee is an indispensable step for the perfection of an appeal. Plaintiff-appellant deposited fee.
the deficiency in the docket fee outside the 15-day reglementary period for appeal. Thus, the CFI (as appellate court) did not acquire jurisdiction as the appeal was not perfected.
ISSUES:
In Lee vs. Republic, the petitioner filed a verified declaration of intention to become a Filipino citizen by sending it through registered mail to the Office of the Solicitor General in 1953 but the
required filing fee was paid only in 1956. Citing Lazaro, this Court concluded that the filing of petitioner's declaration of intention on October 23, 1953 produced no legal effect until the 1. Whether respondent, Pyramid Logistics and Trucking Corporation (Pyramid), which filed a complaint, denominated as one for specific performance and damages, against
required filing fee was paid on May 23, 1956. petitioners Philippine First Insurance Company, Inc. (Philippine First) and Paramount General Insurance Corporation (Paramount) before the Regional Trial Court (RTC) of
Makati, paid the correct docket fee
In Malimit vs. Degamo, the same principles enunciated in Lazaro and Lee were applied. It was an original petition for quo warranto contesting the right to office of proclaimed candidates 2. If in the negative, whether the complaint should be dismissed or Pyramid can still be ordered to pay the fee.
which was mailed, addressed to the clerk of the CFI, within the one-week period after the proclamation as provided therefor by law. However, the required docket fees were paid only after
SC RULING: Whether or not the CA erred in upholding respondent RTC's admission of R-II Builders' Second Amended Complaint despite non-payment of the docket fees for its original complaint and
Yes, Pyramid filed the correct docket fee. Amended and Supplemental Complaint.

In the case of Tacay vs. RTC of Tagum, Davao del Norte, the SC clarified the effect of the Sun Insurance ruling on the Manchester ruling as follows: SC RULING:
Having admitted that its original complaint partook the nature of a real action and having been directed to pay the correct docket fees for its Amended and Supplemental Complaint, R-II
The requirement in Circular No. 7 that complaints, petitions, answers, and similar pleadings should specify the amount of damages being prayed for not only in the body of the pleading but Builders is, furthermore, clearly chargeable with knowledge of the insufficiency of the docket fees it paid. Unmistakably manifesting its intent to evade payment of the correct docket fees,
also in the prayer, has not been altered. What has been revised is the rule that subsequent amendment of the complaint or similar pleading will not thereby vest jurisdiction in the Court, moreover, R-II Builders withdrew its Amended and Supplemental Complaint after its admission and, in lieu thereof, filed its' Second Amended Complaint on the ground that said earlier
much less the payment of the docket fee based on the amount sought in the amended pleading, the trial court now being authorized to allow payment of the fee within a reasonable time but pleading cannot be considered admitted in view of its non-payment of the docket and other fees it was directed to pay. In so doing, however, R-II Builders conveniently overlooked the fact
in no case beyond the applicable prescriptive period or reglementary period. Moreover, a new rule has been added, governing the awards of claims not specified in the pleading – i.e., that the very same argument could very well apply to its original complaint for which — given its admitted nature as a real action — the correct docket fees have also yet to be paid.
damages arising after the filing of the complaint or similar pleading – as to which the additional filing fee therefore shall constitute a lien on the judgment.
For nonpayment of the correct docket fees which, for real actions, should be computed on the basis of the assessed value of the property, or if there is none, the estimated value thereof as
In the case at bar, Pyramid failed to specify in its prayer the amount of claims/damages it was seeking both in the original and amended complaint. It reasoned out that it was not aware of alleged by the claimant, respondent RTC should have denied admission of R-II Builders' Second Amended Complaint and ordered the dismissal of the case. Although a catena of decisions
the extent of the liability of the insurance companies under their respective policies. It left the matter of liability to the trial court’s determination. rendered by this Court eschewed the application of the doctrine laid down in the Manchester case, said decisions had been consistently premised on the willingness of the party to pay the
correct docket fees and/or absence of intention to evade payment of the correct docket fees. This cannot be said of R-II Builders which not only failed to pay the correct docket fees for its
Even assuming that the amounts are yet to be determined, the rule in Manchester, as modified by Sun Insurance, still applies. In the case of Ayala Corporation vs. Madayag, the SC original complaint and Amended and Supplemental Complaint but also clearly evaded payment of the same by filing its Second Amended Complaint.
pronounced the following: While it is true that the determination of certain damages x x x is left to the sound discretion of the court, it is the duty of the parties claiming such damages to
specify the amount sought on the basis of which the court may make a proper determination, and for the proper assessment of the appropriate docket fees. The exception contemplated as By itself, the propriety of admitting R-II Builders' Second Amended Complaint is also cast in dubious light when viewed through the prism of the general prohibition against amendments
to claims not specified or to claims although specified are left for determination of the court is limited only to any damages that may arise after the filing of the complaint or similar pleading intended to confer jurisdiction where none has been acquired yet. Although the policy in this jurisdiction is to the effect that amendments to pleadings are favored and liberally allowed in the
for then it will not be possible for the claimant to specify nor speculate as to the amount thereof. interest of justice, amendment is not allowed where the court has no jurisdiction over the original complaint and the purpose of the amendment is to confer jurisdiction upon the court. Hence,
with jurisdiction over the case yet to properly attach, HGC correctly fault the CA for upholding respondent RTC's admission of R-II Builders' Second Amended Complaint despite non-
payment of the docket fees for its original complaint and Amended and Supplemental Complaint as well as the clear intent to evade payment thereof.
Home Guaranty Corp. v. R-11 Home Guaranty Corp. v. R-11 Builders
G.R. No. 192649, March 9, 2011
TOPIC: Payment of filing/docket fees – jurisdictional Unicapital v. Consing
G.R. Nos. 175277 & 175285, September 11, 2013
FACTS: TOPIC: Payment of filing/docket fees – jurisdictional
On 19 March 1993, a Joint Venture Agreement (JVA) was entered into between respondents National Housing Authority (NHA) and R-II Builders, Inc. (R-II Builders) for the implementation of
the Smokey Mountain Development and Reclamation Project (SMDRP). FACTS:
Petitioner Consing obtained for himself and his mother an 18M loan from Unicapital. It was secured by Promissory notes and a Real estate mortgage constituted on a parcel of land in Cavite
On 26 September 1994, NHA and R-II Builders, alongside petitioner Housing Guaranty Corporation (HGC) as guarantor and the Philippine National Bank (PNB) as trustee, entered into an registered under the name of the mother of petItioner, Cecilia dela Cruz. In pursuance with the option to purchase the mortgaged property, Unicapital agreed to purchase one half of the
Asset Pool Formation Trust Agreement which provided the mechanics for the implementation of the project. To back the project, an Asset Pool was created. property. The payment was set-off by the loan while the other half was purchased by Plus Builders Inc. (PBI), a joint venture partner of Unicapital. Before both companies could develop the
land, they discovered that the property was in the names of Po Willie Yu and Juanito Tan Teng and that dela Cruz' title was a mere forgery. Thus, PBI and Unicapital send separate demand
On the same date, the parties likewise executed a Contract of Guaranty whereby HGC, upon the call made by PNB and conditions therein specified, undertook to redeem the regular letters to dela Cruz and Consing, seeking for the return of the purchase price they had paid for the subject property.
Smokey Mountain Project Participation Certificate (SMPPCs) upon maturity and to pay the simple interest thereon to the extent of 8.5% per annum.
On May 3, 1999, Consing filed a complaint, denominated as a Complex Action for Declaratory Relief and later amended to Complex Action for Injunctive Relief before the RTC-Pasig City
Subsequent to R-II Builders' infusion of P300 Million into the project, the issuance of the SMPPCs and the termination of PNB’s services on 29 January 2001, NHA, R-II Builders and HGC docketed as SCA No. 1759. In his complaint, he claimed that the incessant demands/recovery efforts made upon him constituted harassment and oppression which severely affected his
agreed on the institution of Planters Development Bank (PDB) as trustee on 29 January 2001. By 24 October 2002, however, all the Regular SMPPCs issued had reached maturity and, personal and professional life. He also averred that he was coerced to commit a violation of Batas Pambansa Blg. 22 as Unicapital and PBI, over threats of filing a case against him, kept on
unredeemed, already amounted to an aggregate face value of P2.513 Billion. The lack of liquid assets with which to effect redemption of the regular SMPPCs prompted PDB to make a call forcing him to issue a post-dated check in the amount sought to be recovered, notwithstanding their knowledge that he had no funds for the same. He further alleged that Unicapital and URI
on HGC’s guaranty and to execute in the latter’s favor a Deed of Assignment and Conveyance (DAC) of the entire Asset Pool. required him to sign blank deeds of sale and transfers without cancelling the old ones in violation of the laws on land registration and real estate development. Likewise, he added that
Unicapital and PBI’s representatives were speaking of him in a manner that was inappropriate and libelous. For their part, Unicapital, et al. filed separate Motions to Dismiss Consing's
On 1 September 2005, R-II Builders filed the complaint against HGC and NHA before Branch 24 of the Manila Regional Trial Court, a Special Commercial Court (SCC). Contending that complaint on the ground of failure to state a cause of action. They posited that the RTC-Pasig City did not acquire jurisdiction over the case given that Consing failed to pay the proper
HGC’s failure to redeem the outstanding regular SMPPCs despite obtaining possession of the Asset Pool ballooned the stipulated interests and materially prejudiced its stake on the residual amount of docket fees and that it had no jurisdiction over their supposed violations of the Corporation Code and Revised Securities Act. The RTC-Pasig City issued a Resolution denying the
values of the Asset Pool, R-II Builders alleged, among other matters, that the DAC should be rescinded since PDB exceeded its authority in executing the same prior to HGC’s redemption above motions to dismiss, holding that Consing, Jr.’s complaint sufficiently stated a cause of action for tort and damages pursuant to Article 19 of the Civil Code. Unicapital et al. moved for
and reconsideration which was, however, denied. Aggrieved, they elevated the denial of their motions to dismiss before the CA via a petition for certiorari and prohibition. The CA rendered a
payment of the guaranteed SMPPCs. Decision holding that no grave abuse of discretion was committed by the RTC-Pasig City in refusing to dismiss Consing's complaint.

Having filed its answer to the complaint, in the meantime, HGC went on to move for the conduct of a preliminary hearing on its affirmative defenses which included such grounds as lack of On the other hand, on August 4, 1999, Unicapital filed a complaint for sum of money with damages against Consing and Dela Cruz before the RTC-Makati City, docketed as Civil Case No.
jurisdiction. On 2 August 2007, R-II Builders, in turn, filed a motion to admit its Amended and Supplemental Complaint which deleted the prayer for resolution of the DAC initially prayed for in 99-1418. PBI also filed a complaint for damages and attachment before the RTC of Manila, Branch 12, docketed as Civil Case No. 99-95381, also predicated on the same set of facts. For
its original complaint. In lieu thereof, said pleading introduced causes of action for conveyance of title to and/or possession of the entire Asset Pool, for NHA to pay the Asset Pool the sum of his part, Consing filed a Motion to Dismiss which was, however, denied. Thereafter, he filed a Motion for Consolidation of Civil Case No. 99-1418 with his own initiated SCA No. 1759
P1,803,729,757.88 representing the cost of the changes and additional works on the project and for an increased indemnity for attorney’s fees in the sum of P2,000,000.00. pending before the RTC-Pasig City. The RTC-Makati City dismissed his motion for consolidation. His motion for reconsideration therefrom was also denied. Hence, he filed a petition for
certiorari before the CA ascribing grave abuse of discretion on the part of the RTC-Makati City in refusing his motion for consolidation. The CA rendered a Decision sustaining the orders of
Consistent with its joint order dated 2 January 2008 which held that R-II Builders’ complaint was an ordinary civil action and not an intra-corporate controversy, Branch 24 of the Manila RTC the RTC-Makati City which denied the motion for consolidation. Undaunted, he filed a motion for reconsideration therefrom but was denied.
issued a clarificatory order dated 1 February 2008 to the effect, among other matters, that it did not have the authority to hear the case. As a consequence, the case was re-raffled to
respondent Branch 22 of the Manila RTC. R-II Builders filed a motion to admit it Second Amended Complaint, on the ground that its previous Amended and Supplemental Complaint had not ISSUE:
yet been admitted in view of the non-payment of the correct docket fees therefor. Said Second Amended Complaint notably resurrected R-II Builders’ cause of action for resolution of the Whether or not the CA erred in upholding the RTC- Pasig City’s denial of Unicapital, et al.’s motion to dismiss.
DAC, deleted its causes of action for accounting and conveyance of title to and/or possession of the entire Asset Pool, reduced the claim for attorney’s fees to P500,000.00, sought its
appointment as Receiver pursuant to Rule 59 of the Rules of Court and, after an inventory in said capacity, prayed for approval of the liquidation and distribution of the Asset Pool in SC RULING:
accordance with the parties’ agreements. A cause of action is defined as the act or omission by which a party violates a right of another. It is well-settled that the existence of a cause of action is determined by the allegations in the
complaint. In this relation, a complaint is said to sufficiently assert a cause of action if, admitting what appears solely on its face to be correct, the plaintiff would be entitled to the relief prayed
On 2 September 2008, HGC filed its opposition to the admission of R-II Builders’ Second Amended Complaint on for. Thus, if the allegations furnish adequate basis by which the complaint can be maintained, then the same should not be dismissed, regardless of the defenses that may be averred by the
the ground that respondent RTC had no jurisdiction to act on the case until payment of the correct docket fees. defendants.

HGC’s contention: In this case, the Court finds that Consing, Jr.’s complaint in SCA No.1759 properly states a cause of action since the allegations there insufficiently bear out a case for damages under
Articles 19 and 26 of the Civil Code.
(a) the case is real action and the docket fees paid by R-II Builders were grossly insufficient because the estimated value of properties in the Asset Pool exceeds
P5,000,000,000.00. Records disclose that Consing, Jr.’s complaint contains allegations which aim to demonstrate the abusive manner in which Unicapital and PBI, et al. enforced their demands against him.
(b) a complaint cannot be amended to confer jurisdiction when the court had none; Accordingly, these specific allegations, if hypothetically admitted, may result into the recovery of damages pursuant to Article 19 of the Civil Code.
(c) the RTC should have simply denied the Urgent Ex-Parte Motion for Annotation of Lis Pendens instead of rendering an advisory opinion thereon. In addition, HGC faulted R-II
Builders with forum shopping, in view of its 10 filing of the complaint before Branch 91 of the Quezon City RTC, involving a claim for receivables from the NHA. In turn, R-II Neither should Consing, Jr.’s failureto pay the required docket fees lead to the dismissal of his complaint. It has long been settled that while the court acquires jurisdiction over any case only
Builders opposed the foregoing motion and, on the theory that the Asset Pool was still in danger of dissipation, filed an urgent motion to resolve its application for the upon the payment of the prescribed docket fees, its non-payment at the time of the filing of the complaint does not automatically cause the dismissal of the complaint provided that the fees
appointment of a receiver and submitted its nominees for said position. are paid within a reasonable period. Consequently, Unicapital, et al.’s insistence that the stringent rule on non-payment of docket fees enunciated in the case of Manchester Development
Corporation v. CA should be applied in this case cannot be sustained in the absence of proof that Consing, Jr. intended to defraud the government by his failure to pay the correct amount of
filing fees. As pronounced in the case of Heirs of Bertuldo Hinog v. Hon. Melicor:
RTC:
Plainly, while the payment of the prescribed docket fee is a jurisdictional requirement, even its non-payment at the time of filing does not automatically cause the dismissal of
(a) denied HGC's motion for reconsideration;
the case, as long as the fee is paid within the applicable prescriptive or reglementary period, more so when the party involved demonstrates a willingness to abide by the
(b) granted R-II Builders' application for appointment of receiver
rules prescribing such payment. Thus, when insufficient filing fees were initially paid by the plaintiffs and there was no intention to defraud the government, the Manchester
rule does not apply.
Imputing grave abuse of discretion against the RTC for not dismissing the case and for granting R-II Builders' application for receivership, HGC filed the Rule 65 petition for certiorari and
prohibition before the CA which rendered decision, upon the following findings and conclusions:
In fine, the Court finds no reversible error on the part of the CA in sustaining the RTC-Pasig City's denial of Unicapital, et al.'s motion to dismiss. As such, the petitions in G.R. Nos. 175277
and 175285 must be denied.
(a) Irrespective of whether it is real or one incapable of pecuniary estimation, the action commenced by R-II Builders indubitably falls squarely within the jurisdiction of
respondent RTC;
(b) From the allegations of R-II Builders' original complaint and amended complaint the character of the relief primarily sought, i.e., the declaration of nullity of the DAC, the DURISOL Philippines, Inc. vs Court of Appeals
action before respondent RTC is one where the subject matter is incapable of pecuniary estimation; G.R. No. 121106; February 20, 2002
(c) R-II Builders need not pay any deficiency in the docket fees considering its withdrawal of its Amended and Supplemental Complaint; TOPIC: Kinds of Jurisdiction-General Jurisdiction-RTC
(d) A receiver may be appointed without formal hearing, particularly when it is within the interest of both parties and does not result in the delay of any government infrastructure
projects or economic development efforts; CASE NATURE:
(e) Respondent RTC's act of calling the attention of the Manila Registrar of Deeds to R-II Builders' Urgent Ex-Parte Motion for Annotation of Lis Pendens is well-within its This is a petition for review of the decision of the Court of Appeals in dismissing petitioner Durisol Philippines, Inc.’s petition for annulment of judgment.
residual power to act on matters before it; and
(f) The withdrawal of R-II Builders' Amended and Supplemental Complaint discounted the forum shopping imputed against it by HGC. FACTS:
On January 17, 1962 and December 5, 1969, petitioner Durisol obtained industrial loans from respondent Development Bank o3f the Philippines (DBP) amounting to
P1,213,000.00 and P2,698,800.00, respectively. As security therefor, petitioner executed a mortgage on two parcels of registered land located in Polo (now Valenzuela), Bulacan, covered by
ISSUES: Transfer Certificates of Title Nos. 29906 and 29909.
CAUSE OF THE DISPUTE: ARTICLE 143. Original jurisdiction. — (1) The Shari'a District Court shall have exclusive original jurisdiction over:
After petitioner defaulted in the payment of the loans, DBP instituted a petition for the extrajudicial foreclosure of mortgage. On March 6, 1972, petitioner’s president, Rene xxxx
Knecht, borrowed from DBP the two TCT’s purportedly to obtain new titles in accordance with the approved subdivision plan of the properties. DBP agreed provided that the bank’s existing (b) All cases involving disposition, distribution and settlement of the estate of deceased Muslims, probate of wills, issuance of
encumbrances, including the mortgage, shall be annotated on all the new certificates of title. Contrary to its promise, however, petitioner never returned the titles to the properties to DBP. letters of administration or appointment of administrators or executors regardless of the nature or the aggregate value of the property.
Thus, despite having purchased the properties at the foreclosure sale, DBP was unable to register the property in its name. The determination of the nature of an action or proceeding is controlled by the averments and character of the relief sought in the complaint or petition. The designation
The foreclosure sale was already held, wherein DBP emerged as the highest bidder. Petitioner, however, filed a complaint for annulment of the extrajudicial foreclosure given by parties to their own pleadings does not necessarily bind the courts to treat it according to the said designation. Rather than rely on "a falsa descriptio or defective caption," courts
before the then Court of First Instance (CFI) of Valenzuela, Bulacan. The CFI rendered judgment upholding the validity of the foreclosure which then affirmed by the Court of Appeals. are "guided by the substantive averments of the pleadings."
CASE ORIGINALLY FILED: The court did not agree with the contention of the petitioners that the district court does not have jurisdiction over the case because of an allegation in their answer with a
On February 25, 1977, DBP instituted before the Court of First Instance of Valenzuela, Bulacan, Branch VIII, a petition for surrender of the owner’s duplicate titles covering motion to dismiss that Montañer, Sr. is not a Muslim. Jurisdiction of a court over the nature of the action and its subject matter does not depend upon the defenses set forth in an answer or a
the foreclosed properties. motion to dismiss. Otherwise, jurisdiction would depend almost entirely on the defendant or result in having "a case either thrown out of court or its proceedings unduly delayed by simple
PLAINTIFF’S AVERMENTS: stratagem. Indeed, the "defense of lack of jurisdiction which is dependent on a question of fact does not render the court to lose or be deprived of its jurisdiction."
That petitioner failed to exercise its right of redemption of the properties which were sold at public auction after foreclosure of the mortgage thereof. The Shari’a District Court has the authority to hear and receive evidence to determine whether it has jurisdiction, which requires an a priori determination that the deceased
DEFENDANT’S ANSWER: is a Muslim. If after hearing, the Shari’a District Court determines that the deceased was not in fact a Muslim, the district court should dismiss the case for lack of jurisdiction.
Raising the defenses that the petition fails to state a cause of action; that it had already paid its loans to DBP; that it had a valid adverse claim on the properties covered by
the seven new titles; and that DBP’s action was barred by laches and estoppel.
RTC: Misael Vera v. Hon. Judge Francisco Arca
The trial court rendered summary judgment, ordering petitioner to surrender to the court within five days the seven certificates of title. G.R. No. L-25721, May 26, 1969
Petitioner filed a motion for reconsideration, which contained an alternative prayer to record in the titles its adverse claim representing the amount of improvements it TOPIC: Cases on the constitutionality of treaty, international or executive agreement, law, presidential decree, proclamation, order, instruction
introduced on the property.
The lower court denied petitioner’s motion for reconsideration FACTS:
CA: It was alleged that the Tax Census Act has been enforced and implemented since 1962 and that the parties who filed the suit, now respondents before us, "are required to
Petitioner thus appealed to the Intermediate Appellate Court, the IAC rendered a decision ordering that the case be remanded to the lower court for further proceedings. The make and file Sworn Statements of Assets, Income and Liabilities" in accordance with the Act. It was likewise asserted that the then Senator Camilo Osias filed a bill to repeal such
IAC held that it was improper for the trial court to render summary judgment because there were genuine issues involved. This decision became final and executory. legislation and that the Secretary of Finance had admitted that it had not been able to produce the result expected from it, the information yielded not justifying the trouble caused not only to
More than four years later, or on September 2, 1994, petitioner instituted before the Court of Appeals a petition to annul the trial court’s decision and Resolution, alleging for the public but also to the government.
the first time that the trial court had no jurisdiction over the case. The main portion of the petition before respondent Judge dealt with the alleged infirmity of the Tax Census Act as being violative of the constitutional right to liberty, to the
The Court of Appeals rendered the now assailed decision dismissing the petition for annulment of judgment. Petitioner Durisol’s subsequent motion for reconsideration was guarantee against self-incrimination and the protection against unreasonable searches and seizures with a citation from both Philippine and American cases in support of such a plea. A writ
likewise denied for lack of merit. of preliminary injunction was therein likewise sought, the argument being advanced that the other respondents before us in the petition before respondent Judge "would suffer great and
ISSUE: irreparable damage arising from the non-filing of their Sworn Statement of Assets, Income and Liabilities, as they are subject to criminal prosecution under Section 5 of the Tax Census Law
(1) Whether or not the trial court had jurisdiction over the petition for issuance of new duplicate owner’s certificate of title; and which is in clear violation of herein petitioners' aforesaid constitutional and legal rights and which would render the judgment in favor of herein petitioners ineffectual"
(2) Whether or not petitioner was estopped from challenging the court’s lack of jurisdiction.

RULING: YES CAUSE OF THE DISPUTE:


The first paragraph of Rule 47, Section 2, of the 1997 Rules of Civil Procedure provides:
Grounds for annulment. — The annulment may be based only on the ground of extrinsic fraud and lack of jurisdiction. Then came the order of respondent Judge, which is the basis of the present petition for certiorari and prohibition, noting that a hearing on the plea for the issuance of the writ
At the outset, it should be stressed that in a petition for annulment of judgment based on lack of jurisdiction, petitioner must show not merely an abuse of jurisdictional for preliminary injunction took place on February 19, 1966 and ordering the issuance thereof upon the posting of a bond of P1,000.00, thus restraining petitioners from requiring the other
discretion but an absolute lack of jurisdiction. Lack of jurisdiction means absence of or no jurisdiction, that is, the court should not have taken cognizance of the petition because the law respondents and other similarly situated to file their sworn statements of assets, income and liabilities under Republic Act No. 2070.
does not vest it with jurisdiction over the subject matter. Jurisdiction over the nature of the action or subject matter is conferred by law.
The regional trial court, formerly the court of first instance, is a court of general jurisdiction. All cases, the jurisdiction over which is not specifically provided for by law to be
within the jurisdiction of any other court, fall under the jurisdiction of the regional trial court. But the regional trial court is also a court of limited jurisdiction over, among others, cadastral and
PLAINTIFF’S AVERMENTS:
land registration cases. All proceedings involving title to real property, or specifically land registration cases, including its incidents such as the issuance of owner’s duplicate certificate of
Petitioners predicate their plea on the allegation that respondent Judge gravely abused his discretion in issuing the writ of preliminary injunction as the Tax Census Act is
title, are matters cognizable by the regional trial courts. It has been ruled that the regional trial courts have jurisdiction over all actions involving possession of land, except forcible entry and
valid and constitutional, there being neither any self-incrimination feature nor unreasonable search and seizure taint, there being moreover a presumption of its conformity with the
illegal detainer
fundamental law and no grave and irreparable injury being suffered by the other respondents, petitioners before respondent Judge.
Even assuming arguendo that the regional trial court had no jurisdiction over the surrender of duplicate title, petitioner can no longer raise this ground after having actively
Petitioners likewise justify their contention that there was a grave abuse of discretion on the part of respondent Judge in the issuance of such writ of preliminary injunction
participated in the prosecution of the case. A judgment rendered by a trial court for alleged lack of jurisdiction cannot be considered void where the party who has the right to challenge it
due to his failure to consider the serious injury it would cause the paramount public interest, to realize that the enforcement of penal laws cannot thus be restrained and to take note that the
failed to do so at the first instance.
other respondents as petitioners before him are guilty of laches.
Indeed, it was only two decades after the institution of the case at bar, when the issue of lack of jurisdiction was first raised. However, it is already too late since the judgment
PLAINTIFF’S PRAYER:
had already attained finality, considering that more than four years have elapsed without any action from petitioner.
Petitioners in this special civil action seek the setting aside of the writ of preliminary injunction issued by respondent Judge and would restrain him perpetually from further
Rule 47, Section 3 expressly provides that a petition for annulment of judgment based on lack of jurisdiction must be filed before it is barred by laches or estoppel. Hence, it
hearing the suit for prohibition and injunction pending before him.
has been held that while jurisdiction over the subject matter of a case may be raised at any time of the proceedings, this rule presupposes that laches or estoppel has not supervened.
DEFENDANT’S ANSWER:
Respondents reiterate their argument against the validity of the Act for the asserted transgression on the constitutional protection against self-incrimination and against
Montañer v. Shari’ah District Court unreasonable searches and seizures.
G.R. No. 174975, January 20, 2009 They did likewise question its validity as being in excess of the State's taxing power.
TOPIC: Exclusive Original Jurisdiction- Shari’ah District Court-Settlement of Estate of Deceased Muslims ISSUE:
WHETHER OR NOT RESPONDENT JUDGE OUGHT TO HAVE ISSUED THE WRIT OF PRELIMINARY INJUNCTION TO RESTRAIN THE ENFORCEMENT OF THE TAX CENSUS ACT.
CASE NATURE: RULING: NO
This Petition for Certiorari and Prohibition seeks to set aside the Orders of the Shari’a District Court, Fourth Shari’a Judicial District, Marawi City. Whatever may be said of the original petition for prohibition and injunction filed by the other respondents before the respondent Judge, it cannot be plausibly asserted the
facts have been alleged which would make manifest the violation of any of their constitutional rights. Instead of relying on facts, they contended themselves with the general allegation that
FACTS: for them the Tax Census Act was null. It was assailed for presumably violating the right to liberty, the protection against unreasonable searches and seizures and the prohibition against self-
incrimination. What was thus being sought in effect was a declaration of invalidity based on the belief that its constitutional infirmity is apparent on its face.
Respondent Judge took into consideration purely legal arguments, no evidence being introduced, both for and against the validity of the challenged statute. Moreover, his
attention was invited to the presumption of validity that every legislative act has in its favor as well as the doctrine that the task of suspending the operation of the law "is a matter of extreme
On August 17, 1956, petitioner Luisa Kho Montañer, a Roman Catholic, married Alejandro Montañer, Sr. at the Immaculate Conception Parish in Cubao, Quezon City.
delicacy because that is an interference with the official acts not only with the duly elected representatives of the people in Congress but also of the highest magistrate of the
Petitioners Alejandro Montañer, Jr., Lillibeth Montañer-Barrios, and Rhodora Eleanor Montañer-Dalupan are their children. On May 26, 1995, Alejandro Montañer, Sr. died.
land." Respondent Judge was deaf to the force of such cogent and persuasive constitutional law doctrines. He issued the preliminary injunction nonetheless.
CASE ORIGINALLY FILED:
It is manifest that respondent Judge did overstep the bounds of discretion that set limits to the authority he is entitled to exercise in the issuance of the preliminary injunction
On August 19, 2005, private respondents Liling Disangcopan and her daughter, Almahleen Liling S. Montañer, both Muslims, filed a "Complaint" for the judicial partition of
to restrain the enforcement of a statute. There can be no dissent from the proposition that where the action required of a lower court would be tantamount, even if only for a temporary
properties before the Shari’a District Court.
period, to disregarding the clearly expressed will of the two branches of the government, the need for caution is greatest. Here, respondent Judge was apparently oblivious of such a need.
PLAINTIFF’S AVERMENTS:
Respondent Judge did possess discretion to issue or not to issue a preliminary injunction. That discretion, however, according to the circumstances disclosed, was abused,
(1) in May 1995, Alejandro Montañer, Sr. died;
and abused gravely.
(2) the late Alejandro Montañer, Sr. is a Muslim;
(3) petitioners are the first family of the decedent;
(4) Liling Disangcopan is the widow of the decedent;
(5) Almahleen Liling S. Montañer is the daughter of the decedent; and Constancio Mendoza v. Mayor Enrilo Villas
(6) the estimated value of and a list of the properties comprising the estate of the decedent. .
PLAINTIFF’S PRAYER: G.R. No. 187256, February 23, 2011
Private respondents prayed for the Shari’a District Court to order, among others, the following: TOPIC: Petitions for Certiorari, Prohibition, Mandamus, Quo Warranto, Habeas Corpus, writs of Amparo, Habeas Data, Kalikasan, injunctions
(1) the partition of the estate of the decedent; and CASE NATURE:
(2) the appointment of an administrator for the estate of the decedent This is a Petition filed by Constancio F. Mendoza and Sangguniang Barangay of Balatasan, Bulalacao, Oriental Mindoro. In the Petition, it is prayed that the Court: (1) set
DEFENDANT’S ANSWER: aside the Order dated February 2, 2009 of the Regional Trial Court (RTC), Branch 43 in Roxas, Oriental Mindoro denying petitioners’ motion for reconsideration of the Order dated February
Petitioners filed an Answer with a Motion to Dismiss mainly on the following grounds: 2, 2009; and (2) direct the RTC to continue with the proceedings in Special Civil Action No. 08-10 entitled Constancio Mendoza v. Mayor Enrilo Villas.
(1) the Shari’a District Court has no jurisdiction over the estate of the late Alejandro Montañer, Sr., because he was a Roman Catholic; FACTS:
(2) private respondents failed to pay the correct amount of docket fees; and In the 2007 barangay elections, Mendoza obtained the highest votes for the position of Punong Barangay of Barangay Balatasan, Bulalacao, Oriental Mindoro, while
(3) private respondents’ complaint is barred by prescription, as it seeks to establish filiation between Almahleen Liling S. Montañer and the decedent, pursuant to Article 175 respondent Liwanag Herato obtained the highest number of votes for the position of Barangay Kagawad. Notably, Mayor Enrilo Villas was the incumbent Mayor of Bulalacao, Oriental
of the Family Code. Mindoro at the time of the barangay elections.

On November 22, 2005, the Shari’a District Court dismissed the private respondents’ complaint. The district court held that Alejandro Montañer, Sr. was not a Muslim, and its
jurisdiction extends only to the settlement and distribution of the estate of deceased Muslims.
ISSUE: CAUSE OF THE DISPUTE:
WHETHER OR NOT THE SHARI’A DISTRICT COURT HAS JURISDICTION OVER THE SETTLEMENT OF THE ESTATE OF DECEASED MUSLIMS;
WHETHER OR NOT SHARI’A DISTRICT COURT – MARAWI CITY LACKS JURISDICTION OVER PETITIONERS WHO ARE ROMAN CATHOLICS AND NON-MUSLIMS.
After the elections, the Commission on Elections (COMELEC) proclaimed Mendoza as the duly-elected Punong Barangay of Balatasan. Thus, the losing candidate, Thomas
RULING: YES Pajanel, filed a petition for quo warranto with the Municipal Trial Court (MTC) of Mansalay-Bulalacao. The MTC issued a Decision dated February 23, 2008, disqualifying Mendoza and
Article 143(b) of Presidential Decree No. 1083, otherwise known as the Code of Muslim Personal Laws of the Philippines, provides that the Shari’a District declaring that Herato was entitled to succeed him as Punong Barangay with Herato garnering the highest number of votes as a Barangay Kagawad. Mendoza appealed the MTC Decision to
Courts have exclusive original jurisdiction over the settlement of the estate of deceased Muslims: the COMELEC.
On February 28, 2008, Villas administered the Oath of Office to Herato. Then, Villas issued a Memorandum directing all department heads of the Municipal Government to Program (CARP) coverage; (3) those to be converted to non-agricultural use other than that previously authorized; and (4) those reclassified to residential, commercial, industrial, or other
act only on documents signed or authorized by Herato. non-agricultural uses on or after the effectivity of Republic Act No. 6657 and are to be converted to such uses.
Meanwhile, Mendoza sought the advice of the Department of the Interior and Local Government (DILG) as to who should exercise the powers of Punong Barangay of On 28 February 2002, the Secretary of Agrarian Reform issued another Administrative Order, i.e., DAR AO No. 01-02, entitled "2002 Comprehensive Rules on Land Use
Balatasan given the prevailing controversy. DILG Undersecretary Austere A. Panadero responded to Mendoza’s inquiry informing Villas that Mendoza should occupy the post of Punong Conversion," which further amended DAR AO No. 07-97 and DAR AO No. 01-99, and repealed all issuances inconsistent therewith. The aforesaid DAR AO No. 01-02 covers all applications
Barangay as there was no Writ of Execution Pending Appeal of the MTC Decision. for conversion from agricultural to non-agricultural uses or to another agricultural use.
CAUSE OF THE DISPUTE:
Thereafter, the Secretary of Agrarian Reform amended certain provisions of DAR AO No. 01-02 by formulating DAR AO No. 05-07, particularly addressing land conversion in
time of exigencies and calamities.
CASE ORIGINALLY FILED:
To address the unabated conversion of prime agricultural lands for real estate development, the Secretary of Agrarian Reform further issued Memorandum No. 88, which
Petitioners filed a Petition for Mandamus with Damages and Prayer for the Writ of Preliminary Mandatory Injunction, pending with the Regional Trial Court, Branch 43 in
temporarily suspended the processing and approval of all land use conversion applications.
Roxas, Oriental Mindoro.
PLAINTIFF’S PRAYER: By reason thereof, petitioner claims that there is an actual slowdown of housing projects, which, in turn, aggravated the housing shortage, unemployment and illegal
squatting problems to the substantial prejudice not only of the petitioner and its members but more so of the whole nation.
Petitioners prayed that the LBP be directed to release the funds of Barangay Balatasan to them in order to render necessary, basic public services to the inhabitants of the
PLAINTIFF’S AVERMENTS:
barangay.
Petitioner contends that DAR AO No. 01-02, as amended, was made in violation of Section 65 of Republic Act No. 6657 because it covers all applications for conversion
DEFENDANT’S ANSWER:
from agricultural to non-agricultural uses or to other agricultural uses, such as the conversion of agricultural lands or areas that have been reclassified by the LGUs or by way of Presidential
Proclamations, to residential, commercial, industrial or other non-agricultural uses.
Petitioner further asseverates that Section 2.19, Article I of DAR AO No. 01-02, as amended, making reclassification of agricultural lands subject to the requirements and
Villas and Herato interpose the following affirmative defenses: procedure for land use conversion, violates Section 20 of Republic Act No. 7160, because it was not provided therein that reclassification by LGUs shall be subject to conversion procedures
(1) that the petition for mandamus was defective, being directed against two or more different entities and requiring to perform different acts; and or requirements, or that the DAR’s approval or clearance must be secured to effect reclassification.
(2) that Mendoza does not have any clear and legal right for the writ of mandamus. Petitioner similarly avers that the promulgation and enforcement of DAR AO No. 01-02, as amended, constitute deprivation of liberty and property without due process of law.
There is deprivation of liberty and property without due process of law because under DAR AO No. 01-02, as amended, lands that are not within DAR’s jurisdiction are unjustly, arbitrarily and
oppressively prohibited or restricted from legitimate use on pain of administrative and criminal penalties.
RTC: PLAINTIFF’S PRAYER:
Petitioner avows that DAR Memorandum No. 88 is not a valid exercise of police power for it is the prerogative of the legislature and that it is unconstitutional because it
suspended the land use conversion without any basis.
ISSUE:
The RTC issued the assailed order dismissing the petition on the strength of the COMELEC Resolution disqualifying Mendoza from running in the 2007 elections. As stated, WHETHER OR NOT THE DAR SECRETARY HAS JURISDICTION OVER LANDS THAT HAVE BEEN RECLASSIFIED AS RESIDENTIAL, COMMERCIAL, INDUSTRIAL, OR FOR OTHER
petitioners’ motion for reconsideration of the Order was denied. NON-AGRICULTURAL USES.
From such orders the petitioners went directly to the Supreme Court. RULING: YES
The authority of the Secretary of Agrarian Reform to include "lands not reclassified as residential, commercial, industrial or other non-agricultural uses before 15 June 1988"
in the definition of agricultural lands finds basis in jurisprudence.
ISSUE: In Ros v. Department of Agrarian Reform, this Court has enunciated that after the passage of Republic Act No. 6657, agricultural lands, though reclassified, have to go
WHETHER OR NOT THE SUPREME COURT MAY ENTERTAIN THE CASE. through the process of conversion, jurisdiction over which is vested in the DAR. However, agricultural lands, which are already reclassified before the effectivity of Republic Act No. 6657
RULING: NO which is 15 June 1988, are exempted from conversion.
It bears stressing that the said date of effectivity of Republic Act No. 6657 served as the cut-off period for automatic reclassifications or rezoning of agricultural lands that no
longer require any DAR conversion clearance or authority. It necessarily follows that any reclassification made thereafter can be the subject of DAR’s conversion authority. Having
recognized the DAR’s conversion authority over lands reclassified after 15 June 1988, it can no longer be argued that the Secretary of Agrarian Reform was wrongfully given the authority
The instant petition is a direct recourse from the assailed orders of the RTC. Notably, petitioners did not cite the rule under the Rules of Court by which the petition was filed. and power to include "lands not reclassified as residential, commercial, industrial or other non-agricultural uses before 15 June 1988" in the definition of agricultural lands.
If the petition is to be treated as a petition filed under Rule 65 of the Rules of Court, the petition must be dismissed outright for having been filed prematurely. Primarily, although this Court, the Court of Appeals and the Regional Trial Courts have concurrent jurisdiction to issue writs of certiorari, prohibition, mandamus, quo
Primarily, although the Supreme Court, the Court of Appeals and the Regional Trial Courts have concurrent jurisdiction to issue writs of certiorari, prohibition, mandamus, warranto, habeas corpus and injunction, such concurrence does not give the petitioner unrestricted freedom of choice of court forum.
quo warranto, habeas corpus and injunction, such concurrence does not give the petitioner unrestricted freedom of choice of court forum. The rationale for this rule is two-fold: (a) it would be an imposition upon the precious time of this Court; and (b) it would cause an inevitable and resultant delay, intended or
Similarly, there are no special and important reasons that petitioners cite to justify their direct recourse to this Court under Rule 65. On the other hand, direct recourse to the otherwise, in the adjudication of cases, which in some instances had to be remanded or referred to the lower court as the proper forum under the rules of procedure, or as better equipped to
Court has been allowed for petitions filed under Rule 45 when only questions of law are raised, as in this case. resolve the issues because this Court is not a trier of facts.
Section 1 of Rule 45 clearly states that the following may be appealed to the Supreme Court through a petition for review by certiorari: 1) judgments; 2) final The Court thus reaffirms the judicial policy that it will not entertain direct resort to it unless the redress desired cannot be obtained in the appropriate courts, and exceptional
orders; or 3) resolutions of the Court of Appeals, the Sandiganbayan, the Regional Trial Court or similar courts, whenever authorized by law. The appeal must involve only and compelling circumstances, such as cases of national interest and of serious implications, justify the availment of the extraordinary remedy of writ of certiorari, calling for the exercise of
questions of law, not of fact. its primary jurisdiction.
A direct recourse to the Supreme Court can be taken for a review of the decisions, final orders or resolutions of the RTC, but only on questions of law. Under Section 5 of In the case at bench, petitioner failed to specifically and sufficiently set forth special and important reasons to justify direct recourse to this Court and why this Court should
Article VIII of the Constitution, the Supreme Court has the power to give due course to this petition in the first instance. The present petition should have been initially filed in the Court of Appeals in strict observance of the doctrine on the hierarchy of courts.
(2) Review, revise, reverse, modify, or affirm on appeal or certiorari as the law or the Rules of Court may provide, final judgments and orders of lower courts Failure to do so is sufficient cause for the dismissal of this petition.
in:
(e) All cases in which only an error or question of law is involved.
This kind of direct appeal to this Court of RTC judgments, final orders or resolutions is provided for in Section 2(c) of Rule 41, which reads: Allied Domecq Phil. v. Hon. Villon
SEC. 2. Modes of appeal. G.R. No. 156264, September 30, 2004
(c) Appeal by certiorari. In all cases where only questions of law are raised or involved, the appeal shall be to the Supreme Court by petition for review on TOPIC: Manner of Acquiring Jurisdiction over subject matter
certiorari in accordance with Rule 45. CASE NATURE:
Petition for review on certiorari assailing the Decision of the Court of Appeals dated May 27, 2002 and Resolution dated November 29, 2002 in CA-G.R. SP No. 63802 which dismissed the
Procedurally then, petitioners could have appealed the RTC Decision affirming the MTC (1) to the Court on questions of law only; or (2) if there are factual questions special civil action for certiorari filed by Allied Domecq, Philippines Inc. (ADPI), herein petitioner, for want of jurisdiction.
involved, to the CA -- as they in fact did. Unfortunately for petitioners, the CA properly dismissed their petition for review because of serious procedural defects. This action foreclosed their FACTS:
only available avenue for the review of the factual findings of the RTC. On May 8, 1996, petitioner ADPI entered into an exclusive distributorship agreement with Pedro Domecq, S.A., a corporation organized and existing under the laws of Spain, engaged in the
Thus, the Court shall exercise liberality and consider the instant petition as one filed under Rule 45. The Court noted that it has the discretion to determine whether a petition manufacture of wine and brandy. Under the said agreement, Pedro Domecq, S.A. granted petitioner the sole and exclusive right to import and distribute in the Philippines various Pedro
was filed under Rule 45 or 65 of the Rules of Court. Domecq, S.A. products including Fundador brandy until May 17, 2000. Upon its expiration, the agreement is deemed automatically extended for an indefinite period of time
Nevertheless, even providing that the petition was not filed prematurely, it must still be dismissed for having become moot and academic. Petitioner then applied for a Certificate of Registration with the Bureau of Food and Drugs (BFAD), pursuant to Department of Health Administrative Order No. 17, series of 1979, requiring all
With the conduct of the 2010 barangay elections, a supervening event has transpired that has rendered this case moot and academic and subject to dismissal. This is imported food products to be registered with the BFAD prior to their distribution in the local markets.
because Mendoza’s term of office has expired with the conduct of last year’s local elections. As such, Special Civil Action No. 08-10, where the assailed Orders were issued, can no longer On June 2, 1998, BFAD wrote then Director Quintin L. Kintanar of the Bureau of Customs, requesting that entry of imported shipments of Fundador brandy should not be allowed in
prosper. Mendoza no longer has any legal standing to further pursue the case, rendering the instant petition moot and academic. the Philippines, unless the importer presents a valid Certificate of Registration issued by the BFAD. The Bureau of Customs granted petitioners request and on July 13, 1998, issued
Customs Memorandum Circular No. 228-098.
Cause of dispute:
On April 12, 1999, Clark Liberty Warehouse, Inc. (Clark Liberty), herein private respondent, a duly licensed duty-free shop operating in the Clark Special Economic Zone, imported 800 cases
or a total of 9,420 bottles of Fundador brandy.
CREBA v. DAR Secretary Since the importation by respondent Clark Liberty was not covered by the BFAD Certificate of Product Registration, the Bureau of Customs seized and impounded the shipment pursuant to
Customs Memorandum Circular No. 228-98, in relation to Sections 101 (K) and 2530 of the Tariff and Customs Code.The imported brandy then became the subject of seizure proceedings
G.R. 183409, June 18, 2010 before the District Collector of Customs of the Port of Manila, docketed as S.I. No. 99-140
TOPIC: Petitions for Certiorari, Prohibition
Plaintiff’s argument:
CASE NATURE: Petitioner then filed a motion to intervene in S.I. No. 99-140 alleging, among others, that it sustained damages caused by respondent Clark Libertys illegal importation. However, the Bureau
This is Petition for Certiorari and Prohibition (with application for temporary restraining order and/or writ of preliminary injunction) under Rule 65 of the 1997 Revised Rules of of Customs District Collector failed to resolve the motion.
Civil Procedure, filed by herein petitioner Chamber of Real Estate and Builders Associations, Inc. (CREBA) seeking to nullify and prohibit the enforcement of Department of Agrarian Reform
(DAR) Administrative Order (AO) No. 01-02, as amended by DAR AO No. 05-07,and DAR Memorandum No. 88, for having been issued by the Secretary of Agrarian Reform with grave Plaintiff’s prayer:
abuse of discretion amounting to lack or excess of jurisdiction as some provisions of the aforesaid administrative issuances are illegal and unconstitutional. Issuance of a temporary restraining order (TRO) and a writ of preliminary injunction and demanding that the Clark Liberty cease and desist from importing, distributing, selling, or marketing
FACTS: Fundador brandy in the Philippines and refrain from claiming the seized shipment and participating in the seizure proceedings.

Defendant’s answer:
Petitioner CREBA, a private non-stock, non-profit corporation duly organized and existing under the laws of the Republic of the Philippines, is the umbrella organization of Refused to petioner’s demand
some 3,500 private corporations, partnerships, single proprietorships and individuals directly or indirectly involved in land and housing development, building and infrastructure construction,
materials production and supply, and services in the various related fields of engineering, architecture, community planning and development financing. The Secretary of Agrarian Reform is RTC:
named respondent as he is the duly appointive head of the DAR whose administrative issuances are the subject of this petition. Dismissed the case
The Secretary of Agrarian Reform issued, on 29 October 1997, DAR AO No. 07-97, entitled "Omnibus Rules and Procedures Governing Conversion of Agricultural Lands to Basis/reason
Non-Agricultural Uses," which consolidated all existing implementing guidelines related to land use conversion. The aforesaid rules embraced all private agricultural lands regardless of The trial court held that petitioner failed to prove that respondent Clark Liberty engaged in unfair competition as there is no showing that it employed deceit or otherwise
tenurial arrangement and commodity produced, and all untitled agricultural lands and agricultural lands reclassified by Local Government Units (LGUs) into non-agricultural uses after 15 committed acts constituting bad faith; that the bottles of Fundador brandy imported by respondent are the ones imported by plaintiff and that these bottles are not genuine,
June 1988. defective, or of poor quality.
Subsequently, on 30 March 1999, the Secretary of Agrarian Reform issued DAR AO No. 01-99, entitled "Revised Rules and Regulations on the Conversion of Agricultural Motion for reconsideration RTC:
Lands to Non-agricultural Uses," amending and updating the previous rules on land use conversion. Denied
Its coverage includes the following agricultural lands, to wit: (1) those to be converted to residential, commercial, industrial, institutional and other non-agricultural purposes;
(2) those to be devoted to another type of agricultural activity such as livestock, poultry, and fishpond ─ the effect of which is to exempt the land from the Comprehensive Agrarian Reform CA:
Dismissed the case The petition is meritorious.
Basis/reason Respondents filed the complaint in 1999, at the time Batas Pambansa Blg. (BP) 129, the Judiciary Reorganization Act of 1980, was already amended by Republic Act (RA) No. 7691, An Act
Lack of jurisdiction as quoted Expanding the Jurisdiction of the Metropolitan Trial Courts, Municipal Trial Courts, and Municipal Circuit Trial Courts, amending for the purpose BP Blg. 129.
It is an admitted fact respondent Clark Liberty is one of the duly licensed and authorized duty free shops at the Clark Special Economic Zone since 1998 which sells Section 1 of RA 7691, amending BP Blg. 129, provides that the RTC shall exercise exclusive original jurisdiction on the following actions:
imported grocery items including liquors, appliances, household wares, etc. and is exclusively regulated by the Clark Development Corporation, created by Republic Act No. Section 1. Section 19 of Batas Pambansa Blg. 129, otherwise known as the "Judiciary Reorganization Act of 1980," is hereby amended to read as follows:
7227, known as the Bases Conversion and Development Act of 1992. It is therefore a juridical creation of Republic Act No. 7227 in relation to Executive Order No. 62 and Sec. 19. Jurisdiction in civil cases. – Regional Trial Courts shall exercise exclusive original jurisdiction:
Presidential Proclamation No. 163, creating the Clark Special Economic Zone, under the exclusive jurisdiction, authority and regulation of the Clark Development (1) In all civil actions in which the subject of the litigation is incapable of pecuniary estimation;
Corporation. As such juridical creation, this Court has no jurisdiction to determine whether or not petitioner is entitled to the issuance of an injunctive relief since such (2) In all civil actions which involve the title to, or possession of, real property, or any interest therein, where the assessed value of the property involved exceeds Twenty Thousand Pesos
authority and jurisdiction belong the Honorable Supreme Court in accordance with Section 21 of Republic Act No. 7227 (₱20,000.00) or for civil actions in Metro Manila, where such value exceeds Fifty Thousand Pesos (₱50,000.00), except actions for forcible entry into and unlawful detainer of lands or
buildings, original jurisdiction over which is conferred upon the Metropolitan Trial Courts, Municipal Trial Courts, and Municipal Circuit Trial Courts; x x x
ISSUE: Section 3 of RA 7691 expanded the exclusive original jurisdiction of the first level courts, thus:
Wether or not the CA has jurisdiction of the case Section 3. Section 33 of the same law BP Blg. 129 is hereby amended to read as follows:
Sec. 33. Jurisdiction of Metropolitan Trial Courts, Municipal Trial Courts and Municipal Circuit Trial Courts in Civil Cases. – Metropolitan Trial Courts, Municipal Trial Courts, and Municipal
RULING: Circuit Trial Courts shall exercise:
Jurisdiction is the authority to hear and determine a cause. Jurisdiction over the subject matter is the power to hear and determine the general class to which the xxxx
proceedings in question belong. Jurisdiction over the subject matter is conferred by law and not by the consent or acquiescence of any or all of the parties or by erroneous (3) Exclusive original jurisdiction in all civil actions which involve title to, or possession of, real property, or any interest therein where the assessed value of the property or interest therein
belief of the court that it exists. Basic is the rule that jurisdiction over the subject matter is determined by the cause or causes of action as alleged in the complaint. But where does not exceed Twenty Thousand Pesos (₱20,000.00) or, in civil actions in Metro Manila, where such assessed value does not exceed Fifty Thousand Pesos (₱50,000.00) exclusive of
the actual issues are evident from the records of the case, then jurisdiction over the subject matter need not depend upon the literal averments in the complaint, but on the interest, damages of whatever kind, attorney's fees, litigation expenses and costs: Provided, That in cases of land not declared for taxation purposes, the value of such property shall be
law as applied to established facts. determined by the assessed value of the adjacent lots.
Here, in order to determine whether the court a quo has jurisdiction over petitioners complaint for injunction, we have to interpret the law as applied to the established Respondents filed their Complaint with the RTC; hence, before proceeding any further with any other issues raised by the petitioner, it is essential to ascertain whether the RTC has
facts. There is no question that respondent Clark Liberty is a registered enterprise of the Clark Special Economic Zone and is primarily regulated by R.A. No. 7227, jurisdiction over the subject matter of this case based on the above-quoted provisions.
otherwise known as the Bases Conversion and Development Act of 1992. However, in order to determine which court has jurisdiction over the action, an examination of the complaint is essential. Basic as a hornbook principle is that jurisdiction over the subject
The underlying purpose of the Legislature in enacting R.A. No. 7227 is provided by Section 2, thus: matter of a case is conferred by law and determined by the allegations in the complaint which comprise a concise statement of the ultimate facts constituting the plaintiff's cause of action.
SEC. 2. Declaration of Policies. It is hereby declared the policy of the Government to accelerate the sound and balanced conversion into alternative productive uses of the The nature of an action, as well as which court or body has jurisdiction over it, is determined based on the allegations contained in the complaint of the plaintiff, irrespective of whether or not
Clark and Subic military reservations and their extensions (John Hay Station, Wallace Air Station, ODonnell Transmitter Station, San Miguel Naval Communications Station the plaintiff is entitled to recover upon all or some of the claims asserted therein. The averments in the complaint and the character of the relief sought are the ones to be consulted. Once
and Capas Relay station), to raise funds by the sale of portions of Metro Manila military camps and to apply said funds for the development and conversion to productive vested by the allegations in the complaint, jurisdiction also remains vested irrespective of whether or not the plaintiff is entitled to recover upon all or some of the claims asserted therein.
civilian use of the lands covered under the 1947 Military Bases Agreement between the Philippines and the United States of America, as amended. What determines the jurisdiction of the court is the nature of the action pleaded as appearing from the allegations in the complaint. The averments therein and the character of the relief
It is likewise the declared policy of the Government to enhance the benefits to be derived from said properties in order to promote the economic and social development sought are the ones to be consulted.
of Central Luzon in particular and the country in general. Respondents’ Complaint narrates that they are the duly registered owners of Lot No. 625 of the Limay Cadastre which was covered by TCT No. T-105602. Without their knowledge and
Republic Act No. 7227 goes on further to provide that: consent, the land was divided into several lots under their names through the fraudulent manipulations of Maura. One of the lots was Lot 625-K, which was covered by TCT No. 134785. On
SEC. 4. Purposes of the Conversion Authority. The Conversion authority shall have the following purposes: April 26, 1990, Maura sold the subject lot to Lorna. By virtue of the fictitious sale, TCT No. 134785 was cancelled and TCT No. 134932 was issued in the name of Lorna. Sometime in August
xxx 1990, Lorna sold the lot to petitioner for a consideration in the amount of ₱4,000.00. TCT No. 134932 was later cancelled and TCT No. 137466 was issued in the name of petitioner. Despite
(c) To encourage the active participation of the private sector in transforming the Clark and Subic military reservations and their extensions into other productive uses; demands from the respondents, petitioner refused to surrender possession of the subject property. Respondents were thus constrained to engage the services of a lawyer and incur
The establishment, registration, and operation of respondent Clark Liberty and the other enterprises within the Clark Special Economic Zone are projects (involving the expenses for litigation. Respondents prayed for the RTC (a) to declare TCT No. 137466 null and to revive TCT No. T-105602 which was originally issued and registered in the name of the
private sector) which convert Clark Air Base, a military reservation, into productive uses. In this connection, Section 21 of R.A. No. 7227 provides: respondents; and (b) to order petitioner to pay attorney’s fees in the sum of ₱50,000.00 and litigation expenses of ₱20,000.00, plus cost of suit.
SEC. 21. Injunction and Restraining Order. The implementation of the projects for the conversion into alternative productive uses of the military reservations are urgent and An action "involving title to real property" means that the plaintiff's cause of action is based on a claim that he owns such property or that he has the legal rights to have exclusive control,
necessary and shall not be restrained or enjoined except by an order issued by the Supreme Court of the Philippines. possession, enjoyment, or disposition of the same. Title is the "legal link between (1) a person who owns property and (2) the property itself." "Title" is different from a "certificate of title"
Verily, the Court of Appeals did not err when it dismissed CA-G.R. SP No. 63802 for want of jurisdiction. which is the document of ownership under the Torrens system of registration issued by the government through the Register of Deeds. While title is the claim, right or interest in real property,
a certificate of title is the evidence of such claim.
In the present controversy, before the relief prayed for by the respondents in their complaint can be granted, the issue of who between the two contending parties has the valid title to the
subject lot must first be determined before a determination of who between them is legally entitled to the certificate of title covering the property in question.
Padlan v. Dinglasan From the Complaint, the case filed by respondent is not simply a case for the cancellation of a particular certificate of title and the revival of another. The determination of such issue merely
G.R. No. 180321, March 20, 2013 follows after a court of competent jurisdiction shall have first resolved the matter of who between the conflicting parties is the lawful owner of the subject property and ultimately entitled to its
possession and enjoyment. The action is, therefore, about ascertaining which of these parties is the lawful owner of the subject lot, jurisdiction over which is determined by the assessed
TOPIC: Manner of Acquiring Jurisdiction over subject matter value of such lot.
CASE NATURE: In no uncertain terms, the Court has already held that a complaint must allege the assessed value of the real property subject of the complaint or the interest thereon to determine which
Petition for review on certiorari assailing the Decision dated June 29, 2007 of the Court of Appeals (CA) in CA-G.R. CV No. 86983, and the Resolution dated October 23, 2007 denying court has jurisdiction over the action. In the case at bar, the only basis of valuation of the subject property is the value alleged in the complaint that the lot was sold by Lorna to petitioner in
petitioner's Motion for Reconsideration. the amount of ₱4,000.00. No tax declaration was even presented that would show the valuation of the subject property. In fact, in one of the hearings, respondents’ counsel informed the
FACTS: court that they will present the tax declaration of the property in the next hearing since they have not yet obtained a copy from the Provincial Assessor’s Office. However, they did not present
Elenita Dinglasan (Elenita) was the registered owner of a parcel of land designated as Lot No. 625 of the Limay Cadastre which is covered by Transfer Certificate of Title (TCT) No. T- such copy.
105602, with an aggregate area of 82,972 square meters. While on board a jeepney, Elenita’s mother, Lilia Baluyot (Lilia), had a conversation with one Maura Passion (Maura) regarding the To reiterate, where the ultimate objective of the plaintiffs is to obtain title to real property, it should be filed in the proper court having jurisdiction over the assessed value of the property
sale of the said property. Believing that Maura was a real estate agent, Lilia borrowed the owner’s copy of the TCT from Elenita and gave it to Maura. Maura then subdivided the property into subject thereof. Since the amount alleged in the Complaint by respondents for the disputed lot is only ₱4,000.00, the MTC and not the RTC has jurisdiction over the action. Therefore, all
several lots from Lot No. 625-A to Lot No. 625-O, under the name of Elenita and her husband Felicisimo Dinglasan (Felicisimo). proceedings in the RTC are null and void.
Cause of dispute:
Through a falsified deed of sale bearing the forged signature of Elenita and her husband Felicisimo, Maura was able to sell the lots to different buyers. On April 26, 1990, Maura sold Lot No. Guy v. Court of Appeals
625-K to one Lorna Ong (Lorna), who later caused the issuance of TCT No. 134932 for the subject property under her name. A few months later, or sometime in August 1990, Lorna sold the G.R. No. 165849, December 10, 2007
lot to petitioner Editha Padlan for ₱4,000.00. Thus, TCT No. 134932 was cancelled and TCT No. 137466 was issued in the name of petitioner.
TOPIC: Manner of Acquiring Jurisdiction over subject matter
Plaintiff’s argument: CASE NATURE:
Lacks of jurisdiction over her person Five (5) consolidated cases which stemmed from Civil Case No. 04-109444 filed with the Regional Trial Court (RTC), Branch 24, Manila, subsequently re-raffled to Branch 46 and eventually
Petitioner insists that summons was not validly served upon her, considering that at the time summons was served, she was residing in Japan. Petitioner contends that pursuant to Section to Branch 25
15, Rule 14 of the Rules of Civil Procedure, when the defendant does not reside in the Philippines and the subject of the action is property within the Philippines of the defendant, service FACTS:
may be effected out of the Philippines by personal service or by publication in a newspaper of general circulation. In this case, summons was served only by substituted service to her The instant controversies arose from a family dispute. Gilbert Guy is the son of Francisco and Simny Guy. Geraldine, Gladys and Grace are his sisters. The family feud involves the
mother. Hence, the court did not acquire jurisdiction over her person. ownership and control of 20,160 shares of stock of Northern Islands Co., Inc. (Northern Islands) engaged in the manufacture, distribution, and sales of various home appliances bearing the
3-D trademark.
Lacks of jurisdiction of the subject matter Simny and her daughters Geraldine, Gladys and Grace, as well as Northern Islands and Emilia Tabugadir, have been impleaded as respondents in the above-entitled
Also, petitioner posits that the court lacks jurisdiction of the subject matter, considering that from the complaint, it can be inferred that the value of the property was only ₱4,000.00, which cases. Northern Islands is a family-owned corporation organized in 1957 by spouses Francisco and respondent Simny Guy. In November 1986, they incorporated Lincoln Continental
was the amount alleged by respondents that the property was sold to petitioner by Lorna Development Corporation, Inc. (Lincoln Continental) as a holding company of the 50% shares of stock of Northern Islands in trust for their three (3) daughters, respondents Geraldine,
Plaintiff’s prayer: Gladys and Grace. Sometime in December 1986, upon instruction of spouses Guy, Atty. Andres Gatmaitan, president of Lincoln Continental, indorsed in blank Stock Certificate No. 132
Finally, petitioner stresses that she was a buyer in good faith. It was Maura who defrauded the respondents by selling the property to Lorna without their authority. (covering 8,400 shares) and Stock Certificate No. 133 (covering 11,760 shares) and delivered them to Simny.
Defendant’s answer:
CA was correct in ruling in their favor. Cause of dispute:
RTC: In 1984, spouses Guy found that their son Gilbert has been disposing of the assets of their corporations without authority. In order to protect the assets of Northern Islands, Simny
Padlan is buyer in good faith surrendered Stock Certificate Nos. 132 and 133 to Emilia Tabugadir, an officer of Northern Islands. The 20,160 shares covered by the two Stock Certificates were then registered in the
names of respondent sisters, thus enabling them to assume an active role in the management of Northern Islands.
CA: On January 27, 2004, during a special meeting of the stockholders of Northern Islands, Simny was elected President; Grace as Vice-President for Finance; Geraldine as Corporate
Reverse the decision of RTC and ordered the cancellation of TCT Treasurer; and Gladys as Corporate Secretary. Gilbert retained his position as Executive Vice President. This development started the warfare between Gilbert and his sisters.
Basis/reason
The CA found that petitioner purchased the property in bad faith from Lorna. The CA opined that although a purchaser is not expected to go beyond the title, based on the
circumstances surrounding the sale, petitioner should have conducted further inquiry before buying the disputed property. The fact that Lorna bought a 5,000-square-meter On March 18, 2004, Lincoln Continental filed with the RTC, Branch 24, Manila a Complaint for Annulment of the Transfer of Shares of Stock against respondents, docketed as Civil Case No.
property for only ₱4,000.00 and selling it after four months for the same amount should have put petitioner on guard. With the submission of the Judgment in Criminal Case
No. 4326 rendered by the RTC, Branch 2, Balanga, Bataan, entitled People of the Philippines v. Maura Passion10 and the testimonies of respondents, the CA concluded
that respondents sufficiently established that TCT No. 134932 issued in the name of Lorna and TCT No. 137466 issued in the name of petitioner were fraudulently issued 04-109444. The complaint basically alleges that Lincoln Continental owns 20,160 shares of stock of Northern Islands; and that respondents, in order to oust Gilbert from the management
and, therefore, null and void.
Motion for recon CA:
Denied the motion of Northern Islands, falsely transferred the said shares of stock in respondent sisters names. Lincoln Continental then prayed for an award of damages and that the management of Northern
ISSUE:
Whether or not the Honorable Court has jurisdiction over the person of the petitioner
Islands be restored to Gilbert. Lincoln also prayed for the issuance of a temporary restraining order (TRO) and a writ of preliminary mandatory injunction to prohibit respondents from
Whether or not the Honorable Court has jurisdiction over the subject matter of the case

RULING: exercising any right of ownership over the shares.


Plaintiff’s argument: The NPUM Executive Committee and the Board of Trustees decided to immediately request the services of the General Conference Auditing Service (GCAS) to determine the veracity of the
audit findings. Accordingly, GCAS auditors worked in the campus from December 4 to December 20, 2002 to review the petitioners transactions during the period from April 2002 to October
2002. On December 20, 2002, CGAS auditors reported the results of their review, and submitted their observations and recommendations to the Board of Trustees.
On the question of forum shopping, petitioners Gilbert and Lincoln Continental contend that the acts of respondents in filing a petition for certiorari and mandamus in CA-G.R. SP No. 85069 Upon receipt of the CGAS report that confirmed the initial findings of the auditors on January 8, 2003, the NPUM informed the petitioner of the findings and required him to explain.
On January 15, 2003, Chairman Dayson and the NPUM Treasurer likewise informed the petitioner inside the NPUM office on the findings of the auditors in the presence of the AUP Vice-
President for Financial Affairs, and reminded him of the possible consequences should he fail to satisfactorily explain the irregularities cited in the report. He replied that he had already
and withdrawing the same and their subsequent filing of a petition for certiorari in CA-G.R. SP No. 87104 constitute forum shopping; that respondents withdrew their petition in CA-G.R. SP prepared his written explanation.
The Board of Trustees set a special meeting at 2 p.m. on January 22, 2003. Being the Secretary, the petitioner himself prepared the agenda and included an item on his case. In that
meeting, he provided copies of the auditors report and his answers to the members of the Board of Trustees.After hearing his explanations and oral answers to the questions raised on
No. 85069 after the Tenth Division issued a Resolution dated October 20, 2004 denying their application for a writ of preliminary injunction; that they then filed an identical petition in CA-G.R. issues arising from the report, the members of the Board of Trustees requested him to leave to allow them to analyze and evaluate the report and his answers. Despite a long and careful
deliberation, however, the members of the Board of Trustees decided to adjourn that night and to set another meeting in the following week considering that the meeting had not been
specifically called for the purpose of deciding his case. The adjournment would also allow the Board of Trustees more time to ponder on the commensurate disciplinary measure to be meted
on him.
SP No. 87104 seeking the same relief alleged in their petition in CA-G.R. SP No. 85069; and that by taking cognizance of the petition in CA-G.R. SP No. 87104, instead of dismissing it
On January 23, 2003, Chairman Dayson notified the petitioner in writing that the Board of Trustees would hold in abeyance its deliberation on his answer to the auditors report and would
meet again at 10:00 a.m. on January 27, 2003. Chairman Dayson indicated that some sectors in the campus had not been properly represented in the January 22, 2003 special meeting,
and requested the petitioner as Secretary to ensure that all sectors are duly represented in the next meeting of the Board of Trustees.
outright on the ground of forum shopping, the Court of Appeals committed grave abuse of discretion tantamount to lack or excess of jurisdiction. In the January 27, 2003 special meeting, the petitioner sent a letter to the Board of Trustees. The members, by secret ballot, voted to remove him as President because of his serious
Plaintiff’s prayer: violations of fundamental rules and procedures in the disbursement and use of funds as revealed by the special audit; to appoint an interim committee consisting of three members to
assume the powers and functions of the President; and to recommend him to the NPUM for consideration as Associate Director for Secondary Education.
On January 28, 2003, the petitioner was handed inside the NPUM office a letter, together with a copy of the minutes of the special meeting held the previous day. In turn, he handed to
. The petitioners in CA-G.R. SP No. 85069 prayed that the following Orders be set aside: Chairman Dayson a letter requesting two weeks within which to seek a reconsideration, stating that he needed time to obtain supporting documents because he was then attending to his
dying mother.
In the evening of January 28, 2003, the Board of Trustees, most of whose members had not yet left Cavite, reconvened to consider and decide the petitioners request for reconsideration.
(1) the Order of inhibition dated June 22, 2004 issued by the presiding judge of the RTC of Manila, Branch 24; and During the meeting, he made an emotional appeal to allow him to continue as President, promising to immediately vacate his office should he again commit any of the irregularities cited in
the auditors report. He added that should the Board of Trustees not favor his appeal, he would settle for a retirement package for him and his wife and would leave the church.
The Board of Trustees denied the petitioners request for reconsideration because his reasons were not meritorious. Board Member Elizabeth Role served the notice of the denial on him the
(2) the Order dated July 12, 2004 issued by Branch 46 setting Gilberts application for preliminary injunction for hearing. next day, but he refused to receive the notice, simply saying Alam ko na yan.
The petitioner later obtained a copy of the inter-school memorandum dated January 31, 2003 informing AUP students, staff, and faculty members about his relief as President and the
appointment of an interim committee to assume the powers and duties of the President.
On February 4, 2003, the petitioner brought his suit for injunction and damages in the RTC, with prayer for the issuance of a temporary restraining order (TRO), impleading AUP and its
RTC: Board of Trustees, represented by Chairman Dayson, and the interim committee. His complaint alleged that the Board of Trustees had relieved him as President without valid grounds
Complaint and the Complaint-in-Intervention are hereby DISMISSED despite his five-year term; that the Board of Trustees had thereby acted in bad faith; and that his being denied ample and reasonable time to present his evidence deprived him of his right to
Basis/reason due process.
The trial court held that Civil Case No. 04-109444 is a baseless and an unwarranted suit among family members; that based on the evidence, Gilbert was only entrusted to The suit being intra-corporate and summary in nature, the application for TRO was heard by means of affidavits. In the hearing of February 7, 2003, the parties agreed not to harass each
hold the disputed shares of stock in his name for the benefit of the other family members; and that it was only when Gilbert started to dispose of the assets of the familys other. The RTC used the mutual agreement as its basis to issue a status quo order on February 11, 2003.
corporations without their knowledge that respondent sisters caused the registration of the shares in their respective names In their answer with counterclaim, the respondents denied the allegations of the petitioner, and averred that he had been validly removed for cause; and that he had been granted ample
opportunity to be heard in his defense.
CA: Cause of dispute:
Affirmed the decision of RTC The dispute centers on whether the removal of the petitioner as President of respondent Adventist University of the Philippines (AUP) was valid, and whether his term in that office was five
ISSUE: years, as he insists, or only two years, as AUP insists.
Whether respondents are guilty of forum shopping and that the Court of Appeals committed grave abuse of discretion tantamount to lack or excess of jurisdiction. Plaintiff’s argument:
RULING: THE COURT OF APPPEALS HAS DECIDED CONTRARY TO LAW AND JURISPRUDENCE WHEN IT RULED THAT THE EXTRAORDINARY WRIT OF CERTIORARI APPLIED IN THE
No grave abuse of discretion amounting to lack or excess of jurisdiction CASE AT BAR.
A party is guilty of forum shopping when he repetitively avails of several judicial remedies in different courts, simultaneously or successively, all substantially founded on the same THE COURT OF APPEALS DECIDED A QUESTION OF SUBSTANCE IN A WAY NOT IN ACCORD WITH THE ESTABLISHED LAW AND JURISPRUDENCE THAT ADMISSIONS, VERBAL
transactions and the same essential facts and circumstances, and all raising substantially the same issues either pending in, or already resolved adversely by some other court. [6] It is OR WRITTEN, MADE BY A PARTY IN THE COURSE OF THE PROCEEDINGS IN THE SAME CASE, DOES NOT REQUIRE PROOF, BY REQUIRING PETITIONER BARAYUGA TO
prohibited by Section 5, Rule 7 of the 1997 Rules of Civil Procedure, as amended, which provides: PRESENT EVIDENCE THAT HIS TERM AS PRESIDENT OF AUP IS FOR FIVE (5) YEARS.
SECTION 5. Certification against forum shopping. The plaintiff or principal party shall certify under oath in the complaint or other initiatory pleading asserting a claim for relief, or in a sworn THE COURT OF APPEALS DECIDED A QUESTION OF SUBSTANCE IN A WAY NOT IN ACCORD WITH LAW AND ESTABLISHED FACTS WHEN IT RULED THAT PETITIONER
certification annexed thereto and simultaneously filed therewith: (a) that he has not theretofore commenced any action or filed any other claim involving the same issues in any court, BARAYUGA HAS ONLY A TERM OF TWO (2) YEARS INSTEAD OF FIVE (5) YEARS AS CLEARLY ADMITTED BY PRIVATE RESPONDENT AUP IN ITS ANSWER.
tribunal, or quasi-judicial agency and, to the best of his knowledge, no such other action or claim is pending therein; (b) if there is such other pending action or claim, a complete statement of THE COURT OF APPEALS DECIDED A QUESTION OF SUBSTANCE IN A WAY NOT IN ACCORD WITH LAW AND JURISPRUDENCE BY SOLELY RELYING ON THE CASE
the present status thereof; and (c) if he should thereafter learn that the same or similar action has been filed or is pending, he shall report that fact within five (5) days therefrom to the court OF NATIONAL POWER CORPORATION v. COURT OF APPEALS, WHICH INVOLVE FACTS DIFFERENT FROM THE PRESENT CASE.
wherein his aforesaid complaint or initiatory pleading has been filed. THE COURT OF APPEALS DECIDED A QUESTION OF SUBSTANCE IN A WAY NOT IN ACCORD WITH LAW AND ESTABLISHED FACTS WHEN IT UNJUSTIFIABLY ALLOWED THE
Failure to comply with the foregoing requirements shall not be curable by mere amendment of the complaint or other initiatory pleading but shall be cause for the dismissal of the case WAIVER OF NOTICE FOR THE SPECIAL MEETING OF THE BOARD OF TRUSTEES.
without prejudice, unless otherwise provided, upon motion and hearing. The submission of a false certification or non-compliance with any of the undertakings therein shall constitute indirect THE COURT OF APPEALS DECIDED A QUESTION OF SUBSTANCE IN A WAY NOT IN ACCORD WITH LAW AND ESTABLISHED FACTS WHEN IT ERRONEOUSLY CONCLUDED
contempt of court, without prejudice to the corresponding administrative and criminal actions. If the acts of the party or his counsel clearly constitute willful and deliberate forum shopping, the THAT PETITIONER BARAYUGA WAS MERELY OCCUPYING THE POSITION OF AUP PRESIDENT IN A HOLD-OVER CAPACITY.
same shall be ground for summary dismissal with prejudice and shall constitute direct contempt, as well as a cause for administrative sanctions.
Forum shopping is condemned because it unnecessarily burdens our courts with heavy caseloads, unduly taxes the manpower and financial resources of the judiciary and trifles with and
mocks judicial processes, thereby affecting the efficient administration of justice. [7] The primary evil sought to be proscribed by the prohibition against forum shopping is, however, the
possibility of conflicting decisions being rendered by the different courts and/or administrative agencies upon the same issues.[8] Defendant’s answer:
Forum shopping may only exist where the elements of litis pendentia are present or where a final judgment in one case will amount to res judicata in the other.[9] Litis pendentia as a ground PETITIONER IS NOT AN ELECTED TRUSTEE OF THE AUP BOARD, NOR WAS (HE) ELECTED AS PRESIDENT, AND AS SUCH, HE CAN CLAIM NO RIGHT TO THE AUP
for dismissing a civil action is that situation wherein another action is pending between the same parties for the same cause of action, such that the second action is unnecessary and PRESIDENCY, BEING TWICE DISQUALIFIED BY LAW, WHICH RENDERS MOOT AND ACAMEDIC ALL OF THE ARGUMENTS IN THIS PETITION.
vexatious. The elements of litis pendentia are as follows: (a) identity of parties, or at least such as representing the same interest in both actions; (b) identity of rights asserted and the relief EVEN IF WE FALSELY ASSUME EX GRATIA THAT PETITIONER IS AN ELECTED TRUSTEE AND ELECTED PRESIDENT, THE TWO (2) YEAR TERM PROVIDED IN AUPS BY-LAWS
prayed for, the relief being founded on the same facts; and (c) the identity of the two cases such that judgment in one, regardless of which party is successful, would amount to res judicata in REQUIRED BY THE CORPORATION CODE AND APPROVED BY THE SEC IS WHAT GOVERNS THE INTRA-CORPORATE CONTROVERSY, THE AUPS ADMISSION IN ITS ANSWER
the other.[10] From the foregoing, it is clear that sans litis pendentia or res judicata, there can be no forum shopping. THAT HE HAS A FIVE (5) YEAR TERM BASED ON HIS INVOKED SAMPLE CONSTITUTION, BY-LAWS AND POLICY OF THE SEVENTH DAY ADVENTIST NOTWITHSTANDING.
While the first element of litis pendentia identity of parties is present in both CA-G.R. SP No. 85069 and CA-G.R. SP No. 87104, however, the second element, does not exist. The petitioners PURSUANT TO THE RULES AND SETTLED JURISPRUDENCE, THE ADMISSION IN THE ANSWER IS NOT EVEN PREJUDICIAL AT ALL.
in CA-G.R. SP No. 85069 prayed that the following Orders be set aside: EVEN IF WE FALSELY ASSUME, JUST FOR THE SAKE OF ARGUMENT, THAT THE PETITIONER HAD A FIVE (5) YEAR TERM AS UNIVERSITY PRESIDENT, HE WAS NONETHELESS
(1) the Order of inhibition dated June 22, 2004 issued by the presiding judge of the RTC of Manila, Branch 24; and VALIDLY TERMINATED FOR LOSS OF CONFIDENCE, GIVEN THE NUMEROUS ADMITTED ANOMALIES HE COMMITTED.
(2) the Order dated July 12, 2004 issued by Branch 46 setting Gilberts application for preliminary injunction for hearing. PETITIONER CANNOT COMPLAIN THAT NOTICES OF THE BOARD MEETING WERE NOT SENT TO ALL THE TWENTY FIVE (25) TRUSTEES OF THE AUP BOARD, SINCE: [1] AS
THE AUP SECRETARY, IT WAS HE WHO HAD THE DUTY TO SEND THE NOTICES; [2] WORSE, HE ATTENDED AND EXHAUSTIVELY DEFENDED HIS WRITTEN ANSWER IN THE
In their petition in CA-G.R. SP No. 87104, respondents prayed for the annulment of the writ of preliminary injunction issued by the RTC, Branch 46 after the expiration of the TRO issued by AUP BOARD OF TRUSTEES MEETING, THUS, WAIVING ANY NOTICE OBJECTION; [3] WORST OF ALL, HIS AFTERTHOUGHT OBJECTION IS DECEPTIVELY FALSE IN FACT.
the Tenth Division of the Court of Appeals. Evidently, this relief is not identical with the relief sought by respondents in CA-G.R. SP No. 85069. Clearly, the second element of litis
pendentia the identity of reliefs sought - is lacking in the two petitions filed by respondents with the appellate court. Thus, we rule that no grave abuse of discretion amounting to lack or
excess of jurisdiction may be attributed to the Court of Appeals (Eighth Division) for giving due course to respondents petition in CA-G.R. SP No. 87104. RTC:
Granting Barayuga application for the writ of preliminary injunction
Basis/reason

Barayuga v. Adventist University After further hearing, the RTC issued on April 25, 2003 its controversial order, granting the petitioners application for a writ of preliminary injunction. It thereby resolved three

G.R. No. 168008, August 17, 2011


TOPIC: Manner of Acquiring Jurisdiction over subject matter issues, namely: (a) whether the special board meetings were valid; (b) whether the conflict-of-interest provision in the By-Laws and Working Policy was violated; and (c)
CASE NATURE:
Petition for review on certiorari
FACTS:
whether the petitioner was denied due process. It found for the petitioner upon all the issues. On the first issue, it held that there was neither a written request made by any
AUP, a non-stock and non-profit domestic educational institution incorporated under Philippine laws on March 3, 1932, was directly under the North Philippine Union Mission (NPUM) of the
Southern Asia Pacific Division of the Seventh Day Adventists. During the 3rd Quinquennial Session of the General Conference of Seventh Day Adventists held from November 27, 2000 to
December 1, 2000, the NPUM Executive Committee elected the members of the Board of Trustees of AUP, including the Chairman and the Secretary. Respondent Nestor D. Dayson was
elected Chairman while the petitioner was chosen Secretary. two members of the Board of Trustees nor proper notices sent to the members as required by AUPs By-Laws, which omissions, being patent defects, tainted the special
On January 23, 2001, almost two months following the conclusion of the 3rd Quinquennial Session, the Board of Trustees appointed the petitioner President of AUP. During his tenure, or
from November 11 to November 13, 2002, a group from the NPUM conducted an external performance audit. The audit revealed the petitioners autocratic management style, like making
major decisions without the approval or recommendation of the proper committees, including the Finance Committee; and that he had himself done the canvassing and purchasing of board meetings with nullity. Anent the second issue, it ruled that the purchase of coco lumber from his balae (i.e., mother-in-law of his son) was not covered by the conflict-
materials and made withdrawals and reimbursements for expenses without valid supporting receipts and without the approval of the Finance Committee. The audit concluded that he had
committed serious violations of fundamental rules and procedure in the disbursement and use of funds.
of-interest provision, for AUPs Model Statement of Acceptance form mentioned only the members of the immediate family and did not extend to the relationship between him
3.That to the best of my knowledge and belief, there is no mortgage or encumbrance of any kind whatsoever affecting said land, nor another person having any estate or interest therein,
and his balae. On the third issue, it concluded that he was deprived of due process when the Board of Trustees refused to grant his motion for reconsideration and his legal or equitable, in possession, remainder, reversion or expectancy;
4.That the land was acquired by possessory title in open, continuous, adverse occupation and possession in the concept of owner for more than thirty years since 1960 (please refer to
ANNEX A);
request for additional time to produce his evidence, and instead immediately implemented its decision by relieving him from his position without according him the treatment 5. That the land is adjoined by the following:
NorthWest
NorthEast
befitting a university President. SouthEast
All along line 1-2-3-4-5-6-7-8-9-10 by Flores Avenue, City Road and the Dumaguete Port Road
SouthWest along line 10-1 by Plan Msi-V-20453
8. That the land included is bounded on the West by Flores Avenue and on the North by the City Road, all public highways and on the East by the Dumaguete Port Road, a private road
made part of the Port Zone.
CA:
Defendant’s answer:
Nullified and set aside the order of the RTC
Basis/reason
The Republic of the Philippines, represented by the Director of Lands, and respondent, represented by the Office of the Government Corporate Counsel, filed separate Oppositions to the
The CA rendered its decision nullifying the RTCs writ of preliminary injunction. It rejected the petitioners argument that Article IV, Section 3 of AUPs Constitution and By-Laws
application for registration of petitioner. Both the Republic and respondent averred that petitioner may not register the subject property in its name since petitioner had never been in open,
and Working Policy of the Conference provided a five-year term for him, because the provision was inexistent. It ruled that the petitioners term of office had expired on
continuous, exclusive, and notorious possession of the said property for at least 30 years immediately preceding the filing of the application; and the subject property remains to be a portion
January 22, 2003, or two years from his appointment, based on AUPs amended By-Laws; that, consequently, he had been a mere de facto officer appointed by the members
of the public domain which belongs to the Republic.
of the Board of Trustees; and that he held no legal right warranting the issuance of the writ of preliminary injunction.
RTC:

Dismiss the application for original registration

Basis/reason
The CA declared that the rule on judicial admissions admitted of exceptions, as held in National Power Corporation v. Court of Appeals, where the Court held that admissions The Court agrees with [herein respondent] Philippine Ports Authority that the basis of the [herein petitioners] application for original registration of the subject
lot is Section 14 of the Presidential Decree No. 1529, otherwise known as the Property Registration Decree. A circumspect scrutiny of said Section readily
shows that it refers to alienable and disposable lands of the public domain as proper subjects of registration, provided the applicant has met the other
were not evidence that prevailed over documentary proof; that the petitioners being able to answer the results of the special audit point-by-point belied his allegation of requirements such as open, continuous, exclusive and notorious possession for at least thirty (30) years under a bona fide claim of ownership.

It having been shown by [petitioners] own evidence that the lot subject of the application for original registration is a foreshore land, and therefore not
denial of due process; that AUP was the party that stood to be injured by the issuance of the injunctive writ in the form of a demoralized administration, studentry, faculty and
registerable (Dizon, et al. vs. Bayona, et al., 98 SCRA 942, 944), the application must be denied.

Again as correctly argued by [respondent], [petitioners] reliance on Republic Act 1899 which authorizes all municipalities and chartered cities to undertake
staff, sullied reputation, and dishonest leadership; and that the assailed RTC order sowed confusion and chaos because the RTC thereby chose to subordinate the interest and carry out the reclamation by dredging, filling or other means of any foreshore lands bordering them and which confers ownership on them of the lands
so reclaimed, is misplaced, as such has never been alleged in the application. It is fundamental that a party cannot prove what it has not alleged in his
complaint or application, as in this case.
of the entire AUP community to that of the petitioner who had been deemed not to have satisfied the highest ideals required of his office.
The admission by Engr. Dorado that there is no formal declaration from the executive branch of government or law passed by Congress that the land in
ISSUE: question is no longer needed for public use or special industries x x x further militates against the application.
Wheter the CA correctly ruled that the petitioner had no legal right to the position of President of AUP that could be protected by the injunctive writ issued by the RTC.
RULING: Moreover, the authority granted to municipalities and chartered cities to undertake and carry out at their own expense the reclamation by dredging, filling, or
The removal of the petitioner as President of AUP, being made in accordance with the AUP Amended By-Laws, was valid. With that, our going into the other issues becomes unnecessary. other means, of any foreshore lands bordering them is for the purpose of establishing, providing, constructing, maintaining, and repairing proper and
We conclude that the order of the RTC granting his application for the writ of preliminary injunction was tainted with manifestly grave abuse of discretion; that the CA correctly nullified and adequate docking and harbor facilities as such municipalities and chartered cities may determine in consultation with the Secretary of Finance and the
set aside the order; and that his claim for damages, being bereft of factual and legal warrant, should be dismissed. Secretary of Public Works and Communications.

City of Dumaguete v. Philippine Ports Authority By its own evidence, [petitioner] has utilized the subject property allegedly reclaimed by it as Office of the City Engineer and not as docking and harboring
G.R. No. 168973, August 24, 2011 facilities. [Petitioner] has failed to show that such reclamation was undertaken by it in consultation with the Secretary of Finance and the Secretary of Public Works and
TOPIC: Manner of Acquiring Jurisdiction over subject matter Communications.
CASE NATURE: Motion for reconsideration RTC:
Denied the motion for reconsideration
Basis/reason
Petition for Review under Rule 45 of the Rules of Court assailing the Decision dated March 4, 2005 and Resolution dated June 6, 2005 of the Court Appeals in CA-G.R. SP No. 64379, which

A thorough review and perusal of the disputed order dated September 7, 2000 and December 7, 2000, whereby this Court dismissed [petitioners] petition for registration of
granted the Petition for Certiorari and Prohibition of respondent Philippine Ports Authority and set aside the Orders dated December 7, 2000 and February 20, 2001 of the Regional Trial

Lot No. 1, Dumaguete Cadastre, and later set aside the Order of September 7, 2000, shows that there was honest mistake in declaring said lot 1, as a shoreline. Indeed, the
Court (RTC), Branch 44 of the City of Dumaguete in LRC Case No. N-201.

adjoining lots are already titled and bounded by a City Road. It is not bounded by a sea. The Court wants to correct this error in its findings on the September 7, 2000 Order,

FACTS: that Lot No. 1 is situated on the shoreline of Dumaguete City. The Court simply committed an oversight on the petitioners evidence that the lot in question is a foreshore land

On October 14, 1998, petitioner City of Dumaguete, through Mayor Felipe Antonio B. Remollo (Remollo), filed before the RTC an Application for Original Registration of Title over a parcel of x x x when in fact it is not. And it is for this reason that the court reconsidered and set aside said September 7, 2000 Order, to correct the same while it is true that said

land with improvements, located at Barangay Looc, City of Dumaguete (subject property), under the Property Registration Decree. The application was docketed as LRC Case No. N-201.
September 7, 2000 Order had attained its finality, yet this Court cannot in conscience allow injustice to perpetuate in this case and that hearing on the merits must proceed to
Cause of dispute:
The Republic and the respondent opposed the application, as it remains in the public domain
Plaintiff’s argument: determine the legality and truthfulness of its application for registration of title.
Petitioner alleged in support of its application:
1. That the applicant, City of Dumaguete through its Honorable Mayor Felipe Antonio B. Remollo, is the owner of the land subject of this application with all improvements and buildings CA:
comprising the Engineers Compound where it is now situated and has been in continuous occupation and possession of the same for more than 30 years or from the year 1960 (Affidavit of Set aside the RTC Order
Ownership executed by Felipe Antonio G. Remollo, the City Mayor, dated August 21, 1998 herein attached as ANNEX A). The said land consist of 5,410 square meters and is situated and Basis/reason
bounded and described as shown on the plan (true and photostatic copies of the original plan marked Psu-07-006805 approved by the Regional Technical Director of the [Department of
Environment and Natural Resources] DENR, Regional Office, Cebu City herein attached as ANNEX B) and technical descriptions attached hereto (technical description attached as ANNEX
C) and made a part hereof;
2. That said land at the last assessment for taxation was assessed at P676,250, Philippine currency, with market value of P1,352,500.00, Philippine currency. (Declaration of Real Property
with the assessed and market values attached as ANNEX D);
one; or, third, from the corresponding tax declaration of the real property. In this case, the value of the property cannot be determined using the first method, because the records are bereft
of any affidavit executed by respondent as to the value of the property. Likewise, valuation cannot be done through the second method, because this method finds application only where
The Court of Appeals, in its Decision dated March 4, 2005, found merit in the Petition of respondent and set aside the RTC Orders dated December 7, 2000 and February there are multiple claimants who agree on and make a joint submission as to the value of the property. Here, only respondent Bantigue Point Development Corporation claims the property.
The value of the property must therefore be ascertained with reference to the corresponding Tax Declarations submitted by respondent Corporation together with its application for
registration. From the records, we find that the assessed value of the property is ₱4,330, ₱1,920 and ₱8,670, or a total assessed value of ₱14,920 for the entire property. Based on these
20, 2001. The appellate court, in its Resolution dated June 6, 2005, denied the Motion for Reconsideration of petitioner Tax Declarations, it is evident that the total value of the land in question does not exceed ₱100,000. Clearly, the MTC may exercise its delegated jurisdiction under the Judiciary
Reorganization Act, as amended.
ISSUE:
Wether RTC has jurisdiction over the subject matter People of the Philippines v. Hon. Garfin
RULING: G.R. No. 153176, March 29, 2004

TOPIC: Manner of Acquiring Jurisdiction: Over the Case


RTC has jurisdiction over the subject matter
CASE NATURE: SPECIAL CIVIL ACTION in the Supreme Court. Certiorari and Mandamus.

Jurisdiction over the subject matter of a case is conferred by law and determined by the allegations in the complaint which comprise a concise statement of the ultimate facts constituting the FACTS: private respondent was charged with violation of RA No. 8282, otherwise known as the "Social Security Act." The corresponding information was filed by Prosecutor Tolentino.
Garfin was arraigned and pleaded not guilty. Three days thereafter, the accused filed a motion to dismiss on the ground that the information was filed without the prior written authority or
approval of the city prosecutor as required under Section 4, Rule 112 of the Revised Rules of Court. The Court thus the case dismissed for lack of jurisdiction.
plaintiff's cause of action.
CAUSE OF THE DISPUTE: the case of People vs. Serafin Saballegue was dismissed for lack of jurisdiction. Hence, this petition

Once vested by the allegations in the complaint, jurisdiction also remains vested irrespective of whether or not the plaintiff is entitled to recover upon all or some of the claims asserted ARGUMENTS

PLAINTIFF: In its MR, the People contends that Prosecutor Tolentino, having been duly designated to assist the City Prosecutor in the investigation and prosecution of all SSS cases by the
therein. Regional State prosecutor as alter ego of the Secretary of Justice in Region V, then that authority may be given to him other than the City Prosecutor.

Plaintiff's prayer: Petitioner seeks to declare as null and void the Orders issued by the RTC which dismissed for lack of jurisdiction the case of People vs. Serafin Saballegue and denied
petitioner’s motion for reconsideration.
Republic v. Bantigue Point Development
G.R. No. 162322, March 14, 2012 PRIVATE RESPONDENT'S ANSWER: 1) the instant petition was filed out of time; 2) the special State Prosecutor is only authorized to conduct preliminary investigation and prosecution of
TOPIC: Manner of Acquiring Jurisdiction over subject matter SSS cases and not to sign the information; and 3) the City Prosecutor did not expressly inhibit himself from handling SSS cases nor signing the information.
CASE NATURE:
Rule 45 Petition for review requires this Court to address the issue of the proper scope of the delegated jurisdiction of municipal trial courts in land registration cases RTC: dismissed for lack of jurisdiction the case of People vs. Serafin Saballegue and denied petitioner’s motion for reconsideration.
FACTS:
On 17 July 1997, respondent Bantigue Point Development Corporation filed with the Regional Trial Court (RTC) of Rosario, Batangas an application for original registration of title over a CA: X
parcel of land with an assessed value of ₱4,330, ₱1,920 and ₱8,670, or a total assessed value of ₱14,920 for the entire property, more particularly described as Lot 8060 of Cad 453-D, San
Juan Cadastre, with an area of more or less 10,732 square meters, located at Barangay Barualte, San Juan, Batangas. ISSUE: Whether or not the criminal case was properly dismissed for lack of jurisdiction
On 18 July 1997, the RTC issued an Order setting the case for initial hearing on 22 October 1997. On 7 August 1997, it issued a second Order setting the initial hearing on 4 November
1997. RULING: Yes. In sum, we hold that, in the absence of a directive from the Secretary of Justice designating State Prosecutor Tolentino as Special Prosecutor for SSS cases or a prior written
Petitioner Republic filed its Opposition to the application for registration on 8 January 1998 while the records were still with the RTC. approval of the information by the provincial or city prosecutor, the information in Criminal Case No. RTC 2001-0597 was filed by an officer without authority to file the same. As this infirmity
Cause of dispute: in the information constitutes a jurisdictional defect that cannot be cured, the respondent judge did not err in dismissing the case for lack of jurisdiction.
Land registration cases
Plaintiff’s argument: Discussion: After considering the respective arguments raised by the parties, the Court believes and so resolves that the Information has not been filed in accordance with Section 4, par. 3
First, petitioner argued that the lower court failed to acquire jurisdiction over the application, because the RTC set the date and hour of the initial hearing beyond the 90-day period provided of Rule 112 of the 2000 Rules on Criminal Procedure, thus:
under the Property Registration Decree ‘Rule 112, Section 4 x x x x x x
Second, petitioner contended that since the selling price of the property based on the Deed of Sale annexed to respondents application for original registration was ₱160,000, the MTC did No complaint or information may be filed or dismissed by an investigating prosecutor without the prior written authority or approval of the provincial or city prosecutor or chief state prosecutor
not have jurisdiction over the case. Under Section 34 of the Judiciary Reorganization Act, as amended, the MTCs delegated jurisdiction to try cadastral and land registration cases is limited or the Ombudsman or his deputy.’
to lands, the value of which should not exceed ₱100,000. Expresio unius est exclusio alterius.
RTC: The Information will readily show that it has not complied with this rule as it has not been approved by the City Prosecutor.
RTC Clerk of Court transmitted motu proprio the records of the case to the MTC
Basis/reason The Regional State Prosecutor is not the alter ego of the Secretary of Justice but a mere subordinate official and if ever the former files cases, it is by virtue of a delegated authority by the
The assessed value of the property was allegedly less than ₱100,000. Secretary of Justice. Potestas delegada non potesta delegare (sic) – what has been delegated cannot be redelegated. The Regional State Prosecutor is clearly vested only with the power
of administrative supervision. As administrative supervisor, he has no power to direct the city and provincial prosecutors to inhibit from handling certain cases.

Private respondent and the OSG take the position that the lack of prior authority or approval by the city or provincial prosecutor or chief state prosecutor is an infirmity in the information that
MTC: prevented the court from acquiring jurisdiction over the case. Since lack of jurisdiction is a defect that may be raised as an objection anytime even after arraignment, the respondent judge
Awarded the land to Bantigue Point Development did not err in granting the motion to dismiss based on this ground. As basis, they cite the case of Villa v. Ibañez, et al.[40] where we held, viz:
Basis/reason The defendant had pleaded to an information before he filed a motion to quash, and it is contended that by his plea he waived all objections to the informations. The contention is correct as
MTC entered an Order of General Default and commenced with the reception of evidence. Among the documents presented by respondent in support of its application are far as formal objections to the pleadings are concerned. But by clear implication, if not by express provision of section 10 of Rule 113 of the Rules of Court (now Section 9 of Rule 117), and
Tax Declarations, a Deed of Absolute Sale in its favor, and a Certification from the Department of Environment and Natural Resources (DENR) Community Environment and by a long line of uniform decisions, questions of want of jurisdiction may be raised at any stage of the proceeding. Now, the objection to the respondent’s actuations goes to the very
Natural Resources Office (CENRO) of Batangas City that the lot in question is within the alienable and disposable zone. Thereafter, it awarded the land to respondent foundation of the jurisdiction. It is a valid information signed by a competent officer which, among other requisites, confers jurisdiction on the court over the person of the accused and the
Corporation subject matter of the accusation. In consonance with this view, an infirmity in the information cannot be cured by silence, acquiescence, or even by express consent.[41] mphasis supplied)
The case of Villa is authority for the principle that lack of authority on the part of the filing officer prevents the court from acquiring jurisdiction over the case. Jurisdiction over the subject
CA: matter is conferred by law while jurisdiction over the case is invested by the act of plaintiff and attaches upon the filing of the complaint or information.[42] Hence, while a court may have
Denied the appeal jurisdiction over the subject matter, like a violation of the SSS Law, it does not acquire jurisdiction over the case itself until its jurisdiction is invoked with the filing of the information.
Basis/reason
Acting on an appeal filed by the Republic, the CA ruled that since the former had actively participated in the proceedings before the lower court, but failed to raise the
jurisdictional challenge therein, petitioner is thereby estopped from questioning the jurisdiction of the lower court on appeal. The CA further found that respondent
Corporation had sufficiently established the latters registrable title over the subject property after having proven open, continuous, exclusive and notorious possession and
occupation of the subject land by itself and its predecessors-in-interest even before the outbreak of World War II.
Lazo v. Republic Surety
ISSUE: G.R. No. L-27365, January 30, 1970
Wether or not the Municipal Trial Court failed to acquire jurisdiction over the application for original registration of land title
TOPIC: Manner of Acquiring Jurisdiction: Over the Issues
RULING:
The Municipal Trial Court propely acquired jurisdiction over the case FACTS: Petitioner Lazo is the guarantor of co-petitioner Robles for a loan obtained from the Philippine Bank of Commerce. Lazo executed a real estate mortgage in favor of the defendant
The delegated jurisdiction of the MTC over cadastral and land registration cases is indeed set forth in the Judiciary Reorganization Act, which provides: Republic Surety & Insurance Co., Inc who was co-debtor in the same loan. Robles failed to pay despite several extensions until the loan was transferred to the Surety who foreclosed extra-
Sec. 34. Delegated Jurisdiction in Cadastral and Land Registration Cases. - Metropolitan Trial Courts, Municipal Trial Courts, and Municipal judicially the mortgage on July 1, 1958. Lazo claims the foreclosure as invalid because plaintiff Jose Robles had paid on the mortgage loan the sum of P13,466.36; he continued to make
Circuit Trial Courts may be assigned by the Supreme Court to hear and determine cadastral or land registration cases covering lots where there is no other payments, aggregating P17,250.00.
controversy or opposition, or contested lots where the value of which does not exceed One hundred thousand pesos (₱100,000.00), such value to be
ascertained by the affidavit of the claimant or by agreement of the respective claimants if there are more than one, or from the corresponding tax declaration CAUSE OF THE DISPUTE: This is an appeal by the defendants from the decision of the CFI of Manila
of the real property. Their decision in these cases shall be appealable in the same manner as decisions of the Regional Trial Courts. (As amended by R.A.
No. 7691) (Emphasis supplied.) ARGUMENTS

Thus, the MTC has delegated jurisdiction in cadastral and land registration cases in two instances: first, where there is no controversy or opposition; or, second, over contested lots, the PLAINTIFF:
value of which does not exceed ₱100,000.
Plaintiff's prayer: Lazo filed a petition for the defendant company to render an accounting of the payments thus made so that if the loan had been paid in full then the real estate mortgage
should be cancelled; otherwise the plaintiffs should be allowed to pay, by way of legal redemption, whatever Balance still remained.

RESPONDENT'S ANSWER: filed a motion to dismiss arguing that the complaint did not state a cause of action and that the claim or demand set forth therein had already prescribed.
The case at bar does not fall under the first instance, because petitioner opposed respondent Corporations application for registration on 8 January 1998.
However, the MTC had jurisdiction under the second instance, because the value of the lot in this case does not exceed ₱100,000.
RTC: went entirely out of the issues submitted to it and chose to decide the case on a point which was not at all litigated, the validity of the mortgage foreclosure. Consequently, the court
Contrary to petitioners contention, the value of the land should not be determined with reference to its selling price. Rather, Section 34 of the Judiciary Reorganization Act provides that the
concluded, the real estate mortgage in favor of said defendant was extinguished, and the foreclosure thereof was a nullity.
value of the property sought to be registered may be ascertained in three ways: first, by the affidavit of the claimant; second, by agreement of the respective claimants, if there are more than
CA: X TOPIC: Manner of Acquiring Jurisdiction: Over the Issues

ISSUE: WON the trial court erred in deciding the case on an issue not raised in the complaint CASE NATURE: PETITION for review on certiorari of a decision of the Court of Appeals.

RULING: Yes. FACTS:


The actuation of the trial court was not legally permissible especially because the theory on which it proceeded involved factual considerations neither touched upon the pleadings nor made
the subject of evidence at the trial. Rule 6, Section 1, is quite explicit in providing that "pleadings are the written allegations of the parties of their respective claims and defenses submitted to CAUSE OF THE DISPUTE: Mercader leased for 20 years a property owned by Manreal but the property was foreclosed by DBP because Manreal defaulted on his loan with the bank.
the court for trial and judgment." This rule has been consistently applied and adhered to by the courts. Mercader had already introduced improvements on the property.
The subject matter of any given case is determined ... by the nature and character of the pleadings submitted by the parties to the court for trial and judgment. Belandres vs. Lopez Sugar
Central Mill Co., Inc., 97 Phil. 100, 103. ARGUMENTS

It is a fundamental principle that judgments must conform to both the pleadings and the proof, and must be in accordance with the theory of the action upon which the pleadings were framed PLAINTIFF: The MERCADERs assert that in issuing the challenged decision, the Court of Appeals contravened Section 4, Rule 20 and Section 5, Rule 10 of the Rules of Court by holding
and the case was tried; that a party can no more succeed upon a case proved. but not alleged, than upon one alleged but not proved." (Ramon v. Ortuzar, 89 Phil. 730, 742) that the trial court should not have taken cognizance of the lease-purchase option as a controversial issue since it was not raised in the pleadings. They maintain that the trial court correctly
took cognizance of the lease-purchase option because it was part and parcel of the pre-trial stages, the determination of which will prevent future litigation thereon.
It is a well-known principle in procedure that courts of justice have no jurisdiction or power to decide a question not in issue." (Lim Toco vs. Go Pay, 80 Phil. 166)
Plaintiff's prayer: Mercader prayed in his complaint that DBP respect the lease period and reimbursement for the improvements and for the annotation of their interest in the TCT.
A judgment going outside the issues and purporting to adjudicate something upon which the parties were not heard, is not merely irregular, but extrajudicial and invalid." (Salvante v. Cruz,
88 Phil. 236, 244.) DEFENDANT'S ANSWER:
The parties here went to court and presented their respective sides on the premise, admitted by both, that the mortgage was valid and subsisting. Evidence, therefore, to establish such
premise was unnecessary and uncalled for. Indeed, it was for that reason and because in any event the record of this case, particularly with respect to the actuations of the parties after the RTC: The trial court ruled in favor of the Mercaders.
mortgage was foreclosed, shows with overwhelming preponderance that the said mortgage had not been extinguished 1. It found that the "DBP had unnecessarily and unjustifiably made Mercader understand that his second option [lease-purchase] would be more or less approved, except that the approval
Redemption issue will come from Manila."
The plaintiffs rely on Rule 39, Section 34.1 In this connection Section 9 of Act No. 3135, as amended, may also be cited. 2. It also believed "quite firmly" that the "DBP could not have escaped having a foreknowledge of the existence of the prior unrecorded lease" as the "possession and cultivation of Bernardo
Implicit in the application of these provisions is the premise that the period for redemption of the property sold on execution (on extrajudicial foreclosure of mortgage in the present case) has Mercader xxx [was] a matter of open, notorious and public knowledge in the area."
not yet expired. For if the right to redeem has been lost it stands to reason that there is no redemption price to speak of, to which the rents received by the purchasers are to be applied or 3. It first acknowledged that it is a "court of equity and not merely a court of law" and the "DBP is not authorized to keep real propert[y] longer than ten years or so.”
credited.3
The plaintiffs' position is that since the sheriff's certificate of sale was recorded in the office of the Register of Deeds for Manila on March 28, 1963, the one-year period of legal redemption CA: It found that the trial court erred in treating the lease-purchase option as a controversial issue considering that it was "outside the parties' pleadings." But invoking the Supreme Court's
had not yet expired when the action was commenced on December 12 of the same year. decision in Castro v. Court of Appeals in that "the improvements introduced [into the mortgaged property] are to be considered so incorporated [in the mortgage] only if so owned by the
There are, however, certain circumstances peculiar to this case which take it out of the operation of the rule concerning registration in this regard. mortgagor," the Court of Appeals declared that the improvements introduced on Lot No. 2985 had been improperly included in the foreclosure sale since they were not owned by the
There is, to begin with, the categorical statement in the certificate of sale that "the period of redemption of the said property sold will expire on the 2nd day of July, 1959." Then there is the mortgagors. But since the improvements were already included in the foreclosure sale and the MERCADERs continued the possession and collection of income from the lot, the Court of
fact that no lien or encumbrance, right or claim of any person, other than the mortgage in question, appeared on the transfer certificate of title of the plaintiff spouses covering the mortgaged Appeals, as already earlier adverted to, reversed and set aside the appealed judgment. It entered a new one declaring that the MERCADERs were not entitled to any compensation from the
property, such that when the defendant company obtained a new transfer certificate in its name on March 28, 1963, the same was entirely clean. In other words, no third parties who might DBP. It also ordered the MERCADERs to immediately turn over the possession of the lot to the DBP.
have an interest in the property, either as possible redemptioners or otherwise, had to be protected by due notice of the sale through its registration. Second, Beginning July 1958,
immediately after the foreclosure sale, the plaintiffs — in some instances in the joint names of Jose Robles and Felix Lazo and in other instances in the name of Jose Robles alone — started ISSUE: WON the CA erred
paying rents on the property to the defendant company, indicating that the former owners, while remaining in occupancy, did so in the concept of tenants.
The plaintiffs having reneged on all their repeated promises, the defendant company finally consolidated its title to the property as purchaser at the foreclosure sale on March 28, 1963, and RULING: This Court agrees with the MERCADERs and finds that the Court of Appeals erred in disregarding as material the lease-purchase option on the ground that it was not raised in the
obtained the corresponding transfer certificate of title. That was almost five years after the said sale. pleadings. If the Court of Appeals adverts to the lack of reference to the lease-purchase option in the initiatory pleadings, this can be simply explained by the fact that the trial court only took
It is clear, in the light of the facts and circumstances above set forth, that the parties had abandoned entirely the concept of legal redemption in this case and converted it into one of cognizance thereof when it became an integral component of the pre-trial proceedings. That is why the lease-purchase option was included firstly, in the pre-trial order as one of the issues to
conventional redemption, in which the only governing factor was the agreement between them.Buce vs. Court of Appeals be resolved at trial and secondly, in the supplemental pleading subsequently filed by the MERCADERs on 7 November 1985. As a supplemental pleading, it served to aver supervening facts
which were then not ripe for judicial relief when the original pleading was filed. As such, it was meant to supply deficiencies in aid of the original pleading, and not to dispense with the latter.
Hence, it was patently erroneous for the Court of Appeals to pronounce that the lease-purchase option was not raised in the pleadings. The DBP was even quite aware and knowledgeable
of the supplemental pleading because it filed an opposition thereto.
332 SCRA 151 , May 12, 2000 The records however reveal that the trial court did not promptly rule on the motion to admit the supplemental pleading. And during trial, the trial court also failed to rule on the prompt
objection interposed by the DBP’s counsel to the MERCADERs’ introduction of evidence relative to said lease-purchase option. But undisputed is the trial court's eventual admission in open
TOPIC: Manner of Acquiring Jurisdiction: Over the Issues court of the MERCADERs’ supplemental pleading
The records also show that not only did the DBP’s counsel began to rigorously cross-examine Bernardo Mercader on the lease-purchase option, he also subjected his witness Mr. Ruben
CASE NATURE: SPECIAL CIVIL ACTION in the Supreme Court. Certiorari. Carpio, then Chief of the Collection Department, DBP to an intensive direct examination covering said subject matter. He also offered as evidence the DBP’s letter indicating the three
options to the MERCADERs as Exhibit "1" and the lease-purchase option contained therein as Exhibit "1-A."36
FACTS: Petitioner leased a 56-square meter parcel of land for a period of fifteen years to commence on 1 June 1979 and to end on 1 June 1994 "subject to renewal for another ten (10) The DBP is undoubtedly estopped from questioning the trial court’s inclusion of the lease-purchase option as a controversial issue. This action of the trial court finds anchor on Section 4,
years, under the same terms and conditions." Petitioner then constructed a building and paid the required monthly rental which periodically increased. Rule 20 of the Rules of Court which reads:
Section 4. Record of pre-trial results. -- After the pre-trial the court shall make an order which recites the action taken at the conference, the amendments allowed to the pleadings, and the
CAUSE OF THE DISPUTE: In 1992, the rent was again increased but Buce only tendered checks for a lesser amount which Tiongco, the owner, refused to accept. agreements made by the parties as to any of the matters considered. Such order shall limit the issues for trial to those not disposed of by admissions or agreements of counsel and when
entered controls the subsequent course of the action, unless modified before trial to prevent manifest injustice.
ARGUMENTS Indeed, the pre-trial is primarily intended to make certain that all issues necessary to the disposition of a case are properly raised. The purpose is to obviate the element of surprise, hence,
the parties are expected to disclose at the pre-trial conference all issues of law and fact which they intend to raise at the trial, except such as may involve privileged or impeaching matter.37
PLAINTIFF: (decision by the CA) by ordering her to vacate the premises, the Appellate Court went beyond the bounds of its authority because the case she filed before the RTC was for [See De la Paz v. Panis, 245 SCRA 242, 248-249 [1995], citing Permanent Concrete Products, Inc. v. Teodoro, 26 SCRA 332, 336 [1968].] In the case at bar, the pre-trial order included as
"Specific Performance" not unlawful detainer. The power to order the lessee to vacate the leased premises is lodged in another forum. Additionally, private respondents did not pray for the integral to the complete adjudication of the case the issue of whether the MERCADERs can demand specific performance from the DBP relative to the lease-purchase option. Thus, the
ejectment of petitioners from the leased premises in their Answer with Counterclaim; well-settled is the rule that a court cannot award relief not prayed for in the complaint or compulsory element of surprise that the provision on pre-trial attempts to preclude was satisfied.
counterclaim. Assuming arguendo that the MERCADERs failed to file the supplemental pleading, evidence relative to the lease-purchase option may be legitimately admitted by the trial court in conformity
with Section 5, Rule 10 of the Rules of Court which states:
Plaintiff's prayer: (filed with the RTC) A prayer for consignation and she prayed that private respondents be ordered to accept the rentals in accordance with the lease contract and to Section 5. Amendment to conform to or authorize presentation of evidence. -- When issues not raised by the pleadings are tried by express or implied consent of the parties, they shall be
respect the lease of fifteen years, which was renewable for another ten years, at the rate of P200 a month. treated in all respects, as if they had been raised in the pleadings. Such amendment of the pleadings as may be necessary to cause them to conform to the evidence and to raise these
issues may be made upon motion of any party at any time, even after judgment; but failure so to amend does not affect the result of the trial of these issues. If evidence is objected to at the
DEFENDANT'S ANSWER: The increase was pursuant to Republic Act No. 877 or the Rent Control Act and that the phrase in the lease contract authorizing renewal for another ten years trial on the ground that it is not within the issues made by the pleadings, the court may allow the pleadings to be amended and shall do so freely when the presentation of the merits of the
does not mean automatic renewal; rather, it contemplates a mutual agreement between the parties. action will be subserved thereby and the objecting party fails to satisfy the court that the admission of such evidence would prejudice him in maintaining his action or defense upon the
merits. The court may grant a continuance to enable the objecting party to meet such evidence. mphasis supplied)
RTC: RTC declared the lease contract automatically renewed for ten years and considered as evidence thereof (a) the stipulations in the contract giving the lessee the right to construct This provision envisions two scenarios -- first, when evidence is introduced on an issue not alleged in the pleadings and no objection was interjected and second, when evidence is offered
buildings and improvements and (b) the filing by petitioner of the complaint almost one year before the expiration of the initial term of fifteen years. It then fixed the monthly rent. The RTC again, on an issue not alleged in the pleadings but this time an objection was interpolated. We are concerned with the second scenario. In Co Tiamco v. Diaz, the Court held that "when
held that the continuous increase of rent from P200 to P250 then P300, P400 and finally P1,000 caused "an inevitable novation of their contract." evidence is offered on a matter not alleged in the pleadings, the court may admit it even against the objection of the adverse party, where the latter fails to satisfy the court that the admission
of the evidence would prejudice him in maintaining his defense upon the merits, and the court may grant him a continuance to enable him to meet the new situation created by the evidence.
CA: Court of Appeals reversed the decision of the RTC, and ordered petitioner to immediately vacate the leased premises on the ground that the contract expired on 1 June 1994 without Of course, the court, before allowing the evidence, as a matter of formality, should allow an amendment of the pleading, xxx And, furthermore, where the failure to order an amendment does
being renewed and to pay the rental arrearages at the rate of P1,000 monthly. According to the Court of Appeals, it is unclear as to who may exercise the option to renew. The stipulation not appear to have caused surprise or prejudice to the objecting party, it may be allowed as a harmless error. Well-known is the rule that departures from procedure may be forgiven where
allowing the construction of a building and other improvements and the fact that the complaint was filed a year before the expiration of the contract are not indicative of automatic renewal. It they do not appear to have impaired the substantial rights of the parties."
applied the ruling in Fernandez v. Court of Appeals13 [166 SCRA 577 (1988)] that without a stipulation that the option to renew the lease is solely for the benefit of one party any renewal of a the Court reinforces the Co Tiamco ruling on the application of Section 5, Rule 10 of the Rules of Court in this wise:
lease contract must be upon the agreement of the parties. The failure of a party to amend a pleading to conform to the evidence adduced during trial does not preclude adjudication by the court on the basis of such evidence which may embody new
The Court of Appeals denied petitioner’s motion for reconsideration. issues not raised in the pleadings. Although, the pleading may not have been amended to conform to the evidence submitted during trial, judgment may nonetheless be rendered, not simply
on the basis of the issues alleged but also on the issues discussed and the assertions of fact proved in the course of the trial. The court may treat the pleading as if it had been amended to
ISSUE: WON the CA erred in its ruling ordering petitioner to vacate the premises conform to the evidence, although it had not been actually amended. xxx Clearly, a court may rule and render judgment on the basis of the evidence before it even though the relevant
pleading had not been previously amended, so long as no surprise or prejudice is thereby caused to the adverse party. Put a little differently, so long as the basic requirements of fair play
RULING: Yes. After the lease terminated on 1 June 1994 without any agreement for renewal being reached, petitioner became subject to ejectment from the premises. It must be noted, had been met, as where the litigants were given full opportunity to support their respective contentions and to object to or refute each other's evidence, the court may validly treat the
however, that private respondents did not include in their Answer with Counterclaim a prayer for the restoration of possession of the leased premises. Neither did they file with the proper pleadings as if they had been amended to conform to the evidence and proceed to adjudicate on the basis of all the evidence before it.
Metropolitan Trial Court an unlawful detainer suit against petitioner after the expiration of the lease contact.
As already enunciated, the DBP was not and would not be prejudiced by the incorporation of the lease-purchase option as one of the controverted issues. Moreover, it had been afforded
Moreover, the issues agreed upon by the parties to be resolved during the pre-trial were the correct interpretation of the contract and the validity of private respondents’ refusal to accept ample opportunity to refute and object to the evidence germane thereto, thus, the rudiments of fair play had been properly observed.
petitioner’s payment of P400 as monthly rental. They later limited the issue to the first, i.e., the correct interpretation of the contract.The issue of possession of the leased premises was not Since we agree with the MERCADERs’ contention that the Court of Appeals contravened Section 4, Rule 20 and Section 5, Rule 10 of the Rules of Court in promulgating the questioned
among the issues agreed upon by the parties or threshed out before the court a quo. Neither was it raised by private respondents on appeal. decision, we have to grant their prayer to refer the matter back to said court for a determination of the question of whether the lease-purchase option was already consummated and for a
complete ascertainment of the rights and obligations of the parties.
As correctly contended by the petitioner, the Court of Appeals went beyond the bounds of its authority when after interpreting the questioned provision of the lease contract in favor of the
private respondents it proceeded to order petitioner to vacate the subject premises. Dispositive Portion: Petition is GRANTED DUE COURSE. Resolution of CA are set aside. The case is REFERRED BACK to the Court of Appeals for a determination of whether the lease-
purchase option was consummated with the end view of ascertaining the rights and obligations of the parties.

Mercader v. DBP
G.R. No. 130699, May 12, 2000 Cesar v. Hon. Ricafort-Bautista
G.R. No. 136415, October 31, 2006
Spouses Felix prayed for an extension of time to file their answer – GRANTED!
TOPIC: Manner of Acquiring Jurisdiction: Over the Person or Party: Plaintiff Spouses Felix failed to file their answer. So St. Joseph Resource Development, Inc. filed a motion to declare said spouses in default – GRANTED!

CASE NATURE: PETITION for review on certiorari of a decision of the Court of Appeals. A copy of the resolution was sent to and received by the counsel of the Felix Spouses through registered mail. The copy of the decision addressed to the spouses was
returned to the court after two notices for having been "Unclaimed." However, then counsel for the Felix Spouses received his copy of the decision.
FACTS: After the filing of the complaint, summons9 was issued to petitioner and this was served by Sheriff Juan C. Marquez. As petitioner failed to file his answer to the complaint, private
respondent moved that he be declared in default.This motion was favorably acted upon by public respondent No appeal was made. The motion for a writ of execution was filed and a copy was served to the spouses by registered mail but they failed to oppose the motion.

CAUSE OF THE DISPUTE: Petitioner's failure to pay the construction materials it purportedly purchased under a credit line extended by private respondent Specified Materials Corporation. The court thereafter issued an order granting the motion and directing the issuance of a writ of execution. The counsel for the Felix Spouses received a copy of the said order.

ARGUMENTS Personal properties were levied and sold at a public auction. St. Joseph Resource Development, Inc. won as the highest bidder.
COURT OF APPEALS
PLAINTIFF:Petitioner argues that since the trial court never acquired jurisdiction over his person. He claims that the person who allegedly received the summons on his behalf, and who was The wife, Ofelia Herrera-Felix, filed a petition under Rule 47 of the Rules of Court for the nullification of the trial court's judgment by default, the writ of execution issued by the said court, and
identified in the sheriff's return as Arsenio Robles, was not his employee. He adds that when he conducted an inquiry, he found out that Robles was a native of Batangas and was merely the sale of her properties at public auction.
peddling mango seedlings within the vicinity of his office when the summons was served. He also maintains that had he been given the opportunity to present his defense, he would have Basis: The complaint and summons were handed over to her sister who was merely a visitor in her house and, as such, was not a valid substituted service under Rule 14,
shown that his obligation to private respondent is less than the amount as established by the trial court. Section 7 of the Rules of Court.
Plaintiff's prayer: seeks the annulment of the decision respondent Honorable Helen Ricafort-Bautista of the RTC of Parañaque City St. Joseph Resource averred that even if such substituted service on the petitioner was defective, the defect was cured when the latter, through her counsel, appeared in court
and moved for an extension of time to file her responsive pleading.
RESPONDENT’S ANSWER: Private respondent retorts that petitioner's insistence that the court a quo did not acquire jurisdiction over him is belied by the fact that petitioner had actual
knowledge of all the proceedings since he was furnished with all the copies of the pleadings and court orders. Private respondent points out that the Motion to Admit Amended Complaint CA decided in favor of St. Joseph.
and the Amended Complaint were personally served on petitioner himself as shown by his signatures appearing thereon. Moreover, private respondent is of the view that the sheriff who ISSUE
served the summons upon petitioner enjoys the presumption of regularity in the performance of duty – a presumption which petitioner was unable to overcome. Whether or not the appearance of the spouses’ counsel constitutes a voluntary submission to the jurisdiction of the court
RTC: As petitioner failed to file his answer to the complaint, private respondent moved that he be declared in default. This motion was favorably acted upon by public respondent through the RULING
Order dated 14 March 1997, and private respondent was able to present its evidence. Yes.
CA: Following the denial of its Motion to Set Aside Decision, petitioner filed before the Court of Appeals a Petition for Annulment of Judgment, Preliminary Injunction with Prayer for The court acquires jurisdiction over the person of the defendant:
Temporary Restraining Order.[16] This petition was dismissed for “failure to attach an affidavit of merit as well as the affidavits of witnesses or document supporting the defense.
Petitioner then filed a motion for reconsideration but this was denied by the Court of Appeals because an annulment for judgment is proper only when there are no remedies available. 1. by service of the complaint and summons on him, either:
a. by personal service or
ISSUE: WON the court a quo acquired jurisdiction over the person of the petitioner by virtue of the substituted service of summons effected by sheriff Marquez. b. by substituted service or
c. by extra-territorial service thereof; or
RULING: It is fundamental that courts acquire jurisdiction over the plaintiff once the complaint is filed. On the other hand, there are two ways through which jurisdiction over the defendant or 2. by his voluntary personal appearance before the court or through counsel.
respondent is acquired – either through the service of summons upon them or through their voluntary appearance in court. In the case of Avon Insurance PLC v. Court of Appeals,[28] we
discussed the function of summons in court actions, to be –
Fundamentally, the service of summons is intended to give official notice to the defendant or respondent that an action had been commenced against it. The defendant or respondent is thus In this case, the petitioner appeared before the court, through counsel, and filed a motion for extension of time to file her answer to the complaint which the trial court granted. By filing the
put [on] guard as to the demands of the plaintiff as stated in the complaint. The service of summons upon the defendant becomes an important element in the operation of a court’s said motion, through counsel, the petitioner thereby submitted herself to the jurisdiction of the trial court.
jurisdiction upon a party to a suit, as service of summons upon the defendant is the means by which the court acquires jurisdiction over his person. Without service of summons, or when
summons are improperly made, both the trial and the judgment, being in violation of due process, are null and void, unless the defendant waives the service of summons by voluntarily A voluntary appearance is a waiver of the necessity of a formal notice.
appearing and answering the suit. [29]
Elsewhere, we declared that jurisdiction of the court over the person of the defendant or respondent cannot be acquired notwithstanding his knowledge of the pendency of a case against An appearance in whatever form, without explicitly objecting to the jurisdiction of the court over the person, is a submission to the jurisdiction of the court over the
him unless he was validly served with summons.[30] Such is the important role a valid service of summons plays in court actions. person.
The Rules of Court[31] requires that, whenever practicable, summons must be served by handing a copy thereof to the defendant in person. In case the defendant refuses to receive and
sign for it, by tendering the summons to him or her. Formal method of entering an appearance
However, in the event that summons cannot be served within a reasonable time, the Rules permit that substituted service may be resorted to, thus:
Sec. 7. Substituted service. - If, for justifiable causes, the defendant cannot be served within a reasonable time as provided in the preceding section, service may be effected (a) by leaving
 deliver to the clerk a written direction ordering him to enter the appearance of the person who subscribes it
copies of the summons at the defendant’s residence with some person of suitable age and discretion then residing therein, or (b) by leaving the copies at defendant’s office or regular place
 an appearance may be made by simply filing a formal motion, or plea or answer.
of business with some competent person in charge thereof.
In this case, the sheriff employed the substituted service of summons. The defect, however, in the manner in which he implemented this mode of service of summons is readily apparent on
the face of the return. It must be emphasized that laws providing for modes other than the personal service of summons must be strictly followed in order for the court to acquire jurisdiction
over the person of respondent or defendant. Compliance therewith should appear affirmatively on the return.[32] The essence of this requirement was enunciated in the case of Keister v. This formal method of appearance is not necessary. He may appear without such formal appearance and thus submit himself to the jurisdiction of the court.
Navarro[33] to be –
The summons must be served to the defendant in person. It is only when the defendant cannot be served personally within a reasonable time that a substituted service may be made. He may appear by presenting a motion, for example, and unless by such appearance he specifically objects to the jurisdiction of the court, he thereby gives his assent to the jurisdiction of
Impossibility of prompt service should be shown by stating the efforts made to find the defendant personally and the fact that such efforts failed. This statement should be made in the proof the court over his person. When the appearance is by motion objecting to the jurisdiction of the court over his person, it must be for the sole and separate purpose of objecting to the
of service. This is necessary because substituted service is in derogation of the usual method of service. It has been held that this method of service is “in derogation of the common law; it jurisdiction of the court. If his motion is for any other purpose than to object to the jurisdiction of the court over his person, he thereby submits himself to the jurisdiction of the court.
is a method extraordinary in character, and hence may be used only as prescribed and in the circumstances authorized by statute.” Thus, under the controlling decisions, the statutory
requirements of substituted service must be followed strictly, faithfully and fully, and any substituted service other than that authorized by the statute is considered ineffective.[34] ( mphases
supplied.) JAPRL DEV’T CORP., ET AL. VS. SECURITY BANK CORP.
As the sheriff’s return in the present case does not contain any statement with regard to the impossibility of personal service the same is patently defective and so the presumption of CITATION: G.R. No. 190107. June 6, 2011.
regularity in the performance of official functions will not lie.[35] TOPIC: JURISDICTION OVER THE PERSON OF THE DEFENDANT; BY VOLUNTARY APPEARANCE
Nevertheless, we still hold that jurisdiction was validly acquired by the trial court. Although the substituted service upon him of summons was defective, said defect was cured by his
voluntary appearance. FACTS
As the records of this case disclose, after private respondent moved for the execution of the trial court’s decision, petitioner filed a motion for a re-setting of the court’s hearing thereon. In JAPRL – domestic corp engaged in fabrication, manufacture and distribution of steel products
Flores v. Zurbito,[37] we held that an appearance in whatever form without expressly objecting to the jurisdiction of the court over the person, is a submission to the jurisdiction of the court
over the person of the defendant or respondent, thus: JAPRL applied for a credit facility for P50M with Security Bank Corp. (SBC) – approved!
A voluntary appearance is a waiver of the necessity of a formal notice. An appearance in whatever form, without expressly objecting to the jurisdiction of the court over the person, is a
submission to the jurisdiction of the court over the person. While the formal method of entering an appearance in a cause pending in the courts is to deliver to the clerk a written direction JAPRL Chairman and President (Limson and Arollado) executed a Continuing Suretyship Agreement (CSA) 2 in favor of SBC wherein they guaranteed the due and full payment and
ordering him to enter the appearance of the person who subscribes it, an appearance may be made by simply filing a formal motion, or plea or answer. This formal method of appearance is performance of JAPRL's guaranteed obligations under the credit facility.
not necessary. He may appear without such formal appearance and thus submit himself to the jurisdiction of the court. He may appear by presenting a motion, for example, and unless by
such appearance he specifically objects to the jurisdiction of the court, he thereby gives his assent to the jurisdiction of the court over his person.[38] Subsequently, JAPRL's financial adviser, MRM Management Incorporated (MRM), convened JAPRL's creditors, SBC included, for the purpose of restructuring JAPRL's
Hence, in this case, petitioner’s filing of a Motion for Re-setting of the Hearing effectively cured the defect of the substituted service of summons. Petitioner’s insistence of lack of jurisdiction existing loan obligations. Copies of JAPRL's financial statements (FS) from 1998 to 2001 were given for the creditors to study.
over his person is utterly lacking in any legal basis.
SBC soon discovered material inconsistencies in the FS given by MRM vis-Ã -vis those submitted by JAPRL when it applied for a credit facility, drawing SBC to conclude that JAPRL
HERRERA-FELIX VS. COURT OF APPEALS committed misrepresentation.
CITATION: G.R. No. 143736. August 11, 2004.
TOPIC: JURISDICTION OVER THE PERSON OF THE DEFENDANT; BY VOLUNTARY APPEARANCE As per agreement, any misrepresentation will constitute an event of default committed by JAPRL and its sureties.

FACTS SBC sent a letter of demand to JAPRL and its chairman and president. Because they failed to comply with the demand, SBC filed a complaint for sum of money with application for issuance
Original complainant: of writ of preliminary attachment before the Regional Trial Court (RTC) of Makati City.
St. Joseph Resource Development, Inc. (now respondent) RTC
The Makati RTC at ordered in open court the archiving of SBC's complaint for sum of money until disposition by the Quezon City RTC of JAPRL's petition for rehabilitation.
Complaint:
Sum of money against the Spouses Restituto and Ofelia Felix with a prayer for a writ of preliminary attachment. Consequently, RTC dismissed SBC’s complaint without prejudice.

Cause of action: SBC filed a motion for reconsideration stating that the suspension of the proceedings should only be with respect to JAPRL but not with respect to the chairman and president. – Denied!
During the period from November 16, 1992 to December 14, 1992, the Felix Spouses purchased from the respondent tubs of assorted fish. Balance payable P1,132,065.50.
RTC SBC filed another MR. – denied!
Decision in favor of St. Joseph Resource Development, Inc. RTC also granted the writ of preliminary attachment on a bond of P1,132,065.50 (the amount of balance).
Limson and Arollado opposed, claiming that summons were not served on them, hence, the Makati RTC failed to acquire jurisdiction over their person.
Copies of the writ of preliminary attachment, summons and complaint were served to the spouses at their residence, through the sister of Ofelia Herrera-Felix, Ma. Luisa COURT OF APPEALS
Herrera. (Ofelia was out of the country)
The appellate court held that Limson and Arollado voluntarily submitted themselves to the jurisdiction of the Makati RTC, despite the qualification that the filing of their respective
would bind themselves, jointly and severally with the Rebollado's, to pay the entire obligation subject of the suit. Felisa Tolentino who was then present agreed to this proposal, and so the
"Opposition[s] Ad Cautelam" and "Manifestation[s] Ad Cautelam," was "by way of special appearance" they having sought affirmative relief by praying for the archiving of SBC's complaint.
petitioner Santos, at the request of the petitioner Rodriguez, drew up a motion for judgment on a compromise embodying the terms of the agreement of the parties. On the basis of the said
Limson and Arollado filed a motion for recon – denied!
motion, the city court, on August 14, 1971, rendered judgment, as follows:
ISSUE
Whether or not the court has acquired jurisdiction over Limson and Arollado
Parties herein submitted the following compromise agreement and prayed that judgment be rendered in accordance there with:
RULING
Yes.

When a defendant's appearance is made precisely to object to the jurisdiction of COMPROMISE AGREEMENT
the court over his person, it cannot be considered as appearance in court. xxx xxx xxx
Limson and Arollado glossed over the alleged lack of service of summons, however, and proceeded to exhaustively discuss why SBC's complaint could not prosper against them as sureties.
They thereby voluntarily submitted themselves to the jurisdiction of the Makati RTC.
1. That the defendants admit all the material allegations in the plaintiff's complaint and acknowledged their indebtedness to the plaintiff in the total amount of P5,980.00, which

amount includes expenses of litigation;


RAPID CITY REALTY & DEV’T CORP. VS. ORLANDO VILLA and LOURDES PAEZ-VILLA
CITATION: G.R. No. 184197. February 11, 2010.
TOPIC: JURISDICTION OVER THE PERSON OF THE DEFENDANT; BY VOLUNTARY APPEARANCE
2. That in consideration of defendants acknowledging their said indebtedness and confessing judgment therefor, plaintiff has allowed defendants some consideration by allowing
FACTS them to pay their above-stated account in the following manner, to wit:
RCRDC filed a complaint for declaration of nullity of subdivision plans, mandamus and damages against several defendants including Spouses Orlando and Lourdes Villa.
a) the sum of P200.00 shall be paid upon the signing of this compromise agreement;
After one failed attempt at personal service of summons, the court process server, resorted to substituted service by serving summons upon respondents' househelp who did not
acknowledge receipt thereof and refused to divulge their names. b) the remaining balance shall be paid in installment basis at the rate of P100.00 a week, payable every Saturday beginning August 28, 1971 and every Saturday of the

week thereafter until fully paid.


Despite substituted service, the Villas failed to file their Answer, prompting RCRDC to fle a "Motion to Declare Defendants in Default". – Granted!

More than eight months thereafter, the Villas filed a Motion to Lift Order of Default, claiming that on they have just received the documents recently and they denied the existence of two
women helpers who allegedly refused to sign and acknowledge receipt of the summons. In any event, they contended that assuming that the allegation were true, the helpers had no 3. That in order to secure the prompt payment of the said obligations of the defendants, Federico Tolentino and Felisa Tolentino hereby bind themselves to pay jointly and severally
authority to receive the documents.
with the defendants the said obligations, and in the event of default on the part of the defendants to pay any of the said installments when the same is already due, the judgment
RTC set aside the Order of Default and gave them 5 days to file their Answer. Respondents just the same did not file an Answer, drawing petitioner to again file a Motion to declare them in
which may be rendered by virtue hereof as to full amount remaining unpaid, may likewise be executed as against the properties of Federico Tolentino and Felisa Tolentino;
default, which the trial court again granted.

The Villas filed an Omnibus Motion for reconsideration of the second order declaring them in default and to vacate proceedings, this time claiming that the trial court did not
acquire jurisdiction over their persons due to invalid service of summons. – denied! 4. That failure on the part of the defendants to pay any one of the installments as above-scheduled shall render the remaining balance unpaid immediately due and demandable and

Filed certiorari in CA. the plaintiff shall then be entitled to the execution of the judgment which may be rendered by virtue hereof;
COURT OF APPEALS
CA annulled the trial court's Orders declaring respondents in default for the second time.
WHEREFORE, judgment by COMPROMISE is hereby rendered pursuant to the foregoing agreement, enjoining strict compliance thereto by the parties.
Basis: The Villas focused all their energies on questioning the RTC’s jurisdiction. The latter motion clearly stated prefatorily their counsel's reservation or "special
appearance to question jurisdiction" over the persons of the petitioners. "A party
who makes a special appearance in court challenging the jurisdiction of said court based on the ground of invalid service of summons is not deemed to have The Rebollado's subsequently failed to comply with the terms of the compromise, thus prompting the petitioner Rodriguez to ask the city court for a writ of execution not only against the
submitted himself to the jurisdiction of the court."
Rebollados but as well against the Tolentino's. When this was granted; and later affirmed over the opposition of the Tolentino's, the latter brought an action for certiorari in the respondent
RCRDC filed a petition for review in the SC.
Court of First Instance of Manila, docketed as Civil Case 85998, to enjoin the city court from enforcing any writ of execution against them. On December 20, 1973, after hearing duly had, the
ISSUE
respondent court rendered judgment excluding the Tolentinos from the effects of the writ of execution granted by the city court in Civil Case 204601. It is this judgment that is the subject of
Whether or not the RTC acquired jurisdiction over the person of the Villas
the present appeal.
RULING
Yes. In excluding the Tolentinos from the effects of the judgment on a compromise rendered by the city court, the respondent court invokes two reasons: first, the dispositive portion of the

It is settled that if there is no valid service of summons, the court can still acquire judgment quoted above cannot be executed because it does not explicitly enjoin the Tolentino's to pay, jointly and severally with the Rebollado's, the amount due to the plaintiff; and second,
jurisdiction over the person of the defendant by virtue of the latter's voluntary appearance. Thus Section 20 of Rule 14 of the Rules of Court provides:
the city court never acquired jurisdiction over the persons of the Tolentino's and, therefore, the latter cannot be bound by the judgment rendered in Civil Case 204601.
Sec. 20. Voluntary appearance. — The defendant's voluntary appearance in the action shall be equivalent to service of summons. The inclusion in a motion to dismiss of
other grounds aside from lack of jurisdiction over the person shall not be deemed a voluntary appearance.

Prescinding from the foregoing, it is thus clear that: Thus, the petitioners Belen S. Rodriguez and Jose S. Santos, Jr., have come to this Court on appeal by certiorari.
(1) Special appearance operates as an exception to the general rule on voluntary appearance;
(2) Accordingly, objections to the jurisdiction of the court over the person of the defendant must be explicitly made, i.e., set forth in an unequivocal manner; and
(3) Failure to do so constitutes voluntary submission to the jurisdiction of
the court, especially in instances where a pleading or motion seeking affirmative relief is filed and submitted to the court for resolution.
Issue:
The Villas did not, in their first motion to lift the order of default, allege that their filing thereof was a special appearance for the purpose only to question the jurisdiction over their persons.
Clearly, they had acquiesced to the jurisdiction of the court.
Whether the city court never acquired jurisdiction over the persons of the Tolentino's and, therefore, the latter cannot be bound by the judgment rendered in Civil Case 204601.

Belen Rodriguez and Jose Santos, Jr., Petitioners, v. Hon. Federico Alikpala (CFI of Manila, Branch XXII), Federico Tolentino and Felisa Tolentino, Respondents.
G.R. No. L-38314, June 25, 1974.
Ruling:

Topic: Manner of acquiring jurisdiction over the person or party by voluntary submission.
The respondent court is in error.

There is no question in the mind of the respondent court that the Rebollado's and the Tolentinos freely and voluntarily entered into the compromise agreement which became the basis of the

Facts: judgment of the city court. Be it remembered that neither the Rebollado's nor the Tolentino's question the existence of the indebtedness of the Rebollados or the amount thereof. The

respondent court heard the testimonies of the witnesses first hand and accorded no credence to the version of the Rebollado's and the Tolentino's that Manuel and Fe Rebollado and Felisa
On August 19, 1971 the petitioner Rodriguez, assisted by her counsel, the petitioner Santos, filed an action, docketed as Civil Case 204601, with the city court of Manila against the spouses Tolentino were made to sign the motion for a judgment on a compromise without being permitted to read its contents and, further, that Felisa Tolentino was induced to sign, too, the name of
Manuel and Fe Rebollado for recovery of the sum of P5,320 plus interest, attorney's fees and costs. A writ of preliminary attachment was issued and served on the Rebollados at their store her husband without any authority from the latter. The respondent court analyzed the evidence at length and found that the involvement of the Tolentino's in the compromise agreement
in Divisoria market. Fe Rebollado immediately communicated with the petitioner Santos, and later with the latter's client, the petitioner Rodriguez, to plead for time before the attachment was arose out of their natural filial concern for their daughter Fe whose inventories at Divisoria market were under imminent threat of levy and seizure. The respondent court, moreover, brooks no
to be effectively enforced. Rodriguez agreed to cause the suspension of the attachment writ on condition that Fe Rebollado's parents, the now respondents Federico and Felisa Tolentino, doubt, and we concur with it, that both the Rebollado's and the Tolentino's understood the plain unequivocal terms of the compromise agreement. And by assuming the roles of co-movants
in the motion for a judgment on a compromise, the Tolentino's actively instigated the city court into giving its judicial imprimatur to the said agreement as well as their participation therein. Respondent Quemada in his answer alleged that inasmuch as his action against Mrs. Midgely concerns property located here in which she claims an interest, it is not necessary that

Under the circumstances, the Tolentino's are estopped from denying the very authority they have invoked.3 jurisdiction over her person be acquired. The service of summons upon her was not for the purpose of acquiring jurisdiction over her person but merely as a matter of due process.

Issue:
Moreover, because they signed and executed the compromise agreement willingly and voluntarily, and, in a manner of speaking, with their eyes wide open, they should be bound by its

terms. A person cannot, to paraphrase Justice Alejo Labrador, repudiate the effects of his voluntary acts simply because they do not suit him. In the very words of Justice Labrador, "in a
Whether Judge Ferandos gravely abused his discretion in denying Mrs. Midgely's motion to dismiss based on the grounds of lack of jurisdiction over her person
regime of law and order, repudiation of an agreement validly entered into can not be made without any ground or reason in law or in fact for such repudiation."4

And even if we assume that estoppel does not apply in this case, we nonetheless cannot shunt aside the principle of equity that jurisdiction over a person not formally or originally a party to Ruling:
a litigation may nevertheless be acquired, under proper conditions, thru the voluntary appearance of that person before the court. Thus, judgment may be directed against one who, although

not a formal party in the case, has assumed or participated in the defense.5 By coming forward with the original litigants in moving for a judgment on a compromise and, furthermore, by The Court dismissed the petition for certiorari.

assuming such interest in the final adjudication of the case as would place them in unequivocal liability, together with the Rebollado's, to the plaintiff therein, the Tolentino's effectively

submitted themselves to the jurisdiction of the city court. They were and are thus subject to its judgment. Supposing arguendo that the lower court did not acquire jurisdiction over the person of Mrs. Midgely, still her motion to dismiss was properly denied because Quemada's action against her

may be regarded as a quasi in rem action where jurisdiction over the person of the nonresident defendant is not necessary and where service of summons is required only for the purpose of

complying with the requirement of due process (Perkins vs. Dizon, 69 Phil. 186; Banco Español-Filipino vs. Palanca, 37 Phil. 921; Mabanag vs. Gallemore, 81 Phil. 254).

SOFIA PASTOR DE MIDGELY, Petitioner, v. THE HONORABLE PIO B. FERANDOS, Judge of the Court of First Instance of Cebu, Branch IX and LEWELYN BARLITO QUEMADA, Special An action quasi in rem is an action between parties where the direct object is to reach and dispose of property owned by them, or of some interest therein (1 Am Jur 2nd 574; State ex rel.
Administrator of the Testate and Intestate Estate of ALVARO PASTOR Y TATO, Respondents.
South Brevard Drainage Dist. vs. Smith, 170 So. 440, 126 Fla. 72). Quemada's action falls within that category.
G.R. No. L-34314 May 13, 1975

The service of summons may, with leave of court, be effected out of the Philippines in three ways: (1) by personal service; (2) by publication in a newspaper of general circulation in such
Topic: Manner of acquiring jurisdiction over the res.
places and for such time as the court may order, in which case a copy of the summons and order of the court should be sent by registered mail to the last known address of the defendant,

and (3) service of summons may be effected in any other manner which the court may deem sufficient. That third mode of extraterritorial service of summons was substantially complied with

in this case.
Facts:

In Civil Case No. 274-T the subject matter of the action for reconveyance consists of properties of Alvaro Pastor, Sr. which are located in Cebu. Mrs. Midgely claims an actual interest in
Alvaro Pastor, Sr., a Spanish citizen, was allegedly the owner of properties and rights in mining claims located in Cebu and supposedly held in trust by his son, Alvaro Pastor, Jr., and his those properties. She has been receiving a share of the income therefrom. Therefore, the extraterritorial service of summons upon her was proper.
daughter-in-law, Maria Elena Achaval-Pastor. Pastor, Sr. died on June 5, 1966. He was survived by his wife, Sofia Pastor y Bossio (who died on October 21, 1966) and by his two legitimate

children, Mrs. Midgely and Alvaro Pastor, Jr. Respondent Quemada claims to be his illegitimate child. This Court clarified that in a quasi in rem action jurisdiction over the person of the nonresident defendant is not essential. The service of summons by publication is required "merely to satisfy

the constitutional requirement of due process". Consequently, the lower court had jurisdiction to try the case even if it had not acquired jurisdiction over the person. The judgment would be
Alvaro Pastor, Sr. in his supposed holographic will dated July 31, 1961 devised to LewelynBarlito Quemada thirty percent of his forty-two percent share in certain mining claims and real confined to the res. No personal judgment could be rendered against the non-resident.
properties. In 1970 the alleged will was presented for probate in Special Proceedings No. 3128-R assigned to Branch I in Cebu City of the Court of First Instance of Cebu. Quemada was

appointed special administrator of the decedent's estate. It should be noted that Civil Case No. 274-T is related to the testamentary proceeding (which is a proceeding in rem par excellance) because the former case was filed by Quemada for the

purpose of recovering the properties which, according to his understanding, belong to the estate of Alvaro Pastor, Sr. and which are held by Mrs. Midgely and the spouses Alvaro Pastor, Jr.
As such administrator and as heir of Alvaro Pastor, Sr., Quemada filed in the Court of First Instance of Cebu at Toledo City a complaint dated December 7, 1970 against the spouses Alvaro and Maria Elena Achaval.
Pastor, Jr. and Maria Elena Achaval, Mrs. Midgely, Atlas Consolidated Mining and Development Corporation and Caltex (Philippines), Inc. to settle the question of ownership over certain real

properties and the rights in some mining claims, to obtain an accounting and payment of the royalties and income thereof and for the payment of damages amounting to P25,000.

Quemada's theory is that those properties and income belong to the estate of Alvaro Pastor, Sr.
IDONAH SLADE PERKINS, Petitioner,v.ARSENIO P. DIZON, Judge of First Instance of Manila, EUGENE ARTHUR PERKINS, and BENGUET CONSOLIDATED MINING COMPANY,
Respondents.
Allegedly without complying with the requirements of Rule 14 of the Rules of Court, Quemada caused extraterritorial service of summons to be made in that case through the Department of
G.R. No. 46631, November 16, 1939.
Foreign Affairs and the Philippine Embassy in Madrid, Spain, which effected the service of the summons by registered mail upon Mrs. Midgely and the Pastor, Jr. spouses at their respective

address in Alicante and Barcelona, Spain. Topic: Manner of acquiring jurisdiction over the res.

Alvaro Pastor, Jr. and Mrs. Midgely, in their respective letters to the Philippine Embassy dated February 11 and 12, 1971, acknowledged the service of summons but reserved the right to

contest the courtsjurisdiction over their persons contending that as nonresidents they could be summoned only with leave of court and that the requirements laid down in section 17 of Rule Facts:
14 should have been observed.

On July 6, 1938, respondent, Eugene Arthur Perkins, instituted an action in the Court of First Instance of Manila against the Benguet Consolidated Mining Company for dividends amounting
CFI Ruling to P71,379.90 on 52,874 shares of stock registered in his name, payment of which was being withheld by the company; and, for the recognition of his right to the control and disposal of said

shares, to the exclusion of all others.Benguet Consolidated Mining Companyfiled its answer alleging, by way of defense, that the withholding of such dividends and the non-recognition of
Judge Ferandos denied the motion ruling that Mrs. Midgely and Spouses Pastor, Jr. had been properly summoned.Mrs. Midgely's motion for reconsideration of the order denying her motion plaintiff's right to the disposal and control of the shares were due to certain demands made with respect to said shares by the Petitioner herein, Idonah Slade Perkins, and by one George H.
to dismiss was denied by Judge Ferandos in his order of September 27, 1971 wherein he ruled that the action filed by Quemada was for the recovery of real properties and real rights. Engelhard. The answer prays that the adverse claimants be made parties to the action and served with notice thereof by publication.

Thus,Mrs. Midgely filed the special civil action of certiorari against Judge Ferandos and Quemada in order to set aside the said Order(denying her motion to dismiss based on lack of On September 5, 1938, the trial court ordered respondent Eugene Arthur Perkins to include in his complaint as parties defendant petitioner, Idonah Slade Perkins, and George H. Engelhard.
jurisdiction). The complaint was accordingly amended and in addition to the relief prayed for in the original complaint, respondent Perkins prayed that petitioner Idonah Slade Perkins and George

Engelhard be adjudged without interest in the shares of stock in question and excluded from any claim they assert thereon. Thereafter, summons by publication were served upon the non-

resident defendants, Idonah Slade Perkins.


within its limits that its tribunals can inquire into the non-resident's obligations to its own citizens, and the inquiry can then be carried only to the extent necessary to control the disposition of

On December 10, 1938, Petitioner Idonah Slade Perkins, through counsel, filed her pleading entitled "objection to venue, motion to quash, and demurrer to jurisdiction" wherein she the property. If the non-resident has no property in the State, there is nothing upon which the tribunals can adjudicate." (Pennoyer v. Neff, supra.)

challenged the jurisdiction of the lower court over her person.

In the instant case, there can be no question that the action brought by Eugene Arthur Perkins in his amended complaint against the petitioner, Idonah Slade Perkins, seeks to exclude her

Petitioner Idonah Slade Perkins’ objection, motion and demurrer having been overruled as well as her motion for reconsideration of the order of denial, she now brought the present petition from any interest in a property located in the Philippines. That property consists in certain shares of stocks of the Benguet Consolidated Mining Company, a sociedad anonima, organized in

for certiorari, praying that the summons by publication issued against her be declared null and void for lack of jurisdiction over her person. the Philippines under the provisions of the Spanish Code of Commerce, with its principal office in the City of Manila and which conducts its mining activities therein. The situs of the shares is

in the jurisdiction where the corporation is created, whether the certificated evidencing the ownership of those shares are within or without that jurisdiction. (Fletcher Cyclopedia

Corporations, Permanent ed. Vol. 11, p. 95). Under these circumstances, we hold that the action thus brought is quasi in rem, for while the judgement that may be rendered therein is not
Issue:
strictly a judgment in rem, "it fixes and settles the title to the property in controversy and to that extent partakes of the nature of the judgment in rem." (50 C.J., p 503). As held by the

Supreme Court of the United States in Pennoyer v. Neff (supra);


Whether or not the Court of First Instance of Manila has acquired jurisdiction over the person of the PetitionerIdonah Slade Perkins as a non-resident defendant, or,

notwithstanding the want of such jurisdiction, whether or not said court may validly try the case.
It is true that, in a strict sense, a proceeding in rem is one taken directly against property, and has for its object the disposition of the property, without reference to the title of

individual claimants; but, in a large and more general sense, the terms are applied to actions between parties, where the direct object is to reach and dispose of property owned by

Ruling: them, or of some interest therein.

The present petition for certiorari of Idonah Slade Perkins, praying that the summons by publication issued against her be declared null and void for lack of jurisdiction over her person, is The action being in quasi in rem, The Court of First Instance of Manila has jurisdiction over the person of the non-resident. In order to satisfy the constitutional requirement of due process,
denied. summons has been served upon her by publication. There is no question as to the adequacy of publication made nor as to the mailing of the order of publication to the petitioner's last known

place of residence in the United States. But, of course, the action being quasi in rem and notice having be made by publication, the relief that may be granted by the Philippine court must be
Section 398 of our Code of Civil Procedure provides that when a non-resident defendant is sued in the Philippine courts and it appears, by the complaint or by affidavits, that the action confined to the res, it having no jurisdiction to render a personal judgment against the non-resident. In the amended complaint filed by Eugene Arthur Perkins, no money judgment or other
relates to real or personal property within the Philippines in which said defendant has or claims a lien or interest, actual or contingent, or in which the relief demanded consists, wholly or in relief in personam is prayed for against the petitioner. The only relief sought therein is that she be declared to be without any interest in the shares in controversy and that she be excluded
part, in excluding such person from any interest therein, service of summons maybe made by publication. from any claim thereto.

We have fully explained the meaning of this provision in El Banco Español Filipino vs. Palanca, 37 Phil., 921, wherein we laid down the following rules: Petitioner contends that the proceeding instituted against her is one of interpleading and is therefore an action in personam. Section 120 of our Code of Civil Procedure provides that
(1) In order that the court may validly try a case, it must have jurisdiction over the subject-matter and over the persons of the parties. Jurisdiction over the subject-matter is whenever conflicting claims are or may be made upon a person for or relating to personal property, or the performance of an obligation or any portion thereof, so that he may be made
acquired by concession of the sovereign authority which organizes a court and determines the nature and extent of its powers in general and thus fixes its jurisdiction with reference subject to several actions by different persons, such person may bring an action against the conflicting claimants, disclaiming personal interest in the controversy, and the court may order
to actions which it may entertain and the relief it may grant. Jurisdiction over the persons of the parties is acquired by their voluntary appearance in court and their submission to its them to interplead with one another and litigate their several claims among themselves, there upon proceed to determine their several claims. Here, The Benguet Consolidated Mining
authority, or by the coercive power of legal process exerted over their persons. Company, in its answer to the complaint filed by Eugene Arthur Perkins, averred that in connection with the shares of stock in question, conflicting claims were being made upon it by said
(2) When the defendant is a non-resident and refuses to appear voluntary, the court cannot acquire jurisdiction over his person even if the summons be served by plaintiff, Eugene Arthur Perkins, his wife Idonah Slade Perkins, and one named George H. Engelhard, and prayed that these last two be made parties to the action and served with summons
publication, for he is beyond the reach of judicial process. No tribunal established by one State can extend its process beyond its territory so as to subject to its decisions either by publication, so that the three claimants may litigate their conflicting claims and settle their rights among themselves.
persons or property located in another State x xxxthe proposition that jurisdiction over the person cannot be thus acquired by publication and notice is no longer open to question;

and it is now fully established that a personal judgment upon constructive or substituted service against a non-resident who does not appear is wholly invalid. This doctrine applies to Had not the complaint been amended, including the herein petitioner as an additional defendant, and had the court, upon the filing of the answer of the Benguet Consolidated Mining
all kinds of constructive or substituted process, including service by publication and personal service outside of the jurisdiction in which the judgment is rendered; and the only Company, issued an order under section 120 of the Code of Civil Procedure, calling the conflicting claimants into court and compelling them to interplead with one another, such order could
exception seems to be found in the case where the non-resident defendant has expressly or impliedly consented to the mode of service. not perhaps have validly been served by publication or otherwise, upon the non-resident Idonah Slade Perkins, for then the proceeding would be purely one of interpleading. Such
(3) The general rule, therefore, is that a suit against a non-resident cannot be entertained by a Philippine court. Where, however, the action is in rem or quasi in rem in proceeding is a personal action, for it merely seeks to call conflicting claimants into court so that they may interplead and litigate their several claims among themselves, and no specific relief
connection with property located in the Philippines, the court acquires jurisdiction over the res, and its jurisdiction over the person of the non-resident is non-essential. In order that is prayed for against them x x x x Suffice it to say that here the service of the summons by publication was ordered by the lower court by virtue of an action quasi in rem against the non-
the court may exercise power over the res, it is not necessary that the court should take actual custody of the property, potential custody thereof being sufficient. There is potential resident defendant.
custody when, from the nature of the action brought, the power of the court over the property is impliedly recognized by law. "An illustration of what we term potential jurisdiction over

the res, is found in the proceeding to register the title of land under our system for the registration of land. Here the court, without taking actual physical control over the property,

assumes, at the instance of some person claiming to be owner, to exercise a jurisdiction in rem over the property and to adjudicate the title in favor of the petitioner against all the MELINA P. MACAHILIG, Petitioner, v. The Heirs of GRACE M. MAGALIT, Respondents.
world."
G.R. No. 141423, November 15, 2000.
(4) As before stated, in an action in rem or quasi in rem against a non-resident defendant, jurisdiction over his person is non-essential, and if the law requires in such case
Topic: Manner of acquiring jurisdiction over the res.
that the summons upon the defendant be served by publication, it is merely to satisfy the constitutional requirement of due process. Facts:

The reason for the rule that Philippine courts cannot acquire jurisdiction over the person of a non-resident, as laid down by the Supreme Court of the United States in Pennoyer v. Neff, On February 5, 1965, Pepito Magalit, deceased husband of Dr. Grace M. Magalit (now substituted by her heirs as respondents in view of her recent demise), filed with the then Philippine
supra, may be found in a recognized principle of public law to the effect that "no State can exercise direct jurisdiction and authority over persons or property without its territory. Story, Confl. Fisheries Commission -- now Bureau of Fisheries and Aquatic Resources (BFAR) -- Fishpond Application No. 24400. The application was for eleven (11) hectares of land situated in the
L., ch. 2; Wheat, Int. L., pt. 2, ch. 2. The several States are of equal dignity and authority, and the independence of one implies the exclusion of power from all others. And so it is laid down Municipality of Batan, Province of Aklan.4 On April 13, 1972, Bernardo Macahilig, deceased husband of petitioner, filed with the BFAR Fishpond Application No. 29972 for five of the eleven
by jurists, as an elementary principle, that the laws of one State have no operation outside of its territory, except so far as is allowed by comity; and that no tribunal established by it can hectares which Magalit had previously applied for.5 On February 28, 1972, BFAR rejected Macahilig’s application for his failure to submit all the requirements.
extend its process beyond that territory so as to subject either persons or property to its decisions. "Any exertion of authority of this sort beyond this limit," says Story, "is a mere nullity, and

incapable of binding such persons or property in any other tribunals." Story, Confl. L., sec. 539." (Pennoyer v. Neff, 95 U.S., 714; 24 Law. ed., 565, 568-569.). Undaunted, Macahilig protested Magalit’s application on November 8, 1976 contending that, for a period of 20 years, he had been in actual possession of the five-hectare area included in

Magalit’s application.
When, however, the action relates to property located in the Philippines, the Philippine courts may validly try the case, upon the principle that a "State, through its tribunals, may subject

property situated within its limits owned by non-residents to the payment of the demand of its own citizens against them; and the exercise of this jurisdiction in no respect infringes upon the On August 22, 1979, the Director of BFAR ordered the Committee on Fishpond Claims and Conflict to hear and determine the rights of Macahilig and Magalit over the disputed area. The
sovereignty of the State where the owners are domiciled. Every State owes protection to its citizens; and, when non-residents deal with them, it is a legitimate and just exercise of authority Committee concluded that the former was merely the latter’s laborer and caretaker. On June 6, 1980, BFAR Director Felix R. Gonzales rendered an Order that: (1) the Letter Protest, dated
to hold and appropriate any property owned by such non-residents to satisfy the claims of its citizens. It is in virtue of the State's jurisdiction over the property of the non-resident situated November 8, 1976, filed by Bernardo Macahilig against Pepito Magalit, is dismissed for lack of merit; (2) Fp. A. No. 29972 of Bernardo Macahilig shall remain rejected and Fp. A. No. 24400
filed by Pepito Magalit should be, as it is hereby, given due course to contain 10.0 hectares only; and (3) the Regional Director of Iloilo City is directed to advise Bernardo Macahilig or other Did the trial court acquire jurisdiction over Lot 4417.

occupants to vacate the premises after the finality of the Order.

Ruling:
Macahilig elevated this disposition to the Office of the President. However, then Presidential Assistant for Legal Affairs Manuel M. Lazaro, by "authority of the President," denied the motion

for reconsideration of Bernardo Macahilig with finality.


Petitioner further contends that the trial court gravely abused its discretion in ordering the turnover of Lot 4417 to Dr. Magalit, because of its earlier ruling that it had no jurisdiction over said

property
Subsequently, Macahilig challenged this action via a "Petition for Review with Prayer for an Issuance of a Writ of Injunction and/or Restraining Order." He had filed the Petition originally with
We cannot place much weight on this Order. First, the September 9, 1992 Motion for Reconsideration taken up in said Order has not been attached to or alleged in the herein Petition.
the Supreme Court, which then referred it to the Intermediate Appellate Court (IAC) where it was docketed as AC-GR SP No. 03448. On March 26, 1985, the appellate court rendered a
Hence, we cannot fully consider the nature of the claim that was denied by this Order or speculate on why the trial court ruled that it had no jurisdiction over the movant’s claim. We cannot
Decision denying and dismissing the petition for review for lack of factual basis, and declaring that Magalit had occupied, cleared and improved the land; and that Macahilig was his mere
even guess which Order the unidentified movant wanted to be reconsidered.
caretaker and laborer, and ordering that the petitioner or anybody acting in his behalf is/are to vacate the subject property in question, and to turn it over to the heirs of Pepito Magalit,

considering that the challenged decision has long become final and executory.
More important, it is too late in the day for petitioner to challenge the jurisdiction of the trial court. She clearly submitted to its authority by her unqualified participation in Civil Case No. 3517.

We cannot allow her to attack its jurisdiction simply because it rendered a Decision prejudicial to her position. Participation in all stages of a case before a trial court effectively estops a party
Later on, Magalit instituted Civil Case No. 3517 in the Regional Trial Court of Kalibo, Aklan, for the issuance of a Writ of Execution. On November 29, 1985, Deputy Provincial Sheriff of Aklan
from challenging its jurisdiction.26 One cannot belatedly reject or repudiate its decision after voluntarily submitting to its jurisdiction, just to secure affirmative relief against one’s opponent or
Eriberto Taytayon Jr. implemented the Writ issued by Judge Jaime D. Discaya on October 30, 1985. The heirs of Pepito Magalit, represented by Dr. Magalit, filed on August 6, 1990, a
after failing to obtain such relief.27 If, by deed or conduct, a party has induced another to act in a particular manner, estoppel effectively bars the former from adopting an inconsistent
"Motion for Correction of the Implementation of the Decision of the Court of Appeals in CA-GR SP No. 03448, promulgated on March 26, 1985, and of the Decision of the Court, dated
position, attitude or course of conduct that thereby causes loss or injury to the latter.28
October 30, 1985, praying that the trial court properly implement said IAC Decision by ordering Spouses Macahilig to turn over to her the possession of Lot 4417, which had an area of

2.0805 hectares, more or less, and contending that the Writ of Execution was not satisfied because the spouses had refused to give up the fishpond in question.
Petitioner insists that the trial court had no jurisdiction over the res of Lot 4417 when it issued its September 17, 1992 Order.

On September 17, 1992, Judge Maria Carillo-Zaldivar issued the following Order that (1) finding from the Manifestation of counsel on record for the movant that the two (2) hectares of land
Again, we disagree. Jurisdiction over the res is acquired either (a) by the seizure of the property under legal process, whereby it is brought into actual custody of the law; or (b) as a result of
she desires to be executed thru an alias writ is outside the ten (10) hectares awarded to her by the Fisheries, and (2) this Court has no jurisdiction over her claim.
the institution of legal proceedings, in which the power of the court is recognized and made effective.29 In the latter condition, the property, though at all times within the potential power of

the court, may not be in the actual custody of said court.


On October 9, 1992, Dr. Magalit filed a "Petition for Contempt Against Melina Macahilig," alleging that on November 29, 1985, Bernardo Macahilig had refused to turn over Lot 4417 to her.

The trial court appointed a commissioner to determine whether Lot 4417 was included in the parcels of land awarded to the deceased Magalit. Sheriff Nelson R. dela Cruz, the appointed
The trial court acquired jurisdiction over the disputed lot by virtue of the institution of the Petition for a Writ of Execution filed by the respondents’ predecessors in interest. Without taking
commissioner, submitted his Commissioner’s Report dated May 13, 1993, pertinent portions of which read (1) Petitioner Dr. Grace M. Magalit is in the actual possession of Lot-A with an
actual physical control of the property, it had an impliedly recognized potential jurisdiction or potential custody over the res. This was the jurisdiction which it exercised when it issued the Writ
area of 9.9 has., which is a fully developed fishpond, and without any question from the respondent, (2) however, as per Order of the Bureau of Fisheries and Aquatic Resources, dated June
of Execution directing the surrender of Lot 4417 to Dr. Magalit.
6, 1980, that portion labelled Parcel-A in the sketch with an area of 2.3 has. has to be excluded because it is needed for forest purposes----so if we deduct 2.3 has. from the 9.9 has. the

remaining area for the Petitioner will only be 7.6 has.----that is why the Bureau of Fisheries has to include Lot 4417 and Lot 5216, in order that the area of 10.0 hectares in said order will be
SAMAHAN NG MGA MANGGAGAWA SA HYATT (SAMASAH-NUWHRAIN), Petitioner,
satisfied." vs.
HOTEL ENTERPRISES OF THE PHILIPPINES, INC., Respondent.
G.R. No. 172303
Topic: Nature and purpose of procedural law
In the Order of June 18, 1993, the trial court adopted the Commissioner’s Report and ruled in favor of Dr. Magalit (1) finding from the Commissioner’s Report and the sketch submitted that

the questioned Lot 4417 with an area of 20,805 square meters is actually in the possession of Petitioner Melina Macahilig but which was given to the Respondent’s husband, and (2) letting a
Rules of procedure exist for a noble purpose, and to disregard such rules in the guise of liberal construction would be to defeat such purpose. Procedural rules are not to be disdained as
writ of execution be issued in favor of the Respondent and against Petitioner Melina Macahilig for the delivery of Lot 4417. mere technicalities. They may not be ignored to suit the convenience of a party. Adjective law ensures the effective enforcement of substantive rights through the orderly and speedy
administration of justice. Rules are not intended to hamper litigants or complicate litigation. But they help provide for a vital system of justice where suitors may be heard following judicial
procedure and in the correct forum. Public order and our system of justice are well served by a conscientious observance by the parties of the procedural rules.
Petitioner moved for reconsideration, but her motion was denied in the Order of July 14, 1993, which held that she had no valid reason to possess the disputed lot, considering that her FACTS: Petitioner is a duly registered union and the certified bargaining representative of the rank-and-file employees of Hyatt Regency Manila, a five-star hotel owned and operated by
respondent Hotel Enterprises of the Philippines, Inc. On January 31, 2001, Hyatt’s General Manager, David C. Pacey, issued a Memorandum informing all hotel employees that hotel
husband’s application therefor had been rejected. security have been instructed to conduct a thorough bag inspection and body frisking in every entrance and exit of the hotel.
From February 3, 2001 to March 2, 2001 Angelito Caragdag, a waiter at the hotel’s Cafe Al Fresco restaurant and a director of the union, committed multiple infraction such as refusing to be
frisked by the security personnel, interrupting and intimidating his superior, and leaving work assignment during official working hours
Unfazed by the unfavorable turn of events, Petitioner filed with the CA, on August 12, 1993, a Petition for Certiorari alleging that the trial court had acted with grave abuse of discretion in Because of the succession of infractions he committed, An investigation board was formed, and the matter was set for hearing on May 19, 2001. However, despite notice of the scheduled
hearing, both Caragdag and the Union President failed to attend. Thereafter, the investigating board resolved on the said date to dismiss Caragdag. Caragdag appealed but the investigating
issuing the Orders dated June 18 and July 14, 1993. board affirmed its resolution.
Caragdag’s dismissal was questioned by petitioner, and the dispute was referred to voluntary arbitration upon agreement of the parties.
Arbitrator’s ruling:
The Arbitrator affirmed the decision of the investigating board reasoning that the union officers and members had no right to breach company rules and regulations on
The CA ruled that the trial court did not commit grave abuse of discretion when it issued a Writ of Execution ordering the delivery of Lot 4417 to Dr. Magalit. The records show that the
security and employee discipline on the basis of certain suspicions against management and an ongoing CBA negotiation standoff. The Voluntary Arbitrator also found that when
fishpond application of petitioner’s husband was rejected by the BFAR, and that petitioner did not present any other evidence to prove her right of possession over the disputed property. On Caragdag advised Lacambacal and Alvaro not to give any statement, he threatened and intimidated his superior while the latter was performing his duties. Moreover, there is no
reason why he did not arrange his time-off with the Department Head concerned.
the other hand, Dr. Magalit’s claim was based on the Decision in the Fishpond Case, which upheld her right----as the surviving spouse of the applicant----to possess the ten hectares of land Petitioner sought reconsideration of the decision while respondent filed a motion for partial reconsideration. However, the Voluntary Arbitrator denied both motions on May
26, 2003.
awarded to him, including Lot 4417 which covered an area of more or less 2.0805 hectares. The disputed lot was included in the area awarded to Dr. Magalit because of the report of the CA’s ruling: On the Petition for Certiorari of petitioners
The CA dismissed the petition outright for being the wrong remedy. The CA explained:
commissioner appointed by the trial court to settle the issue. Petitioner had not objected either to the said appointment or to the Report.
Final orders or resolution of voluntary arbitrators is through a Petition for Review which should be filed within fifteen (15) days from the receipt of notice of judgment, order or
resolution of the voluntary arbitrator. Considering that petitioner intends this petition to be a Petition for Certiorari, the Court hereby resolves to dismiss the petition outright for
being an improper mode of appeal.
The Orders of June 18 and July 14, 1993 were based on the evidence presented before the trial court. Consequently, they cannot be regarded as capricious and whimsical exercises of Petitioner duly filed a motion for reconsideration of the dismissal, but the motion was denied by the CA.
Petitioner filed before the SC a petition for review on certiorari.
judicial power. Party arguments:

Hence, this Petition for Review under Rule 45 of the Rules of Court, assailing the Decision1 dated October 15, 1999, and the Resolution dated December 28, 1999, issued by the Court of Petitioner points out, that Rule 43 expressly provides "shall not apply to judgments or final orders issued under the Labor Code of the Philippines." Hence, a petition for
certiorari under Rule 65 is the proper remedy for questioning the decision of the Voluntary Arbitrator.
Appeals (CA) in CA-GR SP No. 31809.
On the other hand, respondent maintains that Section 1 of Rule 43 clearly states that it is the governing rule with regard to appeals from awards, judgments, final orders or
resolutions of voluntary arbitrators.
ISSUE:
WON the CA erred in dismissing outright the petition for certiorari filed before it on the ground that the same is an improper mode of appeal;
Issue:
RULING:
The question on the proper recourse to assail a decision of a voluntary arbitrator has already been settled in Luzon Development Bank v. Association of Luzon Development Bank Remedial rights are those rights granted by remedial or procedural laws. These are rights that only operate to further the rules of procedure or to confirm vested rights. As such, the
Employees, where the Court held that the decision or award of the voluntary arbitrator or panel of arbitrators should likewise be appealable to the Court of Appeals by a petition for certiorari retroactive application of remedial rights will not adversely affect the vested rights of any person. Considering that the right to appeal is a right remedial in nature, we find that Section 4,
under Rule 43, just like those of the quasi-judicial agencies, boards and commissions enumerated therein, and consistent with the original purpose to provide a uniform procedure for the paragraph (k), Rule I of the RACCS applies in this case. Petitioner, therefore, had the right to appeal the decision of the Civil Service Commission that modified its original
appellate review of adjudications of all quasi-judicial entities. decision of dismissal.
Thus, we now hold that the parties adversely affected by a decision in an administrative case who may appeal shall include the disciplining authority whose decision
dismissing the employee was either overturned or modified by the Civil Service Commission.
Hence, upon receipt on May 26, 2003 of the Voluntary Arbitrator’s Resolution denying petitioner’s motion for reconsideration, petitioner should have filed with the CA, within the fifteen (15)-
day reglementary period, a petition for review, not a petition for certiorari. JUDITH YU, Petitioner,
Petitioner insists on a liberal interpretation of the rules but we find no cogent reason in this case to deviate from the general rule. Verily, rules of procedure exist for a noble purpose, and vs.
to disregard such rules in the guise of liberal construction would be to defeat such purpose. Procedural rules are not to be disdained as mere technicalities. They may not be HON. ROSA SAMSON-TATAD, Presiding Judge, Regional Trial Court, Quezon City, Branch 105, and the PEOPLE OF THE PHILIPPINES, Respondents.
ignored to suit the convenience of a party. Adjective law ensures the effective enforcement of substantive rights through the orderly and speedy administration of justice. G.R. No. 170979, February 9, 2011
Rules are not intended to hamper litigants or complicate litigation. But they help provide for a vital system of justice where suitors may be heard following judicial procedure
and in the correct forum. Public order and our system of justice are well served by a conscientious observance by the parties of the procedural rules. Topic: Fresh Period Rule; Applies in Criminal Procedure

WHEREFORE, the petitions for review on certiorari are DENIED. While Neypes involved the period to appeal in civil cases, the Court’s pronouncement of a "fresh period" to appeal should equally apply to the period for appeal in criminal cases.
Were we to strictly interpret the "fresh period rule" in Neypes and make it applicable only to the period to appeal in civil cases, we shall effectively foster and encourage an absurd situation
where a litigant in a civil case will have a better right to appeal than an accused in a criminal case – a situation that gives undue favor to civil litigants and unjustly discriminates against the
accused-appellants. It suggests a double standard of treatment when we favor a situation where property interests are at stake, as against a situation where liberty stands to be prejudiced.
We must emphatically reject this double and unequal standard for being contrary to reason.
FACTS:
Based on the complaint of Spouses Sergio and Cristina Casaclang, an information for estafa against the petitioner was filed with the RTC.
In a May 26, 2005 decision, the RTC convicted the petitioner as charged.
Fourteen (14) days later, or on June 9, 2005, the petitioner filed a motion for new trial with the RTC, alleging that she discovered new and material evidence that would exculpate her of the
crime for which she was convicted.
In an October 17, 2005 order, respondent Judge denied the petitioner’s motion for new trial for lack of merit.
LIGHT RAIL TRANSIT AUTHORITY, represented by its Administrator MELQUIADES A. ROBLES, Petitioner, On November 16, 2005, the petitioner filed a notice of appeal with the RTC, alleging that pursuant to our ruling in Neypes v. Court of Appeals, she had a "fresh period" of 15 days from
vs. November 3, 2005, the receipt of the denial of her motion for new trial, or up to November 18, 2005, within which to file a notice of appeal.
AURORA A. SALVAÑA, Respondent. On December 8, 2005, the prosecution filed a motion to dismiss the appeal for being filed 10 days late, arguing that Neypes is inapplicable to appeals in criminal cases.
G.R. No. 192074, June 10, 2014 The Petition
Topic: Retroactive application of procedural rules The petitioner argues that the RTC lost jurisdiction to act on the prosecution’s motions when she filed her notice of appeal within the 15-day reglementary period provided by the Rules of
It is true that under the Civil Code of the Philippines, "(l)aws shall have no retroactive effect, unless the contrary is provided. But there are settled exceptions to this general rule, Court, applying the "fresh period rule" enunciated in Neypes.
such as when the statute is CURATIVE or REMEDIAL in nature or when it CREATES NEW RIGHTS. The Case for the Respondents
On the other hand, remedial or procedural laws, i.e., those statutes relating to remedies or modes of procedure, which do not create new or take away vested rights, but only operate in In their comment, the Spouses Casaclang aver that the petitioner cannot seek refuge in Neypes to extend the "fresh period rule" to criminal cases because Neypes involved a civil case, and
furtherance of the remedy or confirmation of such rights, ordinarily do not come within the legal meaning of a retrospective law, nor within the general rule against the retrospective operation the pronouncement of "standardization of the appeal periods in the Rules" referred to the interpretation of the appeal periods in civil cases, i.e., Rules 40, 41, 42 and 45, of the 1997 Rules of
of statutes. Civil Procedure among others; nowhere in Neypes was the period to appeal in criminal cases, Section 6 of Rule 122 of the Revised Rules of Criminal Procedure, mentioned.
FACTS: On May 12, 2006, then Administrator of the Light Rail Transit Authority, Melquiades Robles revoked Atty. Aurora A. Salvaña’s designation as Officer-in-Charge (OIC) of the LRTA ISSUE:
Administrative Department. The core issue boils down to whether the "fresh period rule" enunciated in Neypes applies to appeals in criminal cases.
Atty. Salvaña was directed to comply with this office order through a memorandum issued on May 22, 2006 by Atty. Elmo Stephen P. Triste, the newly designated OIC of the administrative RULING:
department. Instead of complying, Salvaña questioned the order with the Office of the President. In Neypes, the Court modified the rule in civil cases on the counting of the 15-day period within which to appeal. The Court categorically set a fresh period of 15 days from a denial of a
In the interim, Salvaña applied for sick leave of absence on May 12, 2006 and from May 15 to May 31, 2006. In support of her application, she submitted a medical certificate issued by Dr. motion for reconsideration within which to appeal, thus:
Grace Marie Blanco of the Veterans Memorial Medical Center (VMMC). The Supreme Court may promulgate procedural rules in all courts. It has the sole prerogative to amend, repeal or even establish new rules for a more simplified and inexpensive process,
LRTA discovered that Dr. Blanco did not issue this medical certificate. Dr. Blanco also denied having seen or treated Salvaña. On June 23, 2006, Administrator Robles issued a notice of and the speedy disposition of cases. In the rules governing appeals to it and to the Court of Appeals, particularly Rules 42, 43 and 45, the Court allows extensions of time, based on
preliminary investigation. justifiable and compelling reasons, for parties to file their appeals. These extensions may consist of 15 days or more.
Because of this Salvaña was found guilty of all the charges against her and imposed on her the penalty of dismissal from service with all the accessory penalties." Henceforth, this "fresh period rule" shall also apply to Rule 40 governing appeals from the Municipal Trial Courts to the Regional Trial Courts; Rule 42 on petitions for review
Civil Service Commission: from the Regional Trial Courts to the Court of Appeals; Rule 43 on appeals from quasi-judicial agencies to the Court of Appeals and Rule 45 governing appeals by certiorari to
Salvaña appealed with the Civil Service Commission. "In her appeal, she claimed that she was denied due process and that there was no substantial evidence to support the charges the Supreme Court. The new rule aims to regiment or make the appeal period uniform, to be counted from receipt of the order denying the motion for new trial, motion for reconsideration
against her." (whether full or partial) or any final order or resolution.
The Civil Service Commission found that Salvaña was guilty only of simple dishonesty. While Neypes involved the period to appeal in civil cases, the Court’s pronouncement of a "fresh period" to appeal should equally apply to the period for appeal in criminal cases
CA: under Section 6 of Rule 122 of the Revised Rules of Criminal Procedure.
On November 11, 2009, the Court of Appeals dismissed the petition and affirmed the Civil Service Commission’s finding that Salvaña was only guilty of simple dishonesty. Were we to strictly interpret the "fresh period rule" in Neypes and make it applicable only to the period to appeal in civil cases, we shall effectively foster and encourage an absurd situation
The appellate court also ruled that Administrator Robles had no standing to file a motion for reconsideration before the Civil Service Commission because that right only belonged to where a litigant in a civil case will have a better right to appeal than an accused in a criminal case – a situation that gives undue favor to civil litigants and unjustly discriminates against the
respondent in an administrative case. LRTA moved for reconsideration of this decision but was denied. accused-appellants. It suggests a double standard of treatment when we favor a situation where property interests are at stake, as against a situation where liberty stands to be prejudiced.
Party arguments: We must emphatically reject this double and unequal standard for being contrary to reason. Over time, courts have recognized with almost pedantic adherence that what is contrary to
Petitioner argues that it has the legal personality to appeal the decision of the Civil Service Commission before the Court of Appeals. It cites Philippine National Bank v. Garcia as basis reason is not allowed in law – Quod est inconveniens, aut contra rationem non permissum est in lege.
for its argument that it can be considered a "person adversely affected" under the pertinent rules and regulations on the appeal of administrative cases. It also argues that respondent’s WHEREFORE, the petition for prohibition is hereby GRANTED.
falsification of the medical certificate accompanying her application for sick leave was not merely simple but serious dishonesty.
Respondent argues the opposite.
Fact relative to the case:
Previous definition: PARTY ADVERSELY AFFECTED refers to the respondent against whom a decision in a disciplinary case has been rendered or to the disciplining authority in an
appeal from a decision exonerating(only) the said employee. (emphasis supplied) San Lorenzo Ruiz Builders and Developers Group, Inc. vs Ma. Cristina F. Bayang
During the pendency of this decision, or on November 18, 2011, the Revised Rules on Administrative Cases in the Civil Service or RACCS was promulgated. The Civil Service G.R. No. 194702, April 20, 2015
Commission modified the definition of a "party adversely affected" for purposes of appeal.
It now reads: PARTY ADVERSELY AFFECTED refers to the respondent against whom a decision in an administrative case has been rendered or to the disciplining authority in an Topic: Fresh Period Rule; not apply to administrative appeals
appeal from a decision reversing or modifying the original decision. It is settled that the "fresh period rule" in Neypes applies only to judicial appeals and not to administrative appeals.
Note: so the question now stands is if LRTA, in a non-exonerating case, may appeal as a PARTY ADVERSELY AFFECTED as newly defined despite that this change in
definition was done after the case was brought into court. FACTS: On April 15, 2000, petitioner SLR Builders (then known as Violago Builders, Inc), as seller, and respondent Ma. Cristina F. Bayang ( Cristina), as buyer, entered into a "contract to
ISSUE: sell" of a sixty (60)-square meter lot in Violago Homes Parkwoods Subdivision, located in Barangay Payatas, Quezon City.
Whether the LRTA, as represented by its Administrator, has the standing to appeal the modification by the Civil Service Commission of its decision
RULING: Upon full payment of the monthly amortizations on the purchased lot, Cristina demanded from SLR Builders the execution of the deed of absolute sale and the lot's certificate of title but the
In previous cases, or as the rule that stood during the time that the case was tried was that disciplining authority were only allowed to appeal from a decision exonerating an employee. In latter failed to deliver, prompting Cristina to file a complaint for specific performance and damages against SLR Builders and its President, Oscar Violago ( petitioners) before the Housing
this case, respondent was not exonerated; she was found guilty, but the finding was modified. and Land Use Regulatory Board (HLURB).
The LRTA had standing to appeal the modification by the Civil Service Commission of its decision
The employer has the right "to select honest and trustworthy employees." When the government office disciplines an employee based on causes and procedures allowed by law, it exercises HLURB Ruling:
its discretion. This discretion is inherent in the constitutional principle that "[p]ublic officers and employees must, at all times, be accountable to the people, serve them with utmost Ruled in favor of respondent.
responsibility, integrity, loyalty, and efficiency; act with patriotism and justice, and lead modest lives." This is a principle that can be invoked by the public as well as the government office
employing the public officer. OP (Office of the President) ruling:
Here, petitioner already decided to dismiss respondent for dishonesty. Dishonesty is a serious offense that challenges the integrity of the public servant charged. To bar a government office The OP dismissed the petitioners' appeal for having been filed out of time. The OP's resolution stated:
from appealing a decision that lowers the penalty of the disciplined employee prevents it from ensuring its mandate that the civil service employs only those with the utmost sense of . . .Arbiter's decision was received by the respondents/appellants (referring to the petitioners) on July 27, 2005. On that date, the 15-day prescriptive period within which to file an
responsibility, integrity, loyalty, and efficiency. appeal began to run. Instead of preparing an appeal, respondents-appellants opted to file a Motion for Reconsideration on August 10, 2005. Their filing of the said motion
Honesty and integrity are important traits required of those in public service. If all decisions by quasi-judicial bodies modifying the penalty of dismissal were allowed to become final and interrupted the period of appeal by that time, however, fourteen (14) days had already elapsed.
unappealable, it would, in effect, show tolerance to conduct unbecoming of a public servant. The quality of civil service would erode, and the citizens would end up suffering for it.
On the application of the amendment to the present case: On April 17, 2006, respondents-appellants received the Resolution denying their Motion for Reconsideration. Following the above rules, respondents-appellants have only one
Procedural laws have retroactive application. In Zulueta v. Asia Brewery: (1) day left, or until April 18, 2006, within which to file their notice of appeal. Unfortunately, they were able to do so only on April 27, 2006, or nine (9) days late.
As a general rule, laws have no retroactive effect. But there are certain recognized exceptions, such as when they are remedial or procedural in nature. This Court explained this exception in
the following language: The petitioners moved to reconsider and argued that the "fresh period rule" enunciated in the case of Domingo Neypes, et at. v. Court of Appeals, et al. should be applied to their case.
It is true that under the Civil Code of the Philippines, "(l)aws shall have no retroactive effect, unless the contrary is provided. But there are settled exceptions to this general rule,
such as when the statute is CURATIVE or REMEDIAL in nature or when it CREATES NEW RIGHTS. The OP, in a resolution dated July 26, 2007, denied the petitioners' motion with finality, stating that the "fresh period rule" applies only to judicial appeals and not to administrative appeals ,
On the other hand, remedial or procedural laws, i.e., those statutes relating to remedies or modes of procedure, which do not create new or take away vested rights, but only operate in such as in petitioners' case.
furtherance of the remedy or confirmation of such rights, ordinarily do not come within the legal meaning of a retrospective law, nor within the general rule against the retrospective operation CA Ruling:
of statutes. Denied the petitioners' petition for review. The CA, likewise, denied the petitioners' motion for reconsideration; hence, the filing of the present petition for review on certiorari with this Court.
Thus, procedural laws may operate retroactively as to pending proceedings even without express provision to that effect. Accordingly, rules of procedure can apply to cases pending at the
time of their enactment. In fact, statutes regulating the procedure of the courts will be applied on actions undetermined at the time of their effectively. Procedural laws are retrospective in that ISSUE:
sense and to that extent. (Emphasis supplied)
The petitioner filed its motion for reconsideration on January 14, 2013, which was 31 days after receiving the assailed decision of the COA on December 14, 2012. Pursuant to Section 3 of
Whether the "fresh period rule" in Neypes applies to administrative appeals, such as an appeal filed from a decision of the HLURB Board of Commissioners to the Office to the President. Rule 64, it had only five days from receipt of the denial of its motion for reconsideration to file the petition. Considering that it received the notice of the denial on July 14, 2014, it had only
until July19, 2014 to file the petition. However, it filed the petition on August 13, 2014, which was 25 days too late.
RULING: We ruled in Pates v. Commission on Elections that the belated filing of the petition for certiorari under Rule 64 on the belief that the fresh period rule should apply was fatal to the recourse.
We DENY the petition. It is settled that the "fresh period rule" in Neypes applies only to judicial appeals and not to administrative appeals. As such, the petitioner herein should suffer the same fate for having wrongly assumed that the fresh period rule under Neypes applied. Rules of procedure may be relaxed only to relieve
The "fresh period rule" in Neypes declares: a litigant of an injustice that is not commensurate with the degree of his thoughtlessness in not complying with the prescribed procedure. Absent this reason for liberality, the
petition cannot be allowed to prosper.
To standardize the appeal periods provided in the Rules and to afford litigants fair opportunity to appeal their cases, the Court deems it practical to allow a fresh period of 15 days within
which to file the notice of appeal in the Regional Trial Court, counted from receipt of the order dismissing a motion for a new trial or motion for reconsideration.
Milagrosa Jocson vs Nelson San Miguel
Henceforth, this "fresh period rule" shall also apply to Rule 40 governing appeals from the Municipal Trial Courts to the Regional Trial Courts; Rule 42 on petitions for review from the G.R. No. 206941, March 9, 2016
Regional Trial Courts to the Court of Appeals; Rule 43 on appeals from quasi-judicial agencies to the Court of Appeals; and Rule 45 governing appeals by certiorari to the Supreme Court.
The new rule aims to regiment or make the appeal period uniform, to be counted from receipt of the order denying the motion for new trial, motion for reconsideration (whether full or partial) Topic: Fresh Period Rule; not apply to administrative rules
or any final order or resolution.
Obviously, these Rules cover judicial proceedings under the 1997 Rules of Civil Procedure. In the present case, the appeal from a decision of the Provincial Adjudicator to the DARAB as provided for under Section 1, Rule XIV of the 2003 DARAB Rules of Procedure, is not judicial
but administrative in nature. As such, the "fresh period rule" in Neypes finds no application therein.

As a final note, it is worthy to emphasize that the right to appeal is not a natural right or a part of due process, but is merely a statutory privilege that may be exercised only in the manner
prescribed by law. The right is unavoidably forfeited by the litigant who does not comply with the manner thus prescribed. In addition, the liberal application of rules of procedure for
What was the proper action? perfecting appeals is still the exception, and not the rule; and it is only allowed in exceptional circumstances to better serve the interest of justice. [36] This exceptional situation, however, does
not obtain in this case.
Petitioner's present case is administrative in nature involving an appeal from the decision or order of the DENR regional office to the DENR Secretary. Such appeal is indeed governed by
Section 1 of Administrative Order No. 87, Series of 1990. As earlier quoted, Section 1 clearly provides that if the motion for reconsideration is denied, the movant shall perfect his appeal FACTS: On September 10, 2008, Milagrosa C. Jocson (Jocson) filed with the DARAB-PARAD, Region III of San Fernando City, Pampanga, a Complaint for ejectment with damages against
"during the remainder of the period of appeal, reckoned from receipt of the resolution of denial;" whereas if the decision is reversed, the adverse party has a fresh 15-day period to perfect respondent Nelson San Miguel (San Miguel) and all persons claiming rights under him.
his appeal.
In the Complaint, Jocson alleged that she is the registered owner of a parcel of agricultural land, located in Magalang, Pampanga. She asserted that 56,000 sq m thereof became the subject
In this case, the subject appeal, i.e., appeal from a decision of the HLURB Board of Commissioners to the OP, is not judicial but administrative in nature; thus, the "fresh period rule" in of an Agricultural Leasehold Contract (Contract) between her and San Miguel, with the latter as tenant-lessee. As part of the contract, they agreed that the subject landholding shall be
Neypes does not apply. devoted to sugar and rice production.

As aptly pointed out by the OP, the rules and regulations governing appeals from decisions of the HLURB Board of Commissioners to the OP are Section 2, Rule XXI of HLURB Resolution According to Jocson, San Miguel, however, occupied the entire landholding and refused to vacate the portion not covered by their Contract despite repeated demands.
No. 765, series of 2004, in relation to Paragraph 2, Section 1 of Administrative Order No. 18, series of 1987:
Section 2, Rule XXI of the HLURB Resolution No. 765, series of 2004, prescribing the rules and regulations governing appeals from decisions of the Board of Commissioners to the Office of On December 15, 2009, Jocson filed a Supplemental Complaint alleging that, during the pendency of the present suit, San Miguel commenced to plant corn on the subject landholding which
the President, pertinently reads: violated their Contract.

Section 2. Appeal. - Any party may, upon notice to the Board and the other party, appeal a decision rendered by the Board of Commissioners to the Office of the President within fifteen (15) In his Answer, San Miguel maintained that he had religiously complied with all the terms and conditions of their Contract and that Jocson has no valid ground to eject him from the disputed
days from receipt thereof, in accordance with P.D. No. 1344 and A.O. No. 18 Series of 1987. landholding.

The pendency of the motion for reconsideration shall suspend the running of the period of appeal to the Office of the President. PARAD Decision

Corollary thereto, paragraph 2, Section 1 of Administrative Order No. 18, series of 1987, provides that in case the aggrieved party files a motion for reconsideration from an adverse The PARAD (Provincial Agrarian Reform Adjudicator) ruled in favor of petitioner.
decision of any agency/office, the said party has the only remaining balance of the prescriptive period within which to appeal, reckoned from receipt of notice of the decision
denying his/her motion for reconsideration.

Thus, in applying the above-mentioned rules to the present case, we find that the CA correctly affirmed the OP in dismissing the petitioners' appeal for having been filed out of time. San Miguel filed a Motion for Reconsideration but it was denied in an Order.
On June 15, 2011, San Miguel filed his Notice of Appeal. It was subsequently denied because it was filed out of time according to PARAD applying the DARAB Rules of Procedure which
WHEREFORE, we DENY the present petition for review on certiorari and AFFIRM. only gives a petitioner five (5) days to file an appeal.

Note: The PARAD found that San Miguel, through his counsel, received his copy of Decision dated January 26, 2011 on February 3, 2011 and thereafter filed his MR on
February 15, 2011, thus, he could have only three (3) days within which to file his Notice of Appeal upon its denial. The MR was denied on May 31, 2011 and San Miguel,
through his counsel, received his copy of the Order on June 2, 2011 and he filed his Notice of Appeal on June 15, 2011 or after twelve (12) days, which, following the rules
abovementioned, is already beyond the period allowed.
FORTUNE LIFE INSURANCE COMPANY, INC., Petitioner,
vs. Undaunted, San Miguel filed a Petition for Certiorari with the CA praying that the new 2009 DARAB Rules of Procedure which adopted the "fresh period rule" should apply to the case.
COMMISSION ON AUDIT (COA) PROPER; COA REGIONAL OFFICE NO. VI-WESTERN VISAYAS; AUDIT GROUP LGS-B, PROVINCE OF ANTIQUE; AND PROVINCIAL
GOVERNMENT OF ANTIQUE, Respondents. Note: In September 1, 2009 or during the pendency of the case the New 2009 DARAB Rules of Procedure was enacted hence the prayer.
G.R. No. 213525, January 27, 2015
CA Ruling:
Topic: Fresh Period Rule; not apply to Rule 64
We ruled in Pates v. Commission on Elections that the belated filing of the petition for certiorari under Rule 64 on the belief that the fresh period rule should apply was fatal to the recourse. The CA issued a decision granting San Miguel's petition and remanding the case to the DARAB-PARAD for further proceedings. The CA held that the "fresh period rule" enunciated in
As such, the petitioner herein should suffer the same fate for having wrongly assumed that the fresh period rule under Neypes applied. Rules of procedure may be relaxed only to relieve Neypes should be applied in the instant case.
a litigant of an injustice that is not commensurate with the degree of his thoughtlessness in not complying with the prescribed procedure. Absent this reason for liberality, the
petition cannot be allowed to prosper. ISSUE:
FACTS:
Respondent Provincial Government of Antique (LGU) and the petitioner executed a memorandum of agreement concerning the life insurance coverage of qualified barangay secretaries, WON the new DARAB Rules of Procedure should apply.
treasurers and tanod, the former obligating ₱4,393,593.60 for the premium payment, and subsequently submitting the corresponding disbursement voucher to COA Antique for pre-audit. WON the “fresh period rule” should apply.
The latter office disallowed the payment for lack of legal basis. Respondent LGU appealed but its appeal was denied.
COA Ruling: RULING:
Consequently, the petitioner filed its petition for money claim in the COA. On November 15, 2012, the COA issued its decision denying the petition, holding that only municipal or city
governments are expressly vested with the power to secure group insurance coverage for barangay workers. Application of the 2003 DARAB Rules of Procedure
The petitioner received a copy of the COA decision on December 14, 2012, and filed its motion for reconsideration on January 14, 2013. However, the COA denied the motion, the denial
being received by the petitioner on July 14, 2014. San Miguel alleged that due to the effectivity of the 2009 DARAB Rules of Procedure, its provisions should be applied instead of the 2003 DARAB Rules of Procedure.
CA Ruling:
The petition for certiorari was dismissed because of The SC ruled in the negative.
(a) the late filing of the petition;
(b) the non-submission of the proof of service and verified declaration; and It must be noted that Section 1, Rule XXIV of the 2009 DARAB Rules of Procedure explicitly states that:
(c) the failure to show grave abuse of discretion on the part of the respondents.
Sec. 1. Transitory Provisions. These Rules shall govern all cases filed on or after its effectivity. All cases pending with the Board and the Adjudicators, prior to the date of effectivity of
these Rules, shall be governed by the DARAB Rules prevailing at the time of their filing.

Thus, pursuant to the above-cited rule, the applicable rule in the counting of the period for filing a Notice of Appeal with the Board is governed by Section 12, Rule X of the 2003 DARAB
Rules of Procedure, which states that:
ISSUES:
In its motion for reconsideration, the petitioner submits that it filed the petition for certiorari within the reglementary period following the fresh period rule enunciated in Neypes v. Court of
The filing of the Motion for Reconsideration shall interrupt the period to perfect an appeal. If the motion is denied, the aggrieved party shall have the remaining period within which to perfect
Appeals.
his appeal. Said period shall not be less than five (5) days in any event, reckoned from the receipt of the notice of denial.
RULING:
The petitioner posits that the fresh period rule applies because its Rule 64 petition is akin to a petition for review brought under Rule 42 of the Rules of Court; hence, conformably with the
Application of the "fresh period rule" enunciated in the Neypes ruling
fresh period rule, the period to file a Rule 64 petition should also be reckoned from the receipt of the order denying the motion for reconsideration or the motion for new trial.
The petitioner’s position cannot be sustained.
This Court likewise finds no merit to San Miguel's contention that the "fresh period rule" laid down in Neypes is applicable in the instant case.
There is no parity between the petition for review under Rule 42 and the petition for certiorari under Rule 64.
The reglementary periods under Rule 42 and Rule 64 are different. In the former, the aggrieved party is allowed 15 days to file the petition for review from receipt of the assailed decision or
In Panolino, this Court held that the "fresh period rule" only covers judicial proceedings under the 1997 Rules of Civil Procedure, to wit:
final order, or from receipt of the denial of a motion for new trial or reconsideration (fresh period rule). In the latter, the petition is filed within 30 days from notice of the judgment or final order
or resolution sought to be reviewed. The filing of a motion for new trial or reconsideration, if allowed under the procedural rules of the Commission concerned, interrupts the period; hence,
The "fresh period rule" in Neypes declares:
should the motion be denied, the aggrieved party may file the petition within the remaining period, which shall not be less than five days in any event, reckoned from the notice of denial.
(fresh period rule not applied)
xxxx
As reflected in the above-quoted portion of the decision in Neypes, the "fresh period rule" shall apply to Rule 40 (appeals from the Municipal Trial Courts to the Regional Trial
Courts); Rule 41 (appeals from the Regional Trial Courts to the [CA] or Supreme Court); Rule 42 (appeals from the Regional Trial Courts to the [CA]); Rule 43 (appeals from quasi- Party List Buhay Legarda, Loren
judicial agencies to the [CA]); and Rule 45 (appeals by certiorari to the Supreme Court). Obviously, these Rules cover judicial proceedings under the 1997 Rules of Civil
Procedure.
Party List Ang Pamilya Party List Gabriela
In the present case, the appeal from a decision of the Provincial Adjudicator to the DARAB as provided for under Section 1, Rule XIV of the 2003 DARAB Rules of Procedure, is not judicial
but administrative in nature. As such, the "fresh period rule" in Neypes finds no application therein.
Party List Akbayan
As correctly observed by PARAD, San Miguel should perfect his appeal during the remainder of the period of appeal, but not less than five (5) days, reckoned from receipt of the resolution of
denial of his MR or until June 7, 2011.

As a final note, it is worthy to emphasize that the right to appeal is not a natural right or a part of due process, but is merely a statutory privilege that may be exercised only in the manner Party List Bayan Muna
prescribed by law. The right is unavoidably forfeited by the litigant who does not comply with the manner thus prescribed. In addition, the liberal application of rules of procedure for
perfecting appeals is still the exception, and not the rule; and it is only allowed in exceptional circumstances to better serve the interest of justice. This exceptional situation, however, does
not obtain in this case. Party List Anak Pawis

WHEREFORE, in consideration of the foregoing disquisitions, the petition is hereby GRANTED.

Those who voted for the passing of the law were classified by petitioners as comprising "Team Patay," while those who voted against it form "Team Buhay"
On February 27, 2013, COMELEC Law Department issued a letter ordering the immediate removal of the tarpaulin; otherwise, it will be constrained to file an election offense against
petitioners.
THE UNITED STATES, Plaintiff-Appellee, v. JOSE TAMPARONG ET AL., Defendants-Appellants. Petitioners initiated this case through this petition for certiorari and prohibition with application for preliminary injunction and temporary restraining order questioning the constitutionality of the
G.R. No. 9527, August 23, 1915 COMELEC’s order.

Topic: Philippine Courts On March 13, 2013, respondents filed their comment arguing that petitioners violated the doctrine of hierarchy of courts in directly filing their petition before this court.

Under the Spanish criminal procedure, appeals from justices’ courts were allowed only to Courts of First Instance. By section 43 of General Orders No. 58, this procedure has been so
amended that appeals can be taken to the Supreme Court in such cases when the validity or constitutionality of a statute is involved. This amendment of the procedure does not carry with it ISSUE:
the right of review of the facts, but is confined to the purpose stated — that is, of determining the validity or constitutionality of the statute or ordinance upon which the judgment was WON petitioners violated the doctrine of hierarchy of courts in directly filing their petition before this court.
predicated. Former cases reviewed, showing that such has uniformly been the interpretation of section 43 by this court.
FACTS:
The defendants were convicted by the justice of the peace of Baguio for having played the game of chance called "monte" in violation of Ordinance No. 35. They appealed to the Court of
RULING:
First Instance, where they were again tried and convicted upon the same charge. An appeal was allowed to this court because the validity of Ordinance No, 35 was drawn in question during
Hierarchy of courts
the trial of the cause in the court below.
This brings us to the issue of whether petitioners violated the doctrine of hierarchy of courts in directly filing their petition before this court.
Two questions are raised by this appeal: (1) Is Ordinance No. 35 valid, and (2) is this court required under the law to examine the evidence for the purpose of determining the guilt or
Respondents contend that petitioners’ failure to file the proper suit with a lower court of concurrent jurisdiction is sufficient ground for the dismissal of their petition. They add that observation
innocence of the defendants?
of the hierarchy of courts is compulsory, citing Heirs of Bertuldo Hinog v. Melicor. While respondents claim that while there are exceptions to the general rule on hierarchy of courts, none of
ISSUES:
these are present in this case.
Is the court required under the law to examine the evidence for the purpose of determining the guilt or innocence of the defendants?
The doctrine that requires respect for the hierarchy of courts was created by this court to ensure that every level of the judiciary performs its designated roles in an effective and efficient
RULING:
manner.
In a long line of cases the Court had constantly ruled that upon appeal the only question to be considered will be that of the validity or invalidity of an ordinance. The Court cannot review the
Trial courts do not only determine the facts from the evaluation of the evidence presented before them. They are likewise competent to determine issues of law which may include the validity
evidence nor pass upon any other question of law which may appear in the record."
of an ordinance, statute, or even an executive issuance in relation to the Constitution.
The court has not, since its organization, held in any case that it has the power to review the facts touching the guilt of an accused person in cases of the character of the one under
The Court of Appeals is primarily designed as an appellate court that reviews the determination of facts and law made by the trial courts.
consideration.
This court, on the other hand, leads the judiciary by breaking new ground or further reiterating — in the light of new circumstances or in the light of some confusions of bench or bar —
existing precedents.
Some discussion has arisen in regard to the language we should use in the final disposition of cases wherein the statute or ordinance has been upheld. Sometimes we say, "The judgment is
In other words, the Supreme Court’s role to interpret the Constitution and act in order to protect constitutional rights when these become exigent should not be emasculated by the doctrine
affirmed," and at other times we have said "the appeal is dismissed," etc. The result is the same and it is of little importance which expression we use. But, as the case comes to us on
in respect of the hierarchy of courts. That has never been the purpose of such doctrine.
appeal for the purpose of testing the legality of the statute or ordinance upon which the judgment rests and as the judgment cannot be executed without the sanction of this court, it is
Thus, the doctrine of hierarchy of courts is not an iron-clad rule. This court has "full discretionary power to take cognizance and assume jurisdiction [over] special civil actions for
perfectly legal to "affirm" or "reverse" the judgment as the case may be.
certiorari . . .filed directly with it for exceptionally compelling reasons or if warranted by the nature of the issues clearly and specifically raised in the petition."
For the foregoing reasons the judgment appealed from is affirmed, with costs against the defendants. So ordered.
As correctly pointed out by petitioners, we have provided exceptions to this doctrine:
First, a direct resort to this court is allowed when there are genuine issues of constitutionality that must be addressed at the most immediate time.
Second is when the issues involved are of transcendental importance. In these cases, the imminence and clarity of the threat to fundamental constitutional rights outweigh the
necessity for prudence.
Third, cases of first impression warrant a direct resort to this court. In cases of first impression, no jurisprudence yet exists that will guide the lower courts on this matter.
Fourth, the constitutional issues raised are better decided by this court.
Fifth, the time element presented in this case cannot be ignored .
THE DIOCESE OF BACOLOD, et al Petitioners,
Sixth, the filed petition reviews the act of a constitutional organ . COMELEC is a constitutional body.
vs.
Seventh, petitioners rightly claim that they had no other plain, speedy, and adequate remedy in the ordinary course of law that could free them from the injurious effects of
COMMISSION ON ELECTIONS, et al Respondents.
respondents’ acts in violation of their right to freedom of expression.
G.R. No. 205728, January 21, 2015
Eighth, the petition includes questions that are "dictated by public welfare and the advancement of public policy, or demanded by the broader interest of justice, or the
Topic: Heirarchy of Courts; Exceptions
orders complained of were found to be patent nullities, or the appeal was considered as clearly an inappropriate remedy."
Thus, the doctrine of hierarchy of courts is not an iron-clad rule. This court has "full discretionary power to take cognizance and assume jurisdiction [over] special civil actions for
It is not, however, necessary that all of these exceptions must occur at the same time to justify a direct resort to this court. While generally, the hierarchy of courts is respected, the present
certiorari . . .
case falls under the recognized exceptions and, as such, may be resolved by this court directly.
After a long discussion on the subject the Court later admitted that the tarpaulin in question is a form of free speech by stating:
xxx
FACTS: On February 21, 2013, petitioners posted two (2) tarpaulins within a private compound housing the San Sebastian Cathedral of Bacolod. The first tarpaulin contains the message This is a form of speech hopeful of a quality of democracy that we should all deserve. It is protected as a fundamental and primordial right by our Constitution. The expression in the medium
"IBASURA RH Law" referring to the Reproductive Health Law of 2012. The second tarpaulin is the subject of the present case. This tarpaulin contains the following: chosen by petitioners deserves our protection.
WHEREFORE, the instant petition is GRANTED. The temporary restraining order previously issued is hereby made permanent. The act of the COMELEC in issuing the assailed notice
dated February 22, 2013 and letter dated February 27, 2013 is declared unconstitutional.
TEAM BUHAY TEAM PATAY

Estrada, JV Angara, Juan Edgardo

Honasan, Gregorio Casiño, Teddy

Magsaysay, Mitos Cayetano, Alan Peter


EDITHA PADLAN, Petitioner,
vs.
ELENITA DINGLASAN and FELICISIMO DINGLASAN, Respondents.
Pimentel, Koko Enrile, Jackie G.R. No. 180321, March 20, 2013
Topic: Jurisdiction; determined on the allegations
In order to determine which court has jurisdiction over the action, an examination of the complaint is essential. Basic as a hornbook principle is that jurisdiction over the subject matter of a
Trillanes, Antonio Escudero, Francis case is conferred by law and determined by the allegations in the complaint which comprise a concise statement of the ultimate facts constituting the plaintiff's cause of action. The nature
of an action, as well as which court or body has jurisdiction over it, is determined based on the allegations contained in the complaint of the plaintiff, irrespective of whether or
not the plaintiff is entitled to recover upon all or some of the claims asserted therein.
Villar, Cynthia Hontiveros, Risa FACTS: Elenita Dinglasan (Elenita) was the registered owner of a parcel of land. While on board a jeepney, Elenita’s mother, Lilia Baluyot (Lilia), had a conversation with one Maura Passion
(Maura) regarding the sale of the said property. Believing that Maura was a real estate agent, Lilia borrowed the owner’s copy of the TCT from Elenita and gave it to Maura. Maura then
subdivided the property into several lots under the name of Elenita and her husband Felicisimo Dinglasan (Felicisimo).
Through a falsified deed of sale bearing the forged signature of Elenita and her husband Felicisimo, Maura was able to sell the lots to different buyers. On April 26, 1990, Maura sold a parcel Concomitant with the filing of a notice of appeal is the payment of the required appeal fees within the 15-day reglementary period set forth in Section 4 of Rule 41.
to one Lorna Ong (Lorna), who later sold the lot to petitioner Editha Padlan. Respondent’s claim that his non-payment of docket and other lawful fees should be treated as mistake and excusable negligence, attributable to the RTC Branch Clerk of Court, is too
After learning what had happened, respondents demanded petitioner to surrender possession of the bought portion of lot, but the latter refused. Respondents were then forced to file a case. superficial to warrant consideration. This is clearly negligence of respondent's counsel, which is not excusable. Negligence to be excusable must be one which ordinary diligence and
Summons was, thereafter, served to petitioner through her mother, Anita Padlan. prudence could not have guarded against.
Petitioner claimed that the court did not acquire jurisdiction over her, because the summons was not validly served upon her person, but only by means of substituted service through her The CA took cognizance over the case, based on the wrong premise that when the RTC issued the Order giving due course to respondent’s Notice of Appeal and directing the Branch Clerk
mother. Petitioner maintained that she has long been residing in Japan after she married a Japanese national and only comes to the Philippines for a brief vacation once every two years. of Court to transmit the entire records of the case to the CA, it ipso facto lost jurisdiction over the case. Section 9, Rule 41 of the Rules explains that the court of origin loses
On April 5, 2001, the RTC issued an Order denying petitioner’s motion to dismiss and declared her in default. Thereafter, trial ensued. jurisdiction over the case only upon the perfection of the appeal filed in due time by the appellant and the expiration of the time to appeal of the other parties. Withal, prior to the
RTC Ruling: transmittal of the original records of the case to the CA, the RTC may issue orders for the protection and preservation of the rights of the prevailing party, as in this case, the issuance of the
On July 1, 2005, the RTC rendered a Decision finding petitioner to be a buyer in good faith and, consequently, dismissed the complaint. writ of execution because the respondent’s appeal was not perfected.
CA Ruling: Since respondent’s appeal was not perfected within the 15-day reglementary period, it was as if no appeal was actually taken. Therefore, the RTC retains jurisdiction to rule on pending
On June 29, 2007, the CA rendered a Decision in favor of the respondent. incidents lodged before it, such as the petitioner’s Motion for Reconsideration, to Dismiss Appeal, and for Issuance of Writ of Execution. Having no jurisdiction over the case, the prudent
Aggrieved, petitioner filed a Motion for Reconsideration. Petitioner argued that not only did the complaint lacks merit, the lower court failed to acquire jurisdiction over the subject matter of thing that the CA should have done was to dismiss the respondent’s appeal for failure to pay the appeal fees, and declare that the RTC Decision dated June 28, 2002 has now become final
the case and the person of the petitioner. and executory.
Petitioner posits amongst others that the court lacks jurisdiction of the subject matter, considering that from the complaint, it can be inferred that the value of the property was only To recapitulate, one who seeks to avail of the right to appeal must strictly comply with the requirements of the rules, and failure to do so leads to the loss of the right to appeal. The rules
₱4,000.00, which under the BP 129 the MTC has jurisdiction to resolve. require that from the date of receipt of the assailed RTC order denying one’s motion for reconsideration, an appellant may take an appeal to the CA by filing a notice of appeal with the RTC
Note: Respondents filed their Complaint with the RTC; hence, before proceeding any further with any other issues raised by the petitioner, it is essential to ascertain whether the and paying the required docket and other lawful fees with the RTC Branch Clerk of Court, within the 15-day reglementary period for the perfection of an appeal. Otherwise, the appellant's
RTC has jurisdiction over the subject matter of this case based on the above-quoted provisions. appeal is not perfected, and the CA may dismiss the appeal on the ground of non-payment of docket and other lawful fees. As a consequence, the assailed RTC decision shall become final
and executory and, therefore, the prevailing parties can move for the issuance of a writ of execution.
While every litigant must be given the amplest opportunity for the proper and just determination of his cause, free from the constraints of technicalities, the failure to perfect an appeal within
ISSUE: the reglementary period is not a mere technicality. It raises jurisdictional problem, as it deprives the appellate court of its jurisdiction over the appeal. After a decision is declared final and
WON the RTC has jurisdiction over the subject matter. executory, vested rights are acquired by the winning party. Just as a losing party has the right to appeal within the prescribed period, the winning party has the correlative right to enjoy the
finality of the decision on the case.
RULING: WHEREFORE, the petition is GRANTED.

In order to determine which court has jurisdiction over the action, an examination of the complaint is essential. Basic as a hornbook principle is that jurisdiction over the subject matter of a
case is conferred by law and determined by the allegations in the complaint which comprise a concise statement of the ultimate facts constituting the plaintiff's cause of action. The nature of
an action, as well as which court or body has jurisdiction over it, is determined based on the allegations contained in the complaint of the plaintiff, irrespective of whether or not TITLE: HERRERA-FELIX VS. COURT OF APPEALS
the plaintiff is entitled to recover upon all or some of the claims asserted therein. CITATION: G.R. No. 143736. August 11, 2004.
Respondents’ Complaint narrates: TOPIC: JURISDICTION OVER THE PERSON OF THE DEFENDANT; BY VOLUNTARY APPEARANCE
- That they are the duly registered owners of the disputed lot.
FACTS
- Without their knowledge and consent, the land was divided into several lots under their names through the fraudulent manipulations of Maura. Original complainant:
St. Joseph Resource Development, Inc. (now respondent)

- One of the lots was by Maura to Lorna who later sold the lot to petitioner for ₱4,000.00. Complaint:
Sum of money against the Spouses Restituto and Ofelia Felix with a prayer for a writ of preliminary attachment.

Cause of action:
- That despite demands from the respondents, petitioner refused to surrender possession of the subject property. During the period from November 16, 1992 to December 14, 1992, the Felix Spouses purchased from the respondent tubs of assorted fish. Balance payable P1,132,065.50.
RTC
Decision in favor of St. Joseph Resource Development, Inc. RTC also granted the writ of preliminary attachment on a bond of P1,132,065.50 (the amount of balance).
Where the ultimate objective of the plaintiffs is to obtain title to real property, it should be filed in the proper court having jurisdiction over the assessed value of the property subject thereof.
Since the amount alleged in the Complaint by respondents for the disputed lot is only ₱4,000.00, hence under Sec. 33 of BP 129 the MTC and not the RTC has jurisdiction over the action. Copies of the writ of preliminary attachment, summons and complaint were served to the spouses at their residence, through the sister of Ofelia Herrera-Felix, Ma. Luisa
Therefore, all proceedings in the RTC are null and void. Herrera. (Ofelia was out of the country)
Consequently, the remaining issues raised by petitioner need not be discussed further.
WHEREFORE, the petition is GRANTED. The Decision of the Court of Appeals in CA-G.R. CV No. 86983, dated June 29, 2007, and its Resolution dated October 23, 2007, are REVERSED Spouses Felix prayed for an extension of time to file their answer – GRANTED!
and SET ASIDE. The Decision of the Regional Trial Court, dated July I, 2005, is declared NULL and VOID. The complaint in Civil Case No. 438-ML is dismissed without prejudice. Spouses Felix failed to file their answer. So St. Joseph Resource Development, Inc. filed a motion to declare said spouses in default – GRANTED!

A copy of the resolution was sent to and received by the counsel of the Felix Spouses through registered mail. The copy of the decision addressed to the spouses was
UGUSTUS GONZALES and spouses NESTOR victor and MA. LOURDES RODRIGUEZ, Petitioners,
returned to the court after two notices for having been "Unclaimed." However, then counsel for the Felix Spouses received his copy of the decision.
vs.
QUIRICO PE, Respondent.
No appeal was made. The motion for a writ of execution was filed and a copy was served to the spouses by registered mail but they failed to oppose the motion.
G.R. No. 167398, August 9, 2011
Topic: Jurisdiction; Docket Fee
The court thereafter issued an order granting the motion and directing the issuance of a writ of execution. The counsel for the Felix Spouses received a copy of the said order.
Section 9, Rule 41 of the Rules explains that the court of origin loses jurisdiction over the case only upon the perfection of the appeal filed in due time by the appellant and the expiration of
the time to appeal of the other parties.
Personal properties were levied and sold at a public auction. St. Joseph Resource Development, Inc. won as the highest bidder.
While every litigant must be given the amplest opportunity for the proper and just determination of his cause, free from the constraints of technicalities, the failure to perfect an appeal within
the reglementary period is not a mere technicality. It raises jurisdictional problem, as it deprives the appellate court of its jurisdiction over the appeal. After a decision is declared final and COURT OF APPEALS
executory, vested rights are acquired by the winning party. Just as a losing party has the right to appeal within the prescribed period, the winning party has the correlative right to enjoy the The wife, Ofelia Herrera-Felix, filed a petition under Rule 47 of the Rules of Court for the nullification of the trial court's judgment by default, the writ of execution issued by the said court, and
finality of the decision on the case. the sale of her properties at public auction.
Basis: The complaint and summons were handed over to her sister who was merely a visitor in her house and, as such, was not a valid substituted service under Rule 14,
FACTS: Section 7 of the Rules of Court.
Respondent Quirico Pe was engaged in the business of construction materials and had been in the business with petitioner spouses Rodriguez. Petitioners were then awarded by DPWH
two contracts and availed of DPWH’s pre-payment program for cement requirement regarding the projects, wherein the DPWH would give an advance payment even before project St. Joseph Resource averred that even if such substituted service on the petitioner was defective, the defect was cured when the latter, through her counsel, appeared in court
completion. Petitioner then gave the blank LBP check to respondent to guarantee payment of 15,698 bags of Portland cement. However a year later respondent filled up the blank LBP and moved for an extension of time to file her responsive pleading.
Check by placing P2,062,000.00. The petitioners then filed a case against the respondent.
RTC CA decided in favor of St. Joseph.
Ruled in favor of the petitioner ISSUE
Respondent filed a Notice of Appeal Whether or not the appearance of the spouses’ counsel constitutes a voluntary submission to the jurisdiction of the court
Petitioners filed a Motion for Reconsideration, to Dismiss Appeal, and for Issuance of Writ of Execution, stating that respondent’s appeal should be dismissed as the same was not perfected
due to non-payment of docket and other lawful fees RULING
The trial court dismissed respondent's appeal and directed the issuance of a writ of execution to implement the RTC Decision Yes.
CA
Rendered a decision in favor of the respondent The court acquires jurisdiction over the person of the defendant:
3. by service of the complaint and summons on him, either:
a. by personal service or
b. by substituted service or
Petitioner’s averments: c. by extra-territorial service thereof; or
Petitioners allege that since respondent failed to pay the docket and other legal fees at the time he filed the Notice of Appeal, his appeal was deemed not perfected in contemplation of the 4. by his voluntary personal appearance before the court or through counsel.
law.
Respondent’s: In this case, the petitioner appeared before the court, through counsel, and filed a motion for extension of time to file her answer to the complaint which the trial court granted. By filing the
Respondent maintains that his appeal has been perfected by the mere filing of the notice of appeal. Respondent theorizes that with the perfection of his appeal, the trial court is now divested said motion, through counsel, the petitioner thereby submitted herself to the jurisdiction of the trial court.
of jurisdiction to dismiss his appeal and, therefore, only the CA has jurisdiction to determine and rule on the propriety of his appeal.
A voluntary appearance is a waiver of the necessity of a formal notice.
ISSUE: An appearance in whatever form, without explicitly objecting to the jurisdiction of the court over the person, is a submission to the jurisdiction of the court over the
WON the CA acquired jurisdiction over the case. person.
RULING: Formal method of entering an appearance
 deliver to the clerk a written direction ordering him to enter the appearance of the person who subscribes it
 an appearance may be made by simply filing a formal motion, or plea or answer.
The petition is meritorious.
This formal method of appearance is not necessary. He may appear without such formal appearance and thus submit himself to the jurisdiction of the court. Whether or not the RTC acquired jurisdiction over the person of the Villas

He may appear by presenting a motion, for example, and unless by such appearance he specifically objects to the jurisdiction of the court, he thereby gives his assent to the jurisdiction of RULING
the court over his person. When the appearance is by motion objecting to the jurisdiction of the court over his person, it must be for the sole and separate purpose of objecting to the Yes.
jurisdiction of the court. If his motion is for any other purpose than to object to the jurisdiction of the court over his person, he thereby submits himself to the jurisdiction of the court.
It is settled that if there is no valid service of summons, the court can still acquire
jurisdiction over the person of the defendant by virtue of the latter's voluntary appearance. Thus Section 20 of Rule 14 of the Rules of Court provides:
Sec. 20. Voluntary appearance. — The defendant's voluntary appearance in the action shall be equivalent to service of summons. The inclusion in a motion to dismiss of
TITLE: JAPRL DEV’T CORP., ET AL. VS. SECURITY BANK CORP. other grounds aside from lack of jurisdiction over the person shall not be deemed a voluntary appearance.
CITATION: G.R. No. 190107. June 6, 2011.
TOPIC: JURISDICTION OVER THE PERSON OF THE DEFENDANT; BY VOLUNTARY APPEARANCE Prescinding from the foregoing, it is thus clear that:
(1) Special appearance operates as an exception to the general rule on voluntary appearance;
FACTS (2) Accordingly, objections to the jurisdiction of the court over the person of the defendant must be explicitly made, i.e., set forth in an unequivocal manner; and
JAPRL – domestic corp engaged in fabrication, manufacture and distribution of steel products (3) Failure to do so constitutes voluntary submission to the jurisdiction of
the court, especially in instances where a pleading or motion seeking affirmative relief is filed and submitted to the court for resolution.
JAPRL applied for a credit facility for P50M with Security Bank Corp. (SBC) – approved!
The Villas did not, in their first motion to lift the order of default, allege that their filing thereof was a special appearance for the purpose only to question the jurisdiction over their persons.
JAPRL Chairman and President (Limson and Arollado) executed a Continuing Suretyship Agreement (CSA) 2 in favor of SBC wherein they guaranteed the due and full payment and Clearly, they had acquiesced to the jurisdiction of the court.
performance of JAPRL's guaranteed obligations under the credit facility.

Subsequently, JAPRL's financial adviser, MRM Management Incorporated (MRM), convened JAPRL's creditors, SBC included, for the purpose of restructuring JAPRL's TITLE: Allan Go v. Cordero
existing loan obligations. Copies of JAPRL's financial statements (FS) from 1998 to 2001 were given for the creditors to study. CITATION: G.R. No. 164703, May 4, 2010
TOPIC: Jurisdiction over the defendant - by voluntary appearance
SBC soon discovered material inconsistencies in the FS given by MRM vis-Ã -vis those submitted by JAPRL when it applied for a credit facility, drawing SBC to conclude that JAPRL
committed misrepresentation. FACTS
Mortimer F. Cordero, Vice-President of Pamana Marketing Corporation (Pamana), ventured into the business of marketing inter-island passenger vessels.
As per agreement, any misrepresentation will constitute an event of default committed by JAPRL and its sureties.
After contacting various overseas fast ferry manufacturers from all over the world, he came to meet Tony Robinson, an Australian national based in Brisbane, Australia, who is the Managing
SBC sent a letter of demand to JAPRL and its chairman and president. Because they failed to comply with the demand, SBC filed a complaint for sum of money with application for issuance Director of Aluminium Fast Ferries Australia (AFFA).
of writ of preliminary attachment before the Regional Trial Court (RTC) of Makati City.
RTC Robinson signed documents appointing Cordero as the exclusive distributor of AFFA catamaran and other fast ferry vessels in the Philippines. As such exclusive distributor, Cordero offered
The Makati RTC at ordered in open court the archiving of SBC's complaint for sum of money until disposition by the Quezon City RTC of JAPRL's petition for rehabilitation. for sale to prospective buyers the 25-meter Aluminium Passenger catamaran known as the SEACAT 25.

Consequently, RTC dismissed SBC’s complaint without prejudice. After negotiations with the lawyers of Allan Go (owner/operator of ACG Express Liner of Cebu City), Cordero was able to close a deal for the purchase of two (2) SEACAT 25.

SBC filed a motion for reconsideration stating that the suspension of the proceedings should only be with respect to JAPRL but not with respect to the chairman and president. – Denied! However, Cordero later discovered that Go was dealing directly with Robinson when he was informed by Dennis Padua of Wartsila Philippines that Go was canvassing for a second
catamaran engine from their company which provided the ship engine for the first SEACAT 25.
SBC filed another MR. – denied!
Cordero filed a case seeking to hold Robinson, Go, Tecson and Landicho liable jointly and solidarily for conniving and conspiring together in violating his exclusive distributorship in bad faith
Limson and Arollado opposed, claiming that summons were not served on them, hence, the Makati RTC failed to acquire jurisdiction over their person. and wanton disregard of his rights, thus depriving him of his due commissions.
COURT OF APPEALS Robinson filed a motion to dismiss grounded on lack of jurisdiction over his person, among others. – denied!
The appellate court held that Limson and Arollado voluntarily submitted themselves to the jurisdiction of the Makati RTC, despite the qualification that the filing of their respective
"Opposition[s] Ad Cautelam" and "Manifestation[s] Ad Cautelam," was "by way of special appearance" they having sought affirmative relief by praying for the archiving of SBC's complaint. RTC decided in favor of Cordero.
Limson and Arollado filed a motion for recon – denied! Defendants filed a petition for certiorari in the CA.

ISSUE ISSUE
Whether or not the court has acquired jurisdiction over Limson and Arollado Whether or not the court has jurisdiction over the person of Robinson

RULING RULING
Yes. Yes.

When a defendant's appearance is made precisely to object to the jurisdiction of Courts acquire jurisdiction over the plaintiffs upon the filing of the complaint, while jurisdiction over the defendants in a civil case is acquired either through the service of summons upon
the court over his person, it cannot be considered as appearance in court. them in the manner required by law or through their voluntary appearance in court and their submission to its authority.

Limson and Arollado glossed over the alleged lack of service of summons, however, and proceeded to exhaustively discuss why SBC's complaint could not prosper against them as sureties. A party who makes a special appearance in court challenging the jurisdiction of said court based on the ground of invalid service of summons is not deemed to have submitted himself to the
They thereby voluntarily submitted themselves to the jurisdiction of the Makati RTC. jurisdiction of the Court.

In this case, however, although the Motion to Dismiss filed by Robinson specifically stated as one of the grounds the lack of "personal jurisdiction," it must be noted that he had earlier filed a
TITLE: RAPID CITY REALTY & DEV’T CORP. VS. ORLANDO VILLA and LOURDES PAEZ-VILLA
Motion for Time to file an appropriate responsive pleading even beyond the time provided in the summons by publication. ( guys sorry wa jud ni sya sa facts ang summons by publication )
CITATION: G.R. No. 184197. February 11, 2010.
Such motion did not state that it was a conditional appearance entered to question the regularity of the service of summons, but an appearance submitting to the jurisdiction of the court by
TOPIC: JURISDICTION OVER THE PERSON OF THE DEFENDANT; BY VOLUNTARY APPEARANCE
acknowledging the summons by publication issued by the court and praying for additional time to file a responsive pleading.
FACTS
Consequently, Robinson having acknowledged the summons by publication and also having invoked the jurisdiction of the trial court to secure af firmative relief in his motion for additional
RCRDC filed a complaint for declaration of nullity of subdivision plans, mandamus and damages against several defendants including Spouses Orlando and Lourdes Villa.
time, he effectively submitted voluntarily to the trial court's jurisdiction. He is now estopped from asserting otherwise, even before this Court.
After one failed attempt at personal service of summons, the court process server, resorted to substituted service by serving summons upon respondents' househelp who did not TTITLE: Lhuillier v. British Airways
acknowledge receipt thereof and refused to divulge their names.
CITATION: G.R. No. 171092. March 15, 2010.
Despite substituted service, the Villas failed to file their Answer, prompting RCRDC to fle a "Motion to Declare Defendants in Default". – Granted!
TOPIC: Jurisdiction over the defendant - by voluntary appearance
More than eight months thereafter, the Villas filed a Motion to Lift Order of Default, claiming that on they have just received the documents recently and they denied the existence of two
FACTS
women helpers who allegedly refused to sign and acknowledge receipt of the summons. In any event, they contended that assuming that the allegation were true, the helpers had no
Petitioner Edna Diago Lhuillier took respondent's flight 548 from London to Rome. Once on board, she allegedly requested Julian Halliday (Halliday), one of the respondent's flight
authority to receive the documents.
attendants, to assist her in placing her hand-carried luggage in the overhead bin. However, Halliday allegedly refused to help and assist her, and even sarcastically remarked that "If I were to
help all 300 passengers in this flight, I would have a broken back!"
RTC set aside the Order of Default and gave them 5 days to file their Answer. Respondents just the same did not file an Answer, drawing petitioner to again file a Motion to declare them in
default, which the trial court again granted.
Petitioner further alleged that when the plane was about to land in Rome another flight attendant, Nickolas Kerrigan (Kerrigan), singled her out from among all the passengers in the
business class section to lecture on plane safety. Allegedly, Kerrigan made her appear to the other passengers to be ignorant, uneducated, stupid, and in need of lecturing on the safety rules
The Villas filed an Omnibus Motion for reconsideration of the second order declaring them in default and to vacate proceedings, this time claiming that the trial court did not
and regulations of the plane. Affronted, petitioner assured Kerrigan that she knew the plane's safety regulations being a frequent traveler. Thereupon, Kerrigan allegedly thrust his face a
acquire jurisdiction over their persons due to invalid service of summons. – denied!
mere few centimeters away from that of the petitioner and menacingly told her that "We don't like your attitude."
Filed certiorari in CA.
Upon arrival in Rome, petitioner complained to respondent's ground manager and
COURT OF APPEALS demanded an apology. However, the latter declared that the flight stewards were "only doing their job."
CA annulled the trial court's Orders declaring respondents in default for the second time.
Complaint
Basis: The Villas focused all their energies on questioning the RTC’s jurisdiction. The latter motion clearly stated prefatorily their counsel's reservation or "special Petitioner filed a Complaint for damages against respondent British Airways before the RTC of Makati City.
appearance to question jurisdiction" over the persons of the petitioners. "A party
who makes a special appearance in court challenging the jurisdiction of said court based on the ground of invalid service of summons is not deemed to have Summons, together with a copy of the complaint, was served on the respondent through Violeta Echevarria, General Manager of Euro-Philippine Airline Services, Inc.
submitted himself to the jurisdiction of the court."
Respondent’s Answer
RCRDC filed a petition for review in the SC. Respondent, by way of special appearance through counsel, filed a Motion to Dismiss on grounds of lack of jurisdiction over the case and over the person of the respondent.

ISSUE
Respondent alleged that only the courts of London, United Kingdom or Rome, Italy, have jurisdiction over the complaint for damages pursuant to the Warsaw Convention, 5 Article 28 (1) of
which provides:
An action for damages must be brought at the option of the plaintiff, either before the court of domicile of the carrier or his principal place of business, or where he has a
place of business through which the contract has been made, or before the court of the place of destination.

Thus, since a) respondent is domiciled in London; b) respondent's principal place


of business is in London; c) petitioner bought her ticket in Italy; and d) Rome, Italy is petitioner's place of destination, then it follows that the complaint should only be filed in the proper courts
of London, United Kingdom or Rome, Italy.

Likewise, it was alleged that the case must be dismissed for lack of jurisdiction over the person of the respondent because the summons was erroneously served on Euro-
Philippine Airline Services, Inc. which is not its resident agent in the Philippines. – Granted! Basis: Our Courts have to apply the principles of international law, and are bound by treaty
stipulations entered into by the Philippines which form part of the law of the land. One of this is the Warsaw Convention.
Petitioner filed an MR – denied! ; filed a certiorari in SC.

ISSUE
Whether or not the court has jurisdiction over the person of the defendant/respondent (British Airways)

RULING
No.

Respondent, in seeking remedies from the trial court through special appearance of counsel, is not deemed to have voluntarily submitted itself to the jurisdiction of the trial court.

The second sentence of Sec. 20, Rule 14 of the Revised Rules of Civil Procedure clearly provides:
Sec. 20. Voluntary appearance. — The defendant's voluntary appearance in the action shall be equivalent to service of summons. The inclusion in a motion to dismiss of
other grounds aside from lack of jurisdiction over the person of the defendant shall not be deemed a voluntary appearance.

In this case, the special appearance of the counsel of respondent in filing the Motion to Dismiss and other pleadings before the trial court cannot be deemed to be voluntary submission to
the jurisdiction of the said trial court.
TITLE: Phil. Women’s Christian Temperance Union v. Yangco

CITATION: G.R. No. 199595, April 2, 2014.


TOPIC: Payment of filing/docket fees- jurisdictional

FACTS
Teodoro R. Yangco (2nd and 3rd Generation Heirs) Foundation, Inc. (TRY Foundation) filed before the RTC of Quezon City, acting as a Land Registration Court, a Petition for the Issuance
of New Title in Lieu of Transfer Certificate of Title (TCT) No. 20970 T-22702 of the Office of the Register of Deeds of Quezon City.

The property was previously donated to PWCTUI with a condition that should the property be used for any other purpose/s not specified, the gift shall become ipso facto null and void and
property given shall automatically revert to the donor, his heirs and assigns.

PWCTUI’s corporate term expired in September 1979. TRY Foundation claimed that the expiration of PWCTUI’s corporate term in 1979 effectively rescinded the donation.

PWCTUI claimed, among others, that the RTC has no jurisdiction over the petition because PWCTUI is unaware of its publication. – denied!

CA affirmed the RTC’s decision.

PWCTUI sought recourse with the Court thru a petition for review on certiorari.

ISSUE
Whether or not the RTC has acquired jurisdiction over the case

RULING
No.

RTC judgment in the land registration case and all proceedings taken in relation thereto were void because the RTC did not acquire jurisdiction over the fundamental subject matter of TRY
Foundation’s petition for the issuance of a title which was in reality, a complaint for revocation of donation. No judgment proclaiming TRY Foundation as the absolute owner of the property
can be arrived at without declaring the deed of donation revoked.

The issues embroiled in revocation of donation are litigable in an ordinary civil proceeding which demands stricter jurisdictional requirements than that imposed in a land registration case.

The payment of docket fees is another jurisdictional requirement for an action for revocation which was absent in the suit filed by TRY Foundation. On the other hand, Section
111 of P.D. No. 1529 merely requires the payment of filing fees and not docket fees.

Filing fees are intended to take care of court expenses in the handling of cases in terms of cost of supplies, use of equipment, salaries and fringe benefits of personnel, etc., computed as to
man hours used in handling of each case. Docket fees, on the other hand, vest the trial court jurisdiction over the subject matter or nature of action.

The absence of the jurisdictional requirements for ordinary civil actions thus prevented the RTC, acting as a land registration court, from acquiring the power to hear and decide the
underlying issue of revocation of donation in the land registration case.

Das könnte Ihnen auch gefallen